N400 Exam 1 (PrepU Questions)

अब Quizwiz के साथ अपने होमवर्क और परीक्षाओं को एस करें!

What percentage of weight change in 6 months is considered abnormal? a) 2% b) 1% c) 5% d) 10%

d) 10%

23. A client stops in the hall after walking 30 ft (9 m) and tells the nurse, "I don't want to do any more exercise because I hurt too much." What is the next action the nurse should implement?

Return the client to bed and provide pain relief measures.

The nurse observes the client's frequent use of the incentive spirometer. The client states "I do not want to have pneumonia while in the hospital." Which vital sign reading demonstrates effectiveness of this intervention?

Temperature of 98.2°F (36.7°C)

Which factor is not known to cause false blood pressure readings?

being in a warm environment

During an initial assessment, the nurse measures the client's apical pulse and compares it to the peripheral pulse. The difference between the two is known as pulse:

deficit

What organ is the primary site of heat loss in the body?

skin

The nurse going back to school for nurse midwifery can trace education for nurse midwifery to which nursing leader? A. Mary Breckinridge B. Sojourner Truth C. Lavinia Dock D. Margaret Sanger

A. Mary Breckinridge

A nurse is educating a postoperative adult client about taking daily temperatures. What statement by the client best indicates understanding of education?

"If my temperature is above 99.6°F( 38.3°C) I should call the health care provider."

34. Which nursing diagnosis will the nurse rank as the priority for premature newborn twins?

Altered Gas Exchange

The nurse is teaching the client about a balanced and nutritious diet. Which dietary choices selected by the client indicate an understanding of dietary needs?

Baked potato, salad with olive oil dressing, and lean steak

After taking vital signs of an older adult, the nurse writes down findings as T = 98.9/37.2 oral, P = 104, R = 18, BP = 130/82. Based on the collected data, which step would the nurse take next?

Take pulse again to assess for tachycardia

Which is the primary reason for a nurse collecting data continuously on a client?

The client's health status can change quickly.

The patient has a heart rate of 72 bpm with a regular rhythm. Where does the nurse determine the impulse arises from?

The sinoatrial node

Clients demonstrating apnea have what?

a temporary cessation of breathing

The nurse assesses that a client is shivering. Which intervention is most appropriate to prevent further stress on the body?

apply a blanket

A pulse deficit is the difference between:

the apical pulse and the radial pulse rates

A nurse is documenting a client's vital signs, height, and weight in the electronic health record. Applying the framework for informatics practice, the nurse would identify these values as which component of the framework? Data Information Knowledge Wisdom

Data

Which is the proper way to document midnight in a client's record? a. 1200 b. 1201 c. 2401 d. 0000

d. 0000

A nurse is working with a culturally diverse group of clients. The nurse understands that cultural norms: a. allow nurses to predict a client's response. b. can be generalized to anyone of that culture. c. are fairly consistent across cultural groups. d. require an individualized approach by the nurse.

d. require an individualized approach by the nurse.

Assessment of the pulse amplitude is accomplished by:

palpating the flow of blood through an artery

A pulse deficit is the difference between

the apical pulse and the radial pulse rates.

What assessment technique would the nurse use to assess a client's chest for color, shape, or contour? a) Auscultation b) Inspection c) Percussion d) Palpation

b) Inspection

A nurse is auscultating the lungs of a client. During the auscultation, the nurse hears high-pitched, harsh, blowing sounds over the larynx and trachea. The nurse identifies these sounds as which type? a) bronchovesicular breath sounds b) adventitious breath sounds c) bronchial breath sounds d) vesicular breath sounds

c) bronchial breath sounds

Upon auscultation of a client's lung fields, the nurse hears a continuous high-pitched sound on expiration. These are characteristics of which adventitious breath sound? a) pleural friction rub b) stertorous breathing c) wheezes d) fine crackles

c) wheezes

How do people of Canadian Indian descent prefer to be identified? a. Indians b. Americans c. Canadians d. First Nations

d. First Nations

Besides being an instrument of continuous client care, the client's health care record also serves as a(an): a. assessment tool. b. incident report. c. Kardex. d. legal document.

d. legal document.

When creating the teaching plan for a client who will be monitoring his or her pulse at home, which factors should the nurse teach the client that may influence the pulse rate by causing an increase in pulse? Select all that apply.

Exercise Fever Stress

4. A treatment based on a nurse's clinical judgment and knowledge to enhance client outcomes is a nursing:

Intervention.

A pregnant client asks the nurse for information on breastfeeding. What type of nursing diagnosis should the nurse formulate? An problem-focused nursing diagnosis A risk nursing diagnosis A possible nursing diagnosis A health promotion nursing diagnosis

A health promotion nursing diagnosis The client is seeking information related to healthy practices. Health promotion nursing diagnoses are formulated to assist the client to meet that need. The client has no health problem, risk of a health problem, or possible problem, so a problem-focused, risk, or possible nursing diagnosis would be inappropriate.

8. Which guideline should the nurse follow when including interventions in a plan of care?

Date the nursing interventions when written and when the plan of care is reviewed.

What must the nurse do to identify actual or potential health problems? a.Meet with significant others b.Call the physician c.Evaluate care implemented d.Gather data from sources

Gather data from sources

Which piece of client information is subjective? a.Generalized myalgia or muscle pain. b.Leukoplakia on the client's oral mucosa c.Alert and oriented to person and place but not time or situation d.A temperature of 102°F (38.9°C)

Generalized myalgia or muscle pain

A client is being admitted from the emergency room reporting shortness of breath, wheezing, and coughing. What would the nurse formulate as an appropriate nursing diagnosis? Bronchial Pneumonia Ineffective Airway Clearance Acute Dyspnea Asthma Attack

Ineffective Airway Clearance Because wheezing, shortness of breath, and coughing are signs of a constricted airway, the nursing diagnosis of Ineffective Airway Clearance is the appropriate diagnosis. Bronchial pneumonia and Asthma Attack are both medical diagnoses. Acute Dyspnea is a symptom.

The nurse is preparing to assess a client's abdomen. Arrange the steps of the assessment in the correct order. Inspection Auscultation Percussion Palpation

Inspection Auscultation Percussion Palpation

The sclerae of a 3-day-old infant have a yellowish tint, and the nurse has just received an order to initiate phototherapy. Which nursing diagnosis should the nurse use to plan care for this client? Risk for Neonatal Jaundice Neonatal Jaundice Risk for Visual Deficit Visual Deficit

Neonatal Jaundice

The nurse is assessing a client for changes in health condition. After listening to the client's lungs for adventitious breath sounds, the nurse also checks the client's latest white blood cell count. The nurse is gathering which type of data when looking up the lab value?

Objective

A nurse is reviewing the medical record of a client at the clinic. The nurse notes that the medication and dosage prescribed for the client was based on information gathered about the client's genetic makeup from the electronic health record. The nurse interprets this as: Genetics Genomics Pharmacogenomics Telehealth

Pharmacogenomics

An informatics nurse is assisting with the design of an clinical information system for use by the staff of a health center. The nurse is working to ensure that the system reflects usability by making sure that the screen display is visually clean and uncluttered and that it provides only the information needed for decision making. Which concept of usability is the nurse incorporating? Simplicity Naturalness Consistency Forgiveness

Simplicity

Which statement is true regarding the autonomic nervous system and its effect on the rate of a person's pulse?

Sympathetic nervous system activation occurs in response to a variety of stimuli, including changes in intravascular volume.

Which client would the nurse consider at risk for low blood pressure?

a client with low blood volume

The home care nurse notices that the client only has a glass thermometer. What is the best response by the nurse?

"Would you consider using a digital thermometer?"

Which theory describes, explains, predicts, and controls outcomes in nursing practice? A. Nursing theory B. Systems theory C. Adaptation theory D. Developmental theory

A. Nursing theory

Which nursing intervention would be most appropriate for a new mother that calls the nursery for help with breastfeeding? A. Refer the mother for a home care visit. B. Ask the mother to come to the emergency room. C. Email the mother a link for breastfeeding. D. Suggest that the mother bottle feed her infant.

A. Refer the mother for a home care visit.

Which types of knowledge are subjective? (Select all that apply.) A. Traditional knowledge B. Authoritative knowledge C. Scientific knowledge D. Evidence-based practice E. Scientific method

A. Traditional knowledge B. Authoritative knowledge

3. A nurse is caring for a client who began taking the antidepressant paroxetine 2 weeks ago. The client recently began giving away prized possessions and tells the nurse, "My mind is made up, I can't do this any longer." What is the best action by the nurse to incorporate this information into the plan of care?

Add the nursing diagnosis: Risk for Self-Harm.

Knowledge gained from someone with a great deal of perceived experience is which type of knowledge? A. Traditional knowledge B. Authoritative knowledge C. Scientific knowledge D. Philosophy knowledge

B. Authoritative knowledge

A group of objects with relationships is which? A. Theory B. Concept C. Deductive reasoning D. Inductive reasoning

B. Concept

What nursing activity forms the bridge between theory and practice? A. Theoretical writing B. Evidence-based research C. Client-focused care D. Case management

B. Evidence-based research

What was the focus of nursing research during the first half of the twentieth century,? A. Client care B. Nursing education C. Treatment issues D. Available nurse supply

B. Nursing education

What type of research study would a hospital conduct to explore clients' and families' perceptions of receiving care? A. Quantitative B. Qualitative C. Ordinal D. Nonscientific

B. Qualitative

Which is the best example of a client-centered approach to care? A. The nurse helps a client ambulate. B. The nurse asks the client about health goals. C. The nurse asks the client what the client would like to order from the menu. D. The nurse draws a blood sample from a client.

B. The nurse asks the client about health goals.

A nurse is discussing dietary issues with a Latino client in the clinic. The client states, "I read a research article about the importance of beans in the Latino diet. The findings from the study suggested that inclusion of meat and beans in the diet would help in the development of muscles and bones." The information that the client is expressing is known as what? A. Traditional knowledge B. Authoritative knowledge C. Scientific knowledge D. Philosophical knowledge

C. Scientific knowledge

29. A home care client with dementia has the nursing diagnosis "Wandering." Which expected client outcome most directly demonstrates resolution of the problem?

Client will not leave the premises without a caregiver.

What might a nurse need to do to ensure the continuation of his or her nursing license? A. Obtain a baccalaureate degree. B. Obtain a master's degree. C. Attend hospital in-services. D. Obtain continuing education credits.

D. Obtain continuing education credits.

An informatics nurse is evaluating a new clinical information system for usability. The nurse notes that the system requires the user to complete a maximum of 3 steps to complete a task. The system also provides shortcuts to frequent users of the system. The nurse would determine that which concept of usability is being addressed? Consistency Minimizing cognitive load Efficient interactions Naturalness

Efficient interactions

The nurse is performing bilateral comparison of pulse sites for strength and quality instead of counting the beats per minute. Which pulse locations will the nurse palpate to gather this assessment data? Select all that apply.

Femoral Dorsalis pedis Popliteal Posterior tibial

The nurse identifies the auscultatory gap while taking the client's blood pressure. What should the nurse do next?

Inflate the cuff about 30 mm Hg above the auscultatory gap.

What is the nurse accountable for, according to state nurse practice acts? Managing the care team effectively Making nursing diagnoses Prescribing PRN (as needed) medications Mentoring other nurses

Making nursing diagnoses State nurse practice acts have included diagnosis as part of the domain of nursing practice for which nurses are held individually accountable. Overall management of the care team is not an explicit responsibility of nurses. Nurses generally do not have prescriptive authority. The responsibility for mentorship is not enacted in law.

1. Which is most important for the nurse to include in a client's plan of care?

Nursing interventions

An informatics nurse specialist is involved with implementing strategies to improve the performance of the clinical information system being used. As part of this process, the nurse specialist is working on updating the plans of care in the system to reflect changes to a procedure based on new evidence. The nurse is also working to streamline the display screens to reduce the need to document the same information in three different areas. The nurse specialist is addressing which aspect of the system? Usability Optimization Standard terminology Interoperability

Optimization

The nurse needs to assess the carotid arteries of the client. Which assessment technique would be appropriate for the nurse to use?

Palpate one artery at a time.

32. When establishing client outcomes with the client, what is the qualifier in the outcome?

The outcome parameter

What is the purpose of establishing a nursing diagnosis? To describe a functional health problem To collaborate with the physician To identify medical problems To meet accreditation criteria

To describe a functional health problem

A nurse is reading a journal article about providing individualized care. Which aspect would the nurse most likely read about as the almost universally accepted method for providing nursing care? a. Nursing process b. Clinical reasoning c. Reflection d. Experience

a. Nursing process

Which nursing skill uses all five senses? a.Listening b.Observation c.Caring d.Documentation

b. Observation

A nurse examining the lungs of a client percusses over the anterior thorax using the proper sequence. This technique helps to identify: a) normal breath sounds. b) masses. c) density and location of lungs. d) muscle tenderness.

c) density and location of lungs.

A client says to the nurse, "Why don't you wear a white cap like nurses do on the soap operas?" This is an ethnocentric statement based on the: a. nursing personality. b. past history. c. media. d. genetics.

c. media

The nurse is caring for a 10-year-old client who is newly diagnosed with a seizure disorder. What variable would alter the nurse's plan for educating the client and parent? a) The client expresses a desire to learn how to manage the medication regime. b) The parents verbalize acceptance of the need to closely monitor their child's condition. c) The parents have comprehensive insurance coverage for their family's medical care. d) The client has a 12-year-old sister who has been treated for a seizure disorder for 3 years.

d) The client has a 12-year-old sister who has been treated for a seizure disorder for 3 years.

A nurse has administered 1 unit of glucose to the client as per order. What is the correct documentation of this information? a. 1 bottle of glucose b. 1U of glucose c. One U of glucose d. 1 Unit of glucose

d. 1 Unit of glucose

Care provided to a client following surgery and until discharge represents which phase of the nurse-client relationship? a) Termination phase b) Orientation phase c) Evaluation phase d) Working phase

d) Working phase

The nurse is taking the client's temperature. The nurse understands that the rectal route is one of the most reliable. Which client can safely handle the rectal route of taking temperature?

the 65-year old male who just finished drinking coffee

A 57-year-old male client is admitted to the medical unit with a 3-day history of sharp, nonradiating epigastric pain and vomiting. He denies seeing blood in his stool. When assessing this client's abdomen, what assessment technique would the nurse perform last? a) Auscultation b) Percussion c) Inspection d) Palpation

d) Palpation

7. A nurse administers an antihypertensive medication according to the standardized plan of care for a client admitted with uncontrolled hypertension. Which assessment information indicates the expected client outcome has been met within the first 24 hours?

Client is normotensive.

39. Which phase of the nursing process most involves establishing priorities?

Outcome identification and planning

The nurse is assessing the apical pulse of a client using auscultation. What action would the nurse perform after placing the diaphragm over the apex of the heart?

Listen for heart sounds.

A nurse has identified a risk nursing diagnosis for a client. When writing this diagnosis, the nurse would write a statement consisting of how many parts? One Two Three Four

Two

The nurse recognizes that health problems that the nurse can address by independent nursing interventions are called: dependent nursing diagnoses. actual or potential nursing diagnoses. collaborative nursing diagnoses. syndrome nursing diagnoses.

actual or potential nursing diagnoses.

The nurse knows that a client who is being taught to perform home blood pressure monitoring (HBPM) understands the teaching plan when he makes which statement about the size of the BP cuff? The cuff should:

fit snug around the upper arm with room to slip a fingertip under the cuff and should be 1 in (2.5 cm) above the crease of the elbow.

The nurse understands that accurate blood pressure taking is dependent on several factors. Which example will most likely render an accurate blood pressure reading?

placing the client's arm at heart level

The client's blood pressure has gradually decreased in the last 2 days. Which condition would cause this change?

the client who has had persistent diarrhea

26. What are specific measurable and realistic statements of goal attainment?

Outcomes

41. Which action should the nurse perform during the planning phase of the nursing process?

Identify measurable goals or outcomes.

One of the primary reasons for conducting nursing research is to: A. quantify outcomes related to clients. B. determine outcomes for clients. C. generate knowledge to guide practice. D. prevent further disease and death.

C. generate knowledge to guide practice.

The nurse is studying nursing models for a Baccalaureate course. Which action made by the nurse exemplifies proper application of the Adaptation model? A. alleviating the client's pain with medication B. letting the client perform activities of daily living independently C. helping the client adjust to blood sugar testing each morning D. teaching the client about the risks of asbestos

C. helping the client adjust to blood sugar testing each morning

When taking the client's temperature, the student nurse will require further education when they state:

"The axillary route is the most accurate of all routes."

Place the steps of evidence-based practice in the correct order.

1. Design a question related to a clinical area of interest. 2. Collect the most relevant and best evidence available. 3. Critically evaluate the collected evidence. 4. Integrate the evidence with clinical expertise, client preferences, and values as the decision is made to make a change. 5. Evaluate the decision or change.

To improve quality care for clients, there are four steps that the nurse recognizes as being crucial for the process. Place them in the correct order. Use all options. Evaluate a change Plan a strategy using indicators Implement a change Discover a problem

1. Discover a problem 2. Plan a strategy using indicators 3. Implement a change 4. Evaluate a change

The evolution of nursing research has significant markers in time. Place the following events in the correct order.

1. Florence Nightingale kept records while caring for victims of the Crimean War. 2. Nursing research was deemed to be increasingly important, and research was utilized for the development of nursing practice standards. 3. Clinical research became the emphasis of nursing research resulting in procedures for primary client care, such as vital signs and treatments. 4. The National Institute for Nursing Research funded research for the scientific basis for client care

The evolution of nursing research has significant markers in time. Place the following events in the correct order:

1. Florence Nightingale kept records while caring for victims of the Crimean War. 2. Nursing research was deemed to be increasingly important, and research was utilized for the development of nursing practice standards. 3. Clinical research became the emphasis of nursing research resulting in procedures for primary client care, such as vital signs and treatments. 4. The National Institute for Nursing Research funded research for the scientific basis for client care.

During the Christian era (AD 1-500), nursing care excluded which area? A. Nutrition B. Personal counseling C. Comfort measures D. Psychiatric services

D. Psychiatric services

The formulation of nursing diagnoses is unique to the nursing profession. Which statement accurately represents a characteristic of diagnosing? Nursing diagnoses remain the same for as long as the disease is present. Nurses formulate nursing diagnoses to identify diseases. Nurses write nursing diagnoses to describe client problems that nurses can treat. Nursing diagnoses focus on identifying healthy responses to health and illness.

Nurses write nursing diagnoses to describe client problems that nurses can treat.

11. A computerized information system developed to classify client outcomes is the:

Nursing Outcome Classification system

A nurse documents the following in the client chart: Risk for Decreased Cardiac Output related to myocardial ischemia. This is an example of what aspect of client care? Nursing diagnosis Nursing assessment Medical diagnosis Collaborative problem

Nursing diagnosis

A nurse working in a primary care provider's office is using the clinical information system to review a client's health information. The nurse is able to review the client's last visit to the primary care provider as well as information from a recent hospitalization, and also a visit that the client made to the cardiologist last week. The nurse's ability to review this information is based on which aspect of the clinical information system? Usability Optimization Interoperability Security

Interoperability

The nurse is caring for a client who ascribes to the theory of animism. When attempting to explain this theory to other staff members, the nurse should state: A. "Everything in nature is alive with invisible forces." B. "The physician is viewed as a god-like figure." C. "The nurse is the handmaiden of the physician." D. "Pets can help heal clients."

A. "Everything in nature is alive with invisible forces."

The director of nurses (DON) in a long-term care facility has noticed an increased number of urinary tract infections (UTIs) on the east wing of the facility, and would like the infection control nurse to investigate this problem. What is the best problem statement for this study? A. "Is there a relationship between the personnel caring for specific clients and whether or not these clients developed UTIs?" B. "Is there an increase in the number of UTIs on the east wing of the facility?" C. "Does the east wing have a greater number of UTIs than the west wing of the facility?" D. "What is the client census on the east wing as opposed to the other wings of the facility?"

A. "Is there a relationship between the personnel caring for specific clients and whether or not these clients developed UTIs?"

Research is included as an essential component of nursing by which of the following organizations? (Select all that apply.) A. ANA B. AMA C. International Council of Nurses D. IOM E. Nursing specialty organizations

A. ANA C. International Council of Nurses E. Nursing specialty organizations

19. The nurse is caring for Isabel, a 45-year-old ventilator-dependent quadriplegic. The nurse is in the process of placing IV access when the ventilator alarms occlusion. The nurse assesses Isabel, and she appears mildly uncomfortable but is not in acute distress. What is the nurse's priority in the nursing outcome planning?

Assess tracheostomy for patency.

The nurse has palpated the client's radial artery and identified a heart rate of 88 beats per minute with an irregular rate. What is the nurse's most appropriate action?

Auscultate the client's apical heart rate.

The nurse is conducting an interview with a newly admitted client. Which listening behavior should the nurse implement to have a successful interview? a.Avoid the impulse to interrupt. b.Fill in the words for the client. c.Focus mainly on verbal comments. d.Fill in quiet spaces and pauses.

Avoid the impulse to interrupt.

A nurse is preparing to conduct a research study using the qualitative method. When conducting the research, which characteristics would be included? Select all that apply. A. The research would use deductive reasoning. B. The data collected would primarily be subjective. C. The approach would be holistic. D. Initial concepts would be fairly well developed. E. The researcher would follow formal protocols for data collection.

B. The data collected would primarily be subjective. C. The approach would be holistic.

Teaching a woman about breast self-examination is an example of what broad aim of nursing? A. treating illness B. preventing illness C. restoring health D. facilitating coping

B. preventing illness

A client presents to the emergency department with profuse bleeding from a crushing injury while at work. Which set of vital signs does the nurse anticipate finding in such this client?

Blood pressure 80/50 mm Hg, heart rate 120 beats/min, respiratory rate 24 breaths/min

The nurse is caring for a client who underwent surgery 1 day ago. Which client problem can be addressed by independent nursing diagnoses? The client has a temperature of 101°F (38.3°C). The client has diminished breath sounds. The client has a blood pressure of 160/95 mm Hg. The client is requesting medication for pain.

The client has diminished breath sounds.

22. The nurse recognizes that an example of a cognitive outcome is:

The client identifies three foods high in potassium by August 8.

An informatics nurse specialist is conducting an in-service education program for a group of staff nurses. The topic is ensuring electronic client data is secure and private. The specialist determines that the teaching was successful when the group identifies which aspect as essential to ensuring the security of electronic data when using clinical systems? Use of strong passwords Thorough knowledge of interoperability Intuitive system design Testing

Use of strong passwords

A nurse plans to measure the temperature of a client with mild diarrhea, but the client has just had hot soup. Which action should the nurse perform to obtain the accurate temperature of the client?

Wait for 15 to 20 minutes before measuring the oral temperature

Can a nurse develop a nursing diagnosis when there is not enough evidence to support the presence of a problem, but the nurse would like to gather more evidence? Yes, this defines a risk diagnosis. No, a nursing diagnosis describes an existing problem. No, the nurse must have all of the evidence before formulating the diagnosis. Yes, this defines a possible nursing diagnosis.

Yes, this defines a possible nursing diagnosis.

A client recovering after an appendectomy is reporting pain. The nurse administers the ordered pain medication and assists the client to splint the incision. What is the nurse's next step in implementing the plan of care? a) Reassess the client to determine the effectiveness of the interventions. b) Perform additional nonpharmacological pain interventions. c) Instruct the client that pain medication is available at regular intervals. d) Notify the physician that the client has required pain medications

a) Reassess the client to determine the effectiveness of the interventions.

The nurse cares for a client who is a member of a different culture from the nurse's. Which question is most important for the nurse to ask to assess the client's beliefs about treatment? a. "What are your expectations about being in the hospital?" b. "What do you eat for breakfast?" c. "How do you feel about being in the hospital?" d. "What do you believe about health care?"

a. "What are your expectations about being in the hospital?"

A nurse is engaged in the most basic level of reflection. Which question would the nurse most likely ask? a. "What happened?" b. "How did the client value the experience?" c. "Were assumptions made correctly?" d. "How did the client perceive the event?"

a. "What happened?"

A nurse is documenting evaluation of the care provided for an infant born with Down syndrome. Which nursing actions exemplify the appropriate documentation process? Select all that apply. a. After the data have been collected to determine client outcome achievement, the nurse writes an evaluative statement to summarize the findings. b. The nurse writes a 2-part evaluative statement that includes a decision about how well the outcome was met, along with client data that support the decision. c. The nurse has three decision options for how goals have been met. d. The nurse determines whether a client goal has been met or not met. In each case, the goal is discontinued. e. The nurse does not increase the complexity of a goal after it has been achieved to prevent client anxiety and distrust. f. If a nurse writes a properly written goal, it is not affected by client, nurse, or health care variables.

a. After the data have been collected to determine client outcome achievement, the nurse writes an evaluative statement to summarize the findings. b. The nurse writes a 2-part evaluative statement that includes a decision about how well the outcome was met, along with client data that support the decision. c. The nurse has three decision options for how goals have been met.

Which statement by a new nurse regarding validation of data collected during client assessment indicates a need for further training? a.All data collected need to be validated. b. Validation is the act of confirming or verifying. c. Validation helps to keep data as free from error as possible. d. Validation is an important part of assessment.

a. All data collected need to be validated.

The nurse should evaluate client outcomes at which time? a. As early as possible b. The day of discharge c. Within 24 hours after identifying them d. Several days after discharge

a. As early as possible

The parent of a 33-year-old client who is admitted to the hospital for drug and alcohol withdrawal asks about the client's condition and treatment plan. Which action by the nurse is most appropriate? a. Ask the client if information can be given to the parent. b. Explain the reasons for the hospitalization, but give no further information. c. Provide the information to the parent. d. Take the parent to the client's room and have the client give the requested information.

a. Ask the client if information can be given to the parent.

A nurse is evaluating the plan of care for a client and determines that the achievement of goals is difficult to evaluate. What should the nurse do when evaluating the plan to ensure that the outcomes are achievable? Select all that apply. a. Be sure that the criteria for appropriate response are clearly specified. b. Be certain that the subject is the client or some part of the client. c. Make sure the client's expected behavior is written in observable, measurable terms. d. Rewrite the plan of care so that the client meets the expected outcomes. e. Specify time limits in the plan.

a. Be sure that the criteria for appropriate response are clearly specified. b. Be certain that the subject is the client or some part of the client. c. Make sure the client's expected behavior is written in observable, measurable terms. e. Specify time limits in the plan.

Which activity is the clearest example of the evaluation step in the nursing process? a. Checking the client's blood pressure 30 minutes after administering captopril b. Taking a client's blood pressure on both arms at the beginning of a shift c. Recognizing that the client's blood pressure of 172/101 is an abnormal finding d. Giving the client an as-needed dose of captopril in light of an abnormal blood pressure reading

a. Checking the client's blood pressure 30 minutes after administering captopril

A nurse who recently graduated is performing an assessment on a client who was admitted for nausea and vomiting. During the assessment, the client reports mild chest pain. The nurse does not know whether the chest pain is related to the gastrintestinal symptoms or should be reported to the physician. Which action should the nurse perform next? a.Consult with another nurse. b.Chart the information. c.Call the family. d.Wait and see whether the pain subsides.

a. Consult with another nurse.

A facility has participated in the Hospital Consumer Assessment of Healthcare Providers and Systems (HCAHPS) survey and received the results. A nurse is part of the team reviewing the information. For which areas should the nurse expect the survey to provide information? Select all that apply. a. Facility cleanliness b. Courteous treatment c. Pain control d. Costs of services e. Emergency department waiting times

a. Facility cleanliness b. Courteous treatment c. Pain control

A nurse is part of an orientation team for a group of newly hired nurses. The nurse is to prepare a presentation for the group about different cultural groups common to the facility. As part of the presentation, the nurse is planning to describe how culture is communicated to provide a foundation for culturally competent care. Which methods of communication would the nurse include? Select all that apply. a. Language b. Behavior c. Symbols d. Implicit beliefs e. Lifeways

a. Language b. Behavior c. Symbols

A nurse working in a rural setting is documenting care using a paper format. The nurse records the routine care, normal findings, and client problems in a narrative note. The nurse reviews the physician's information in the physician's progress notes. The nurse is using which method of documentation? a. Source-oriented b. PIE charting c. Problem-oriented d. Charting by exception

a. Source-oriented

Which statement regarding FOCUS charting is most accurate? a. The charting focuses on client strengths, problems, or needs. b. Problem, intervention, evaluation (PIE) charting is used with FOCUS charting. c. The charting focuses on the injury or illness only. d. Each note should include each section of the data, action, response (DAR) format of charting.

a. The charting focuses on client strengths, problems, or needs.

Which purpose of the evaluation phase of the nursing process is a priority during client care? a. To examine the client's behavioral response to the care received b. To provide basis for the revision of plan of care c. To limit assessment to only the beginning phase of the nursing process d. To appraise the collaboration of the client and family

a. To examine the client's behavioral response to the care received

When an American client states, "I only want an American doctor," the client is expressing: a. ethnocentrism. b. cultural relativity. d. cultural pervasiveness. e. racism.

a. ethnocentrism.

A client is a poor historian of the client's past medical history. Whom should the nurse consult about the client's past history? a.Family b.Old chart c.Physician d.Social worker

a. family

Identifying the kind and amount of nursing services required is a possible solution for: a. inadequate staffing. b. client who fail to communicate their needs. c. nurses who are bored. d. nurses frustrated with substandard care.

a. inadequate staffing.

A nurse in a long-term care facility is writing a note that addresses the care a resident has received during the day and the resident's response to care. What type of note does this represent? a. narrative note b. SOAP note c. PIE note d. flow sheet

a. narrative note

A client who is bleeding profusely from a stab wound is brought to the emergency department. Which type of assessment is most appropriate for this client? a. Initial b. Emergency c. Focused d. Time-lapse

b. Emergency

Which teaching statement best exemplifies cultural competence in relation to time for the American culture? a. It is a sign of respect to be late for your health care appointments. b. It is important to be on time for your health care appointment. c. It is important to be future-oriented when considering your appointment time. d. It is important to arrive within 20 minutes of your scheduled appointment time.

b. It is important to be on time for your health care appointment.

A client describes pain in the right leg as aching at 8/10 on a pain scale. What type of cue is a client's description of pain in the right leg? a. Explanatory b. Subjective c. Objective d. Severe

b. Subjective

Upon assessment of an older adult, the nurse notes the client's skin to have a yellow color. The nurse interprets this finding as a result of which health condition? a) Diverticulitis b) Appendicitis c) Hepatitis d) Cellulitis

c) Hepatitis

The nurse is caring for a client 4 days after total hip arthroplasty and notes the client has lost weight. The unlicensed assistive personnel reports the client's food intake has decreased. Which question will the nurse ask the client to determine if cultural causes are responsible for the weight loss? a. "Is there something wrong with the food?" b. "Would you like to speak with a nutritionist?" c. "What type of food do you like to eat at home?" d. "Can you ask your family to bring you something you like?"

c. "What type of food do you like to eat at home?"

Which is the primary purpose of client records? a. Legal protection b. Performance improvement c. Communication d. Reimbursement

c. Communication

A client admitted with dehydration reports feeling dizzy with ambulation. What teaching would the nurse provide to the client?

"Dizziness when you change position can occur when fluid volume in the body is decreased."

An informatics nurse specialist teaches a group of staff nurses how to document client care in the new electronic health record. To document assessment findings, the nurses are to select the correct information from a drop-down list. During the teaching session, one of the nurses asks, "Why do we have to use this list. Why can't we just type in what we want to say?" Which response by the informatics nurse specialist would reflect the most important reason for documenting in this way? "Documenting this way allows nurses' actions to be measured." "This type of documentation is what the system will support." "It avoids issues related to typing that most nurses have." "This type of free-text documentation is more effective for communication."

"Documenting this way allows nurses' actions to be measured."

An informatics nurse specialist is conducting an in-service program for a group of staff nurses about this specialty. One of the nurses asks, "What exactly is nursing informatics?" Which response by the informatics nurse specialist would be most appropriate? "It involves working primarily with computers and programming codes." "It refers to the use of the electronic health record." "It combines nursing science with information management and analytical sciences." "It is a specialty that deals with online client educational programs.""

"It combines nursing science with information management and analytical sciences."

A client comes to the primary care provider's office for a routine follow-up visit. While completing the assessment, the nurse tells that client about a new patient portal that the office has begun to use. The client asks the nurse, "I guess this is a good thing, but what is it and what does it do?" Which response by the nurse would be appropriate? Select all that apply. "You can use it to set up contact with your health insurance company." "It's a site you can access through your home computer or smartphone." "It allows you to get information about your health status and care." "You can communicate with your provider through the system." "It allows you to request refills for your prescriptions."

"It's a site you can access through your home computer or smartphone." "It allows you to get information about your health status and care." "You can communicate with your provider through the system." "It allows you to request refills for your prescriptions."

A nurse asks an informatics nurse, "How can I make sure that the passwords I create are strong enough to prevent a security problem?" Which action would be appropriate for the informatics nurse to suggest to the nurse? Select all that apply. "Make sure that your password is at least eight characters long." "Sharing your password with a coworker is okay but not with anyone else." "Wait at least 2 months before you reuse a password." "Try using a mnemonic that works only for you." "Avoid using multifactor authentication whenever possible."

"Make sure that your password is at least eight characters long." "Try using a mnemonic that works only for you."

The nurse is teaching a newly diagnosed hypertensive client how to take his or her own BP at home. The client asks why it is so important to do this. What is the nurse's best response?

"Monitoring your BP at home will assist in controlling your BP, thereby decreasing your risk for heart attack and stroke."

An informatics nurse specialist is conducting a discussion with a group of staff nurses who have been participating in the implementation of an updated electronic protocol system. The informatics nurse specialist is gathering information about the system's optimization. One of the participants asks, "Why is this information important?" Which response by the informatics nurse specialist would be appropriate? "This information will help us meet accreditation standards." "We need to show evidence to the executives that the money has been well spent." "We can make improvements to the system to improve our delivery of care." "This is part of the standard protocol required by federal legislation."

"We can make improvements to the system to improve our delivery of care."

An informatics nurse is discussing the implementation of a new documentation system with a group of staff nurses who are using the system. Which response by the group would indicate to the nurse that the system's usability is effective? Select all that apply. "We've noticed that this system really helps to save us valuable time." "Using the system is highly intuitive." "This system makes it challenging to complete tasks." "This system fits nicely into how we work." "The screen has a lot of confusing information on it."

"We've noticed that this system really helps to save us valuable time." "Using the system is highly intuitive." "This system fits nicely into how we work."

The nursing student is discussing the benefits of electronic charting with a precepting nurse who is frustrated with computerized documentation. Which statement by the student requires intervention from the nursing instructor? "You don't have to worry about trying to read poor handwriting." "The computer reminds the nurse to enter information and inhibits omissions." "You save time because you don't have to look for the physical chart." "You can make extra money with overtime pay with end-of-shift charting."

"You can make extra money with overtime pay with end-of-shift charting."

Which are examples of objective data? Select all that apply.

*Laboratory test results *Breath sounds on auscultation *A client's temperature

A nurse is beginning the preparatory phase of the nursing interview for a client who fractured the left leg in a fall. Which nursing actions occur in this phase of the nursing interview? Select all that apply.

*The nurse ensures that the interview environment is private and comfortable. *The nurse arranges the seating in the interview room to facilitate an easy exchange of information. *The nurse prepares to meet the client by reading current and past records and reports.

The nurse is talking to an older adult client who performs home blood pressure monitoring (HBPM) and finds that recently her BP measurements have consistently been low. Which factors may contribute to causing this client's BP to be falsely decreased? Select all that apply.

-Applying too wide a cuff -Using cracked or kinked tubing -Releasing the valve rapidly

35. The nurse has identified short- and long-term goals for a client after surgery to remove a leg tumor. When determining interventions for the goals, which questions are important for the nurse to consider? Select all that apply.

-Are the interventions realistic and do they require resources available to the nurse? -Are the interventions compatible with the client's values, beliefs, and cultural and psychosocial background? -Are the interventions compatible with other planned therapies? Are the interventions evidence-based?

30. Which elements are common to any type of plan of care? Select all that apply.

-Nursing diagnoses -Client goals -Nursing interventions

What instructions should be provided to a newly diagnosed hypertensive client about home blood pressure monitoring? Select all that apply

-Rest 3 to 5 minutes before taking your BP. -Take three measurements and average together. -Use a validated monitor with an automatic inflation cuff.

18. Which are characteristics of appropriate client outcome statements? Select all that apply.

-Specific -Realistic -Measurable

When creating the teaching plan for a client who will be monitoring his or her pulse at home, which factors should the nurse teach the client that may influence the pulse rate by causing an increase in pulse? Select all that apply

-Stress -Exercise -Fever

6. What behaviors reflect planning? Select all that apply.

-The nurse considers the developmental level of the client when selecting education materials. -The nurse seeks input from the client and family regarding acceptable, nonpharmacologic pain management strategies. -The nurse decides to assist the client with ambulation in the hallway twice per shift.

A nurse is assessing clients in the emergency department for body temperature. Which nursing actions reflect proper technique when assessing body temperature by various methods? Select all that apply.

-When assessing an oral temperature with an electronic thermometer, place the probe beneath the client's tongue in the posterior sublingual pocket. -When assessing rectal temperature with an electronic thermometer, lubricate about 1 in (2.5 cm) of the probe with a water-soluble lubricant. - When assessing temperature with an electronic thermometer, hold the thermometer in place in the assessment site until a beep is heard.

A client has an axillary temperature of 102.6 F (39.2°C). Which clinical manifestations would the nurse anticipate? Select all that apply.

-respiratory rate 30/min -headache -red or flushed skin

During measurement of a rectal temperature, the thermometer probe should be inserted about how many inches (centimeters) into the anus in an infant?

1 in (2.5 cm)

The nurse must assess a client's systolic blood pressure using a Doppler ultrasound. Place the following steps to this procedure in the correct order. Use all options.

1)Center the bladder of the cuff over the artery, lining the artery marker on the cuff up with the artery. 2)Wrap the cuff around the limb smoothly and snugly, and fasten it. 3)Place a small amount of conducting gel over the artery. 4)Place the Doppler tip in the gel and move it around until hearing the pulse. 5) Inflate the cuff while continuing to use the Doppler device on the artery. 6)Note the point on the gauge where the pulse disappears.

Based upon circadian rhythms, when would the nurse note the highest temperature during a 24-hour period?

1700

A nurse is assessing the respiratory rate of a sleeping infant. What would the nurse document as a normal finding?

30 to 60 breaths/min

The nurse is performing a telephone follow-up with parents whom she taught to monitor their newborn's BP and pulse at home. Which results reported by the parents would indicate that the parents are performing the technique correctly and there is no cause for concern?

80/50 mm Hg and 145 bpm

The nurse formulates the following nursing diagnosis: Disturbed Body Image related to decreased ability to cope with surgical removal of right breast as evidenced by the client refusing to look at the surgical site and stating, "I'm ugly. My husband will no longer find me desirable." What is the etiology identified in this nursing diagnosis? A - Decreased ability to cope with surgical removal of right breast B - "I'm ugly. My husband will no longer find me desirable." C - Disturbed body image D - Refusal of the client to look at the surgical site

A - Decreased ability to cope with surgical removal of right breast

Which information ensures accuracy when the nurse is developing a nursing diagnosis? A set of lab values Abnormal diagnostic test results A cluster of clinical cues Specific nursing interventions

A cluster of cues

Which scenario is an example of a time-lapse reassessment? Seeing a client down on the floor, the nurse assesses the client's airway, breathing, and circulation, calls for help, and begins a quick neurological exam. a.A nurse in a long-term skilled nursing facility assesses a new resident's baseline health status. b.A nurse just coming on shift performs a focused physical assessment on each client, based on the client's diagnosis. c.A nurse assesses a client with mobility issues to see how the client is doing with fall prevention strategies they practiced before.

A nurse assesses a client with mobility issues to see how the client is doing with fall prevention strategies they practiced before.

The nurse caring for a client with obesity would like to address the possible health problems that can develop related to obesity. To plan care for this client, what type of nursing diagnosis would the nurse formulate? A problem-focused nursing diagnosis A risk nursing diagnosis A possible nursing diagnosis A health promotion nursing diagnosis

A risk nursing diagnosis

The nurse is attempting to provide anticipatory guidance for the parents of an 18-month-old child. Which statement would be best for the nurse to make? A. "Keep all medications in a locked cabinet." B. "Does the child have nightmares often?" C. "The child is in the 95th percentile for weight." D. "Do not give the child a bottle now."

A. "Keep all medications in a locked cabinet."

Which advancement in the nursing profession occurred immediately after World War II? A. Advanced nursing education was established. B. National standards for nursing were developed. C. Care of clients in the hospital setting declined. D. Government funding supported minority nurses.

A. Advanced nursing education was established.

The first nursing journal owned, operated, and published by nurses was: A. American Journal of Nursing B. Nurse Educator C. Nursing Research D. Standards of Clinical Nursing Practice

A. American Journal of Nursing

Which organization has established standards that help the nurse determine which clinical actions fall under the scope of nursing practice? A. American Nurses Association B. National League for Nursing C. International Council of Nurses D. National Council of State Boards of Nursing

A. American Nurses Association

Which scenario is the best example of the nurse in the role of teacher/educator? A. Assessing whether the client is able to perform a dressing change B. Teaching a first-grader to read C. Communicating discharge status to a home care agency D. Conducting research on dressing changes

A. Assessing whether the client is able to perform a dressing change

The nurse is performing care for a client in the end stage of cancer. How can the nurse best facilitate the client and family's ability to cope? Select all that apply. A. Assist the client with activities of daily living (ADLs). B. Encourage the family to leave and let the nurse take over care. C. Inform the family that there is nothing they can do for their loved one. D. Assist the client and family with the preparation for end-of-life. E. Refer the client and family to hospice services.

A. Assist the client with activities of daily living (ADLs). D. Assist the client and family with the preparation for end-of-life. E. Refer the client and family to hospice services.

For which research activity is a phenomenological research method most appropriate? A. Attempting to understand non-English speaking immigrants' experiences of being hospital clients B. Describing the health maintenance activities that are practiced by homeless, intravenous drug users C. Explaining how the particular characteristics of the Vietnam War affected the roles of nurses D. Understanding how clients cope with a new diagnosis of Alzheimer's disease

A. Attempting to understand non-English speaking immigrants' experiences of being hospital clients

A nurse is planning to pursue further education in the hopes of becoming an expert in geriatric nursing who carries out direct care. For which expanded career role is the nurse preparing? A. Clinical nurse specialist B. Nurse manager C. Nurse-midwife D. Physician assistant

A. Clinical nurse specialist

During the course of any given day of work in the acute care setting, the nurse may need to perform which roles? Select all that apply. A. Communicator B. Counselor C. Teacher D. Financier E. Statistician

A. Communicator B. Counselor C. Teacher

A nursing theory differs from a theoretical framework in which way? A. Concepts and propositions are more specific B. Concepts are in a meaningful configuration C. Cause and effect of nursing actions are described D. Ideas about aggregates are described

A. Concepts and propositions are more specific

A nurse is reviewing a qualitative research study. Which aspects of this type of study would the nurse need to keep in mind? Select all that apply. A. Control or manipulation is rarely used. B. Reality is not viewed as a fixed entity. C. Objectivity is valued. D. Biases are controlled to avoid contamination. E. Intuition is used for analysis.

A. Control or manipulation is rarely used. B. Reality is not viewed as a fixed entity. E. Intuition is used for analysis.

A public health nurse is working with a large group of clients who require a variety of patient care, health teaching, and health promotion. Using Virginia Henderson's Basic Needs Theory, how should the nurse proceed in order to provide a variety of medical, health teaching, and health promotion interventions? A. Determine general basic needs of the group and additional needs specific to each client. B. Identify unhealthy environmental factors that require modification to promote health. C. Assist individuals with health promotion in order to maximize their self-care potential. D. Plan changes to the external environment to promote a healthy lifestyle.

A. Determine general basic needs of the group and additional needs specific to each client.

The new nursing graduate is concerned about some of the critical changes that will be occuring in nursing. What changes does the nurse anticipate will impact nursing care? A. Difficulty for nurses to remain current in a rapidly changing medical and technology environment B. Rapid growth of nurses and shortage of job opportunities C. Decrease in health care costs D. Limitations that nurses have in the workforce

A. Difficulty for nurses to remain current in a rapidly changing medical and technology environment

There are various levels of preparation for nursing practice. Which degree program will best prepare a nurse to work as a nurse researcher? A. Doctoral B. Diploma C. Baccalaureate D. Master's

A. Doctoral

The Nurse Corps of the United States Army was established by whom? A. Dorothea Dix B. Lillian Wald C. Florence Nightingale D. Isabel Hampton Robb

A. Dorothea Dix

When looking at a model for evidence-based practice, what is the final step of the process? A. Evaluating practice change B. Searching the literature C. Appraising evidence D. Formulating a clinical question

A. Evaluating practice change

A nurse researcher is examining the cause-and-effect relationship between the consumption of tap water containing minimal amounts of bleach, and the incidence of cancer in rats. The research is taking place in a laboratory setting. What type of quantitative research is being used based upon this description? A. Experimental research B. Descriptive research C. Correlational research D. Quasi-experimental research

A. Experimental research

The registered nurse is performing a nutritional assessment to ensure that the client's diet is optimal for wound healing. The nurse's intervention can be traced back to which key contributor to nursing? A. Florence Nightingale B. Clara Barton C. Dorothea Dix D. Linda Richards

A. Florence Nightingale

Who is considered to be the first nursing theorist who conceptualized nursing in terms of manipulating the environment? A. Florence Nightingale B. Sister Callista Roy C. Dorothea Orem D. Lydia Hall

A. Florence Nightingale

Which are valid purposes for concepts in nursing practice? (Select all that apply.) A. Guide the steps of the nursing process B. Suggest appropriate nursing interventions C. Help identify information relevant to a specific client issue D. Provide a constant for all nursing care E. Suggest appropriate nursing outcomes for a client

A. Guide the steps of the nursing process B. Suggest appropriate nursing interventions C. Help identify information relevant to a specific client issue E. Suggest appropriate nursing outcomes for a client

The statement, "More frequent handwashing will significantly lower the rate of infection in hospitalized clients," is an example of what research component? A. Hypothesis B. Proposal C. Research question D. Implication

A. Hypothesis

A nurse researcher is involved in an experimental research study. Which component would the nurse researcher manipulate? A. Independent variable B. Dependent variable C. Research design D. Data analysis

A. Independent variable

The founder of public health nursing is: A. Lillian Wald B. Clara Barton C. Linda Richards D. Dorothea Dix

A. Lillian Wald

Attracting minorities to the profession of nursing is an important consideration for the future of nursing. Which key historical nursing figure set a precedent in this area? A. Mary Eliza Mahoney B. Nora Gertrude Livingston C. Mary Agnes Snively D. Mary Ann Bickerdyke

A. Mary Eliza Mahoney

A nurse is reviewing nursing theory for usefulness in a home healthcare setting. Which is the appropriate application of Dorothea Orem's Self-Care Model in a home healthcare setting? A. Match the client's self-care needs with appropriate supportive interventions. B. Counsel client on methods to adapt to current state to reduce stress. C. Assist the client to conserve and maintain principles of personal and social integrity. D. Monitor therapeutic interventions in the client's home.

A. Match the client's self-care needs with appropriate supportive interventions.

The client's plan of care is created by the nurse using which guideline for nursing practice? A. Nursing process B. Nursing's Social Policy Statement C. Nurse practice act D. ANA Standards of Nursing Practice

A. Nursing process

Which activity systematically inquires about the problems encountered in nursing practice and into the modalities of client care? A. Nursing research B. Managed care C. Outcome criteria D. Scientific inquiry

A. Nursing research

The student nurse is changing the bed linens of a hospitalized patient. While placing the contaminated materials into the laundry bag, the outside of the bag is touched by the material. What should the student nurse do next? A. Obtain another laundry bag and place the contaminated bag inside while in the patient's room. B. Notify his instructor of the error. C. Carefully remove the linens from the bag and place them into a clean bag. D. Take the bag to the disposal room and label the bag as contaminated.

A. Obtain another laundry bag and place the contaminated bag inside while in the patient's room.

The focus of nursing is always on which of the four common concepts in nursing theory? A. Person B. Environment C. Health D. Nursing

A. Person

A nurse researcher is studying female clients who have survived breast cancer. The nurse asks each client to describe her experience and then analyzes the data for the meaning of the experience within each person's own reality. This nurse has used what type of qualitative research method? A. Phenomenology B. Grounded theory C. Ethnography D. Historical

A. Phenomenology

A group of nurses is planning to investigate the effectiveness of turning immobilized stroke clients more frequently in order to prevent skin breakdown. The team has begun by formulating a PICO question. Which element will the "O" in the team's PICO question refer to? A. Preventing skin breakdown B. Clients who have experienced a stroke C. Turning clients more frequently D. The currently used turning schedule

A. Preventing skin breakdown

Which are goals of nursing theory? (Select all that apply.) A. Provide knowledge and a rationale for client interventions B. Provide a rationale for appropriate nursing actions in a given situation C. Identify and define concepts that are important to nursing D. Increase the nursing body of knowledge E. Provide a definition for nursing

A. Provide knowledge and a rationale for client interventions B. Provide a rationale for appropriate nursing actions in a given situation C. Identify and define concepts that are important to nursing D. Increase the nursing body of knowledge

Which scenario is the best example of a nurse implementing care according to the ANA standards of practice? A. Providing nutrition teaching to a client with diabetes B. Analyzing a client's blood glucose levels C. Taking a client's apical and radial pulse rates D. Determining that a client is at risk for falls

A. Providing nutrition teaching to a client with diabetes

A nurse researcher must decide on the method for conducting the research. The researcher that plans to emphasize collection of numerical data and analysis would select which method of research? A. Quantitative research B. Qualitative research C. Basic research D. Applied research

A. Quantitative research

Advocating for financial support for university-based schools of nursing was noted in which government document? A. The Goldmark Report B. Hill-Burton Amendment C. Health Care Initiative Reform Act D. Lysaught Report

A. The Goldmark Report

The registered nurse communicates with the physical therapist that a client is now on strict bed rest due to bradycardia. Which statement best explains the standard exemplified by the nurse? A. The RN coordinates care delivery. B. The RN identifies outcomes. C. The RN analyzes client data. D. The RN collects client data.

A. The RN coordinates care delivery.

The nurse conducts a home safety assessment for a client. Which statement best explains the standard of care being implemented? A. The RN promotes a safe environment. B. The RN identifies client outcomes. C. The RN educates about preventing falls. D. The RN develops a plan of care.

A. The RN promotes a safe environment.

The American Nurses Association (ANA) Standards of Professional Nursing Practice provides standard of care for all nurses. Which statement on the assessment of the nursing process is accurate? A. The nurse collects comprehensive data pertinent to the client's health or situation. B. The nurse dictates the plan that prescribes strategies of care. C. The nurse monitors the ethical conduct of authorities and clients. D. The nurse evaluates progress toward implemented actions.

A. The nurse collects comprehensive data pertinent to the client's health or situation.

A group of concepts and the relationships among them is what? A. Theory B. Evidence-based practice C. Deductive reasoning D. Inductive reasoning

A. Theory

A nurse working in a long-established hospital learned a specific approach to administering intravenous injections from the previous generation of nurses at the hospital. This is an example of which type of knowledge? A. Traditional knowledge B. Authoritative knowledge C. Scientific knowledge D. Philosophy knowledge

A. Traditional knowledge

A nurse is reading a journal article about evidence-based practice. The nurse understands that this practice is an outcome of which type of research? A. Translational B. Quantitative C. Qualitative D. Experimental

A. Translational

Which aspect of the nursing research process addresses a client's understanding of the potential risks and benefits of the study? A. Undergoing the informed consent process with the client B. Obtaining the client's signature on a permission document C. Meeting with the hospital's institutional review board (IRB) D. Giving the client the opportunity to ask questions about the study

A. Undergoing the informed consent process with the client

The nursing process is: A. a critical thinking method used by nurses to provide nursing care that is individualized and holistic. B. a mechanism for increasing the knowledge and skill of the nurse through programs of education. C. the promotion of health, prevention of illness, and care of ill, disabled, and dying individuals. D. an approach for identifying and analyzing the best available scientific evidence for nursing care.

A. a critical thinking method used by nurses to provide nursing care that is individualized and holistic.

A registered nurse (RN) is caring for four clients on a medical-surgical unit. Which task is most appropriate for the nurse to delegate to the licensed practical nurse (LPN)? A. administering bedside blood glucose testing B. administering blood products C. administering intravenous push medication D. administering chemotherapy

A. administering bedside blood glucose testing

A nurse is preparing to conduct a research study and uses the PICO format to develop the foreground question which is: "In adults, does reducing salt intake, compared to no change in salt intake, lower blood pressure?" The nurse identifies the "P" as: A. adults. B. reducing salt intake. C. no change in salt intake. D. lower blood pressure.

A. adults.

During World War II, nurses were actively recruited and enlisted in the military. What effect did this have on the nursing profession? A. caused a civilian nursing shortage B. encouraged more men to become nurses C. increased demand for ambulatory services D. inspired the formation of home health nursing

A. caused a civilian nursing shortage

A nurse is discussing the history of the profession with a client and describes that a shift in societal focus from religion to warfare had a negative impact on nursing due to: A. female criminals recruited as nurses. B. nurses caring for war victims and soldiers. C. nurses no longer praying for their clients. D. unmarried females recruited as nurses.

A. female criminals recruited as nurses.

What was one barrier to the development of the nursing profession in the United States after the Civil War? A. lack of educational standards B. hospital-based schools of nursing C. lack of influence from nursing leaders D. independence of nursing orders

A. lack of educational standards

The nurse is utilizing knowledge about a blood pressure medication's actions and side effects to determine whether or not to give a client, whose blood pressure is low, the prescribed blood pressure medication. What best describes the aspect of nursing demonstrated? A. science of nursing B. art of nursing C. conduction of research D. quality improvement

A. science of nursing

Which is the best example of a nurse demonstrating the role of caregiver? A. starting an intravenous line in the client's arm B. referring the client to a mental health clinic C. assigning a room for a new hospital client D. reporting lab values to a client's physician

A. starting an intravenous line in the client's arm

Evidence-based care emphasizes decision making based on the best available evidence and: A. use of outcome studies to guide decisions. B. evaluation by experts to direct care in specialty areas. C. care based on pathophysiologic factors. D. cost efficiency of treatment models.

A. use of outcome studies to guide decisions.

A nurse is treating a client with congestive heart failure. The client reports having difficulty walking up the stairs at home and barely being able to walk to the store. Which is an accurate actual nursing diagnosis for this client? Activity Intolerance related to congestive heart failure as evidenced by inability to walk up and down stairs Activity Intolerance as evidenced by inability to walk up and down stairs and inability to walk to the store Noncompliance with new diagnosis of congestive heart failure Risk for Impaired Coping related to congestive heart failure

Activity Intolerance related to congestive heart failure as evidenced by inability to walk up and down stairs

A nurse is using Gordon's functional health patterns as an organizing framework for client assessment. The client has significant problems related to breathing, for which the nurse identifies several nursing diagnostic labels, including Ineffective Breathing Pattern and Impaired Gas Exchange. The nurse understands that these nursing diagnoses would be organized under which functional pattern? Activity-exercise Nutritional-metabolic Coping-stress tolerance Cognitive-perceptual

Activity-exercise

After assessing a client, a nurse identifies the nursing diagnosis, "Ineffective Airway Clearance related to thick tracheobronchial secretions." The nurse would classify this nursing diagnosis as which type? Actual Risk Possible Health promotion

Actual "Ineffective Airway Clearance related to thick tracheobronchial secretions" is an actual nursing diagnosis, because it describes a human response to a health problem that is being manifested. A health promotion nursing diagnosis is a diagnostic statement that describes the human response to levels of wellness in an individual, family, or community that has a potential for enhancement to a higher state. A risk nursing diagnosis describes human responses to health conditions or life processes that may develop in a vulnerable individual, family, or community. A possible nursing diagnosis is made when not enough evidence supports the presence of the problem, but the nurse concludes that it is highly probable and wants to collect more information.

"Acute Pain related to instillation of peritoneal dialysate as evidenced by client wincing and grimacing during procedure, client description of experience as 'stabbing'" is an example of which type of nursing diagnosis? Actual nursing diagnosis Risk nursing diagnosis Health promotion nursing diagnosis Potential nursing diagnosis

Actual nursing diagnosis This is an actual nursing diagnosis as it contains the diagnostic label (acute pain), related factors (instillation of peritoneal dialysate), and defining characteristics (wincing, grimacing during procedure, stabbing sensation). A risk nursing diagnosis is a two-part statement that includes a diagnostic label and risk factors. A health promotion nursing diagnosis is one-part statement that includes a diagnostic label. A potential nursing diagnosis is a two-part statement that includes a diagnostic label and unknown related factors.

24. A nurse is caring for a client who began taking the antidepressant paroxetine 2 weeks ago. The client recently began giving away prized possessions and tells the nurse, "My mind is made up, I can't do this any longer." What is the best action by the nurse to incorporate this information into the plan of care?

Add the nursing diagnosis: Risk for Self-Harm.

While caring for a client admitted to the hospital for a fractured tibia, the nurse notes the client's blood pressure readings are consistently higher the expected range for the client's age. How would the nurse most appropriately plan to care for this client? Address the collaborative problem PC: Hypertension. Address the nursing diagnosis, "Risk for Injury related to hypertension." Address the possible nursing diagnosis, "Ineffective Tissue Perfusion related to hypertension." Address the medical diagnosis of Hypertensive disorder.

Address the collaborative problem PC: Hypertension.

The nurse is preparing to interview a client who demonstrates significant abdominal pain and rates the pain at 10 on a 0 to 10 pain scale. What action by the nurse can improve the outcome of the interview?

Administer prescribed pain medication prior to conducting the interview.

During examination a client becomes very tired but still needs to answer questions so that the nurse has sufficent data for planning care. Which action by the nurse would be most appropriate in this situation?

Ask the client whether it is okay to interview the client's spouse for the answers to the interview questions.

What would be a nursing priority when assessing a client who weighs 250 lb (112.50 kg) and stands 5 ft, 3 in (1.58 m) tall? a.Begin client education regarding a low-fat diet. b.Assess blood pressure with a large cuff. c.Assess cholesterol levels. d.Obtain an electrocardiogram daily.

Assess blood pressure with a large cuff.

The nurse is attempting to assess a client's radial pulse. The pulse is weak, irregular and unable to be counted. What action would the nurse take next?

Assess the apical pulse.

The nurse takes a client's vital signs and notes a blood pressure of 88/56 mm Hg with a pulse rate of 60 beats/min. Which action should the nurse take first?

Assess the client for dizziness.

A client is receiving home care due to an unstable blood pressure. Which nursing intervention is a priority? a.Assess the client's medication regimen. b.Assess the client's blood pressure. c.Assess the client's activity level. d.Assess the client's diet.

Assess the client's blood pressure.

When planning initial care for a 16-year-old client and the client's newborn, the nurse formulates a nursing diagnosis of "Risk for Impaired Attachment." What would be the nurse's most appropriate action to take next? Assess the client's interactions with the newborn. Direct all education of infant care to the client's mother. Initiate referrals to available community services. Develop a comprehensive education plan for infant care.

Assess the client's interactions with the newborn. To address a risk nursing diagnosis, the nurse is required to collect additional data. Observing the client's interactions with the newborn would be the most effective way to evaluate attachment. It is inappropriate to assume that the client's mother will be doing all the infant care, which would also be detrimental to the client's attachment to the infant. It is premature to initiate referrals to community services until further data are collected. It is also premature to develop a comprehensive education plan until the needs of the client are known.

The care plan for a client who has been frequently admitted to the hospital for exacerbation of chronic obstructive pulmonary disease (COPD) includes a nursing diagnosis of "Noncompliance related to lack of knowledge as evidenced by frequent admissions to the hospital." What is the most appropriate method for the nurse to use to validate the nursing diagnosis? Assess the severity of the client's illness. Assess the client's knowledge of COPD. Assess the client's financial resources. Assess the client's access to health care.

Assess the client's knowledge of COPD. The nurse has theorized that the client is noncompliant because of a lack of knowledge. Therefore, the nurse must assess what knowledge the client has. The severity of the client's illness has no bearing on how compliant the client is or on the client's level of knowledge about the disease. The client's financial resources and access to health care may be other causes of noncompliance but do not support the nurse's theory of the client's lack of knowlege leading to noncompliance.

A facility plans to implement a new electronic medication documentation system. An informatics nurse specialist conducts a focus group with staff nurses to gather information. The nurse specialist asks the group about the current system being used, including a step-by-step account of the actions they perform. The nurse specialist also asks the nurses how they see this new system affecting this process and their overall daily tasks. The informatics nurse specialist is demonstrating which ANA informatics competency? Assessment Outcomes identification Planning Coordination of activities

Assessment

Put the phases of the nursing process in the correct order:

Assessment Diagnosis Planning Implementation Evaluation

A client asks an RN to prescribe a medication for pain. What is the best answer by the nurse? A. "No nurse can prescribe a medication." B. "Only advanced practice registered nurses have prescriptive authority." C. "Take two ibuprofen every 4 to 6 hours." D. "Take one acetaminophen every 4 hours."

B. "Only advanced practice registered nurses have prescriptive authority."

Which could be considered an example of a qualitative research project? A. A BSN student is looking at the prevalence of alcohol hand sanitizer use on an adult medical floor. The student is collecting data by observing nurses going in and out of rooms and noting how often they use hand sanitizer and how often they do not. B. A PhD nursing student is collecting data on how Somali immigrants perceive the care they receive in one community clinic. The nursing student is taping and analyzing interviews with clients that have been in the country and community 2 years or less and who have sought care at least once at this clinic. C. A nurse is a member of the wound care team on an inpatient floor in the hospital. The nurse is collecting data for a research project that is interested in uptake of information given to clients by nurses. The nurse is giving clients a quiz about the information both before client education occurs and afterward. D. A member of a nursing research team is interested in the prevalence of anemia in a group of clients with Crohn disease. This researcher is looking at the serum hemoglobin and ferritin levels of these clients.

B. A PhD nursing student is collecting data on how Somali immigrants perceive the care they receive in one community clinic. The nursing student is taping and analyzing interviews with clients that have been in the country and community 2 years or less and who have sought care at least once at this clinic.

After hearing a presentation about the American Nurses Association (ANA), a nurse decides to join the organization based on the understanding that: A. membership is open to all nurses in the United States. B. ANA aims at fostering high standards of nursing in the United States. C. members include nurses, other health care providers, and lay people. D. ANA invites students showing excellence in scholarship to become members.

B. ANA aims at fostering high standards of nursing in the United States.

A nurse is completing a family assessment on a routine home health visit. The parents have a child with a severe peanut allergy. The mother states that she does not purchase any foods with peanut or peanut oil for her family. The other children are allowed to have foods containing peanuts while they are at school and visiting with friends and family. The nurse would conclude that which of the following theories would most like be the basis for this family's functioning? A. General Systems Theory B. Adaptation Theory C. Developmental Theory D. Maslow's Theory

B. Adaptation Theory

A nurse develops the following foreground question using the PICOT format in preparation for a research study: "In overweight clients, how do chromium supplements compared to no supplements help with weight loss?" Which part of the question reflects the intervention? A. Overweight clients B. Chromium supplements C. No supplements D. Weight loss

B. Chromium supplements

An abstract idea or object that relates to other abstract ideas or objects is which? A. Theory B. Concept C. Deductive reasoning D. Inductive reasoning

B. Concept

The nurse is caring for a neonate at risk for low body temperature. Which processes should the nurse address? Select all that apply. A. Dilation B. Convection C. Radiation D. Conduction E. Evaporation

B. Convection C. Radiation D. Conduction E. Evaporation

Which activity best helps the nurse apply theory to practice? A. Theory development B. Evidence-based research C. Client-focused care D. Case management

B. Evidence-based research

Which individual provided community-based care and founded public health nursing? A. Mary Adelaide Nutting B. Lillian Wald C. Sojourner Truth D. Clara Barton

B. Lillian Wald

What is the central theme of Florence Nightingale's nursing theory? A. Humans are in a constant relationship with stressors in the environment. B. Meeting the personal needs of the client within the environment. C. Nursing is an art. D. Nursing is a therapeutic, interpersonal, and goal-oriented process.

B. Meeting the personal needs of the client within the environment.

A nurse is planning to conduct a nursing research study and is seeking federal funding. Which institution would be most helpful for the nurse to contact regarding acquiring funding? A. National Institutes of Health B. National Institute of Nursing Research C. Institute of Medicine D. ANA Cabinet on Nursing Research

B. National Institute of Nursing Research

Nurses in an ICU noticed that their clients required fewer interventions for pain when the ICU was quiet. They then asked a researcher to design a study about the effects of noise on the pain levels of hospitalized clients. How does this demonstrate the ultimate goal of expanding the nursing body of knowledge? A. Nursing research involves clients in their care while hospitalized. B. Nursing research helps improve ways to promote and maintain health. C. Nursing research draws conclusions about the quality of client care. D. Nursing research explains ongoing medical studies to clients, and asks for participation.

B. Nursing research helps improve ways to promote and maintain health.

Which is most similar to a nursing model? A. Adaptation B. Nursing theory C. Nursing construct D. Proposition

B. Nursing theory

A nurse incorporates the health promotion guidelines established by the U.S. Department of Health document Healthy People 2010. Which disease/condition is a health indicator discussed in this document? A. Cancer B. Obesity C. Diabetes D. Hypertension

B. Obesity

Which are core concepts in nursing theory? (Select all that apply.) A. Society B. Person (client) C. Environment D. Health E. Nursing

B. Person (client) C. Environment D. Health E. Nursing

When a researcher begins to form plans for a research project, the researcher must decide on the method for conducting the research. The nurse researcher that plans to emphasize collection of narrative data and the analyses would select which of the following methods of research? A. Quantitative research B. Qualitative research C. Basic research D. Applied research

B. Qualitative research

The nurse asks a client about his spiritual health. Which statement best explains the standard of care utilized by the nurse? A. The RN provides spiritual counseling. B. The RN collects comprehensive data. C. The RN prays with clients. D. The RN collaborates with spiritual healers.

B. The RN collects comprehensive data.

A nurse is providing care for clients in a long-term care facility. What should be the central focus of this care? A. The nurse's actions B. The client receiving the care C. The nurse as the caregiver D. Nursing as a profession

B. The client receiving the care

The nurse is caring for a client who cannot meet health needs independently. Which action made by the nurse depicts concern and attachment? A. telling the client, "I will be back in 15 minutes to change your dressing." B. asking the client, "How are you today? I am really worried about you." C. talking about diabetes and teaching the client how to do foot care D. organizing the work for the day and evaluating how the day went

B. asking the client, "How are you today? I am really worried about you."

A nurse working in an army hospital in the 1800s faced deplorable conditions while caring for clients. What nursing actions would directly aid in the reduction of high death rates? (Select all that apply.) A. writing letters to the military to change hospital conditions B. bathing clients and washing linens and clothing daily C. advocating for formal nursing education D. providing daily meals and feeding clients E. opening windows to improve ventilation

B. bathing clients and washing linens and clothing daily D. providing daily meals and feeding clients E. opening windows to improve ventilation

A nurse is preparing to conduct a research study and chooses a mixed method of research. The nurse chooses this method based on the understanding that the results will: A. be more holistic. B. provide a more complex level of understanding. C. reveal gaps in data analysis. D. identify the problem more succinctly.

B. provide a more complex level of understanding.

What is the primary purpose of standards of nursing practice? A. to provide a method by which nurses perform skills safely B. to ensure knowledgeable, safe, comprehensive nursing care C. to establish nursing as a profession and a discipline D. to enable nurses to have a voice in health care policy

B. to ensure knowledgeable, safe, comprehensive nursing care

Although all of the following are nursing responsibilities, which one would be expected of a nurse with a baccalaureate degree? A. providing direct physical care B. using research findings to improve practice C. administering medications as prescribed D. collaborating with other health care providers

B. using research findings to improve practice

A homeless client in the public health clinic has a strong body odor and is wearing clothes that are visibly soiled. What nursing diagnosis would be most appropriate for the nurse to identify? Bathing Self-care Deficit related to lack of access to bathing facilities as evidenced by a strong body odor Homelessness Syndrome related to lack of housing as evidenced by visibly soiled clothing Inadequate Hygiene related to homelessness as evidenced by client's stink Impaired Impulse Control related to poor socioeconomic conditions as evidenced by visibly soiled clothing

Bathing Self-care Deficit related to lack of access to bathing facilities as evidenced by a strong body odor

A homeless client in the public health clinic has a strong body odor and is wearing clothes that are visibly soiled. What nursing diagnosis would be most appropriate for the nurse to identify? Bathing Self-care Deficit related to lack of access to bathing facilities as evidenced by a strong body odor Homelessness Syndrome related to lack of housing as evidenced by visibly soiled clothing Inadequate Hygiene related to homelessness as evidenced by client's stink Impaired Impulse Control related to poor socioeconomic conditions as evidenced by visibly soiled clothing

Bathing Self-care Deficit related to lack of access to bathing facilities as evidenced by a strong body odor The most appropriate diagnosis would be "Bathing Self-care Deficit. The client is homeless and would not be able to access bathroom facilities. Homelessness has not been identified as a syndrome and there is only evidence of one problem. Inadequate hygiene has not been identified as a nursing diagnosis; furthermore, the word "stink" is an offensive term that must be avoided in nursing documentation. There is no evidence to suggest that the client has any issues with impulse control.

The nurse is performing an assessment on a newly admitted client and understands the importance of validating all data. When is the best time to validate such data?

Both during the collection and at the end of the collection

A nurse is justified in independently identifying and documenting which diagnosis related to impaired elimination? Bowel Incontinence Ulcerative Colitis Irritable Bowel Syndrome Small Bowel Obstruction

Bowel Incontinence Bowel incontinence is a NANDA-I approved nursing diagnosis under the domain of Elimination. Ulcerative colitis, irritable bowel syndrome, and small bowel obstruction are medical diagnoses.

A nurse has been providing discharge teaching to a client with type 1 diabetes. Which outcome indicates that the teaching has been effective? a. Before discharge, the client will attempt to administer a subcutaneous injection. b. By a certain date, the client will talk to a dietitian regarding information for a diabetic diet. c. Before discharge, the client will understand proper foot care and eye care. d. By a certain date, the client will verbalize signs and symptoms of hypoglycemia.

By a certain date, the client will verbalize signs and symptoms of hypoglycemia.

The registered nurse is teaching a community health class about illness prevention. Which statement reflects understanding of this concept? A. "I will take my medication for hypertension every day." B. "My family will call hospice for an evaluation." C. "It is important to enroll in a smoking cessation class." D. "If I am injured, I will call an ambulance."

C. "It is important to enroll in a smoking cessation class."

A client preparing to be a subject in a research study informs the nurse that the client would like to withdraw from the research study immediately. What is the best response by the nurse? A. "You have signed a consent form indicating that you will be a willing participant in the study." B. "If you don't continue with the study, there will be consequences for you, such as a lawsuit." C. "You have the right to withdraw your consent for being a research subject at any time." D. "If you will stay in the study, we will find monetary reimbursement for you."

C. "You have the right to withdraw your consent for being a research subject at any time."

The nurse is evaluating client health. Which client should the nurse determine to be exhibiting the most signs of health? A. A client with diabetes who is in denial of the diabetes and refuses to take insulin B. A client in acute emotional distress due to spouse's death C. A client with a leg amputation who performs activities of daily living with a prosthesis D. A client with depression who refuses to get out of bed

C. A client with a leg amputation who performs activities of daily living with a prosthesis

Which is an appropriately stated nursing intervention? A. Ambulate in the hall. B. Stand at bedside with assistance. C. Ambulate 30 ft (9 m) twice a day with the assistance of a walker. D. Ambulate with the assistance of a walker.

C. Ambulate 30 ft (9 m) twice a day with the assistance of a walker.

Which nursing group provides a definition and scope of practice for nursing? A. International Council of Nurses (ICN) B. American Academy of Neurology (AAN) C. American Nurses Association (ANA) D. The Joint Commission

C. American Nurses Association (ANA)

In what way can a nurse differentiate strong research from poor research? A. By conducting the research B. Through author dialogue C. By critiquing the study D. Through the nurse's own informal investigation

C. By critiquing the study

A 9-year-old child asks the nurse why he keeps breathing while he is asleep. The nurse knows which of the following gases stimulate normal respiration? A. Oxygen B. Hydrogen C. Carbon dioxide D. Helium E. Nitrogen

C. Carbon dioxide

Which is true of concepts? A. Concepts are derived through deductive and inductive reasoning. B. Concepts provide a means of testing knowledge. C. Concepts describe objects, properties, and events and the relationships among them. D. Concepts differentiate nursing from other disciplines.

C. Concepts describe objects, properties, and events and the relationships among them.

The process of considering an idea as a whole and then dividing it into smaller, more specific ideas is which? A. Theory B. Concept C. Deductive reasoning D. Inductive reasoning

C. Deductive reasoning

A nurse is completing a family assessment on a routine home health visit. There is a mother, father, and three children ages 2, 4, and 6 years of age. She watches the children individually as they play and interact with one another. She compares each child with the norms established for the age groups and determines and they are all within normal limits for their ages. The nurse would be utilizing which of the following theories as the basis for this part of the family assessment? A. General Systems Theory B. Adaptation Theory C. Developmental Theory D. Maslow's Theory

C. Developmental Theory

A parent has brought a 6-year-old child into the clinic. The parent is concerned that the child does not seem to skip as well as the other children in the child's class. In planning assessments and care for this child, the nurse would be best served by choosing which theory as a foundation for decision making? A. General systems theory B. Adaptation theory C. Developmental theory D. Maslow's theory

C. Developmental theory

The nurse is reading about nursing theorists for a class. The theory reveals the following, "Individuals who use self-care to sustain life and health, to recover from disease or injury, or to cope with its effects." Which individual developed this theory? A. Sister Callista Roy B. Virginia Henderson C. Dorothea Orem D. Florence Nightingale

C. Dorothea Orem

Which of the following actions should the nurse implement when working with a medically homeless client? A. Encourage client to utilize the free healthcare clinic. B. Assist the client in finding housing. C. Encourage the client to utilize the emergency room when ill. D. Assist the client in finding a job.

C. Encourage client to utilize the free healthcare clinic

A nursing theorist examines a hospital environment by studying each ward and how it works individually, and then relates this information to the hospital as a whole working entity. This is an example of the use of which theory? A. Adaptation theory B. Developmental theory C. General systems theory D. Psychosocial theory

C. General systems theory

A nurse educator is discussing the role of nursing based on the American Nurses Association (ANA). Which statement best describes this role? A. Nursing is a profession dependent upon the medical community as a whole. B. It is the role of the physician, not the nurse, to assist clients in understanding their health problems. C. It is the role of nursing to provide a caring relationship that facilitates health and healing. D. The essential components of professional nursing care are strength, endurance, and cure.

C. It is the role of nursing to provide a caring relationship that facilitates health and healing.

A nurse would like to study the effect that a new hand washing technique has on client infection rates. What is the independent variable? A. Nurse B. Wearing gloves C. New hand washing technique D. Client infection rates

C. New hand washing technique

What is the best explanation for the way evidence-based practice (EBP) has changed the way nursing care is delivered? A. Nurses now spend time looking up the best way to give nursing care. B. Nurses now have to take part in research. C. Nursing care now uses EBP as a means of ensuring quality care. D. Nursing care now incorporates research studies into client care.

C. Nursing care now uses EBP as a means of ensuring quality care.

Which nursing actions demonstrate the aim of nursing to facilitate coping? Select all that apply. A. Teaching a class on the nutritional needs of pregnant women B. Changing the bandages of a client who has undergone heart surgery C. Teaching a client and her family how to live with diabetes D. Assisting a client and his family to prepare for death E. Starting an intravenous line for a malnourished older adult client F. Providing counseling for the family of a teenager with an eating disorder

C. Teaching a client and her family how to live with diabetes D. Assisting a client and his family to prepare for death F. Providing counseling for the family of a teenager with an eating disorder

How are the first stages of the nursing process and nursing research linked? A. They will answer a posed question. B. Each begins with goal development. C. The nurse assesses problems initially. D. There is a period of evaluation.

C. The nurse assesses problems initially.

The postoperative patient's plan of care contains an intervention to ambulate twice a day 200 feet with assistance of one member of the healthcare team. The intervention was not completed one morning due to his pain. Which of the following would be the appropriate way to document the missed event? A. The patient was in too much pain to complete the morning ambulation. B. The patient complained of too much pain to ambulate and was returned to bed. C. The patient reported his pain at a level of 8/10 at 30 minutes after dosing. The patient was returned to bed and the physician was notified of the uncontrolled pain level with the current medication order. D. The patient was assisted to standing but reported severe pain and begged to return to bed. Patient was assisted to bed. The intervention was not completed due to patients severe pain level.

C. The patient reported his pain at a level of 8/10 at 30 minutes after dosing. The patient was returned to bed and the physician was notified of the uncontrolled pain level with the current medication order.

The patient has just received a central venous catheter placed by the physician. Which of the following should the nurse anticipate next? A. The patient will become short of breath. B. The patient will develop an arrhythmia. C. The patient will have an X-ray to confirm placement of the device. D. The patient will have a stat order for lab tests.

C. The patient will have an X-ray to confirm placement of the device.

Which statement best explains the importance of theoretic frameworks? A. Theoretic frameworks guide physiologic nursing care. B. Theoretic frameworks guide psychosocial nursing care. C. Theoretic frameworks advance nursing knowledge and practice. D. Theoretic frameworks advance the ethical aspects of practice.

C. Theoretic frameworks advance nursing knowledge and practice.

The need for university-based nursing education programs was brought to light during which important historical time? A. Spanish-American War B. World War I C. World War II D. Korean War

C. World War II

A nurse must possess several characteristics to be successful in this profession. Secondary to critical thinking skills, which is of great value? A. educational attainment such as Master's of Nursing B. delegation of responsibilities C. advocating for the client at all times D. good teamwork and team-building skills

C. advocating for the client at all times

A nurse admitting a client to the home health service is gathering information for the client's database. Within the database, the nurse gathers information on the client's health history, past surgeries, diet, exercise, psychosocial status, family and support persons, and spirituality. This information will assist in the development of the client's plan of care. This is considered a: A. psychosocial database. B. physical database. C. holistic database. D. limited database.

C. holistic database.

A prospective nursing student desires a career that will allow the opportunity to provide client care and to assist professional nurses with routine technical procedures. The prospective student needs to be employed in a full-time position quickly due to economic hardship. What type of nursing program would best suit this student? A. associate of science registered nursing program B. baccalaureate of science registered nursing program C. licensed or vocational nursing program D. diploma nursing program

C. licensed or vocational nursing program

From 1900 to 1940, research in nursing focused on: A. cost-effectiveness of nursing care. B. improving client care outcomes. C. nursing education and teaching. D. increasing workforce in the nursing profession.

C. nursing education and teaching.

Due to the rising cost of health care services, many procedures and treatments are being delivered in what type of setting? A. hospitals B. medical centers C. outpatient facilities D. community health care centers

C. outpatient facilities

The National Institute of Nursing Research (NINR) was established following a study in 1983. The center's purpose is to: A. advance the level of nursing research funding. B. investigate and expand nursing theory. C. promote research for health promotion. D. evaluate the effect of current research.

C. promote research for health promotion.

Which guideline should the nurse follow when assessing a client's blood pressure using a Doppler ultrasound?

Center the bladder of the cuff over the artery, lining up the artery marker on the cuff with the artery.

A nurse has selected a nursing diagnosis and is preparing to validate it. With whom would the nurse do this? Another staff nurse Client's health care provider Client The unit's nurse manager

Client

The nurse is assessing the client's blood pressure (BP) and heart rate (HR) for orthostatic hypotension. In which step should this nurse intervene because of potential danger?

Client stands at bedside, becomes pale, diaphoretic.

5. A client with food poisoning has the nursing diagnosis "diarrhea." Which expected client outcome most directly demonstrates resolution of the problem?

Client will have formed stools within 24 hours.

The nurse is examining the assessment data of a client and diagnoses a problem of impaired tissue perfusion based on the following assessment data cues: left foot cool and pale with capillary refill > 3 seconds, diminished dorsalis pedis and posterior tibial pulses, client reports cramping pain in left foot. The nurse is doing what? Clustering significant data cues Formulating a nursing diagnosis Validating the nursing diagnosis Identifying contributing factors

Clustering significant data cues

An informatics nurse specialist working at an ambulatory care center is interviewing clients about their experiences in using the center's patient portal. A majority of the clients have voiced dissatisfaction, stating that they "find the display screen confusing and are not sure what to click on to get the necessary information." They also talk about difficulties in trying to find their test results. Based on this information, the nurse specialist identifies a need to talk with the information technology team about ways to make the portal easier for clients to use. The informatics nurse specialist is demonstrating which ANA informatics competency? Professional practice evaluation Collaboration Resource utilization Environmental health

Collaboration

While developing a plan of care for a client, what should the nurse do before selecting a nursing diagnosis? Collect client subjective and objective data. Establish short- and long-term client goals. Perform a focused assessment related to the reason for admission. Verify the primary care provider's written orders.

Collect client subjective and objective data.

A community group has requested the public health nurse to present a program describing the advised schedule of immunizations for children. To plan for this program, what nursing diagnosis would be most appropriate for the nurse to select? Readiness for Enhanced Knowledge: Childhood Immunizations Ineffective Health Maintenance related to lack of knowledge of childhood immunizations Risk for Infection Transmission related to lack of immunizations Risk for Complications related to childhood illnesses

Collect client subjective and objective data. Nursing diagnoses are developed as the second step of the nursing process. The first step is to collect all assessment data so that appropriate actual or potential nursing problems can be selected and addressed in the client's plan of care. Nursing diagnoses are not related to the medical diagnosis or the specific written orders from the primary care provider. Goals can only be established after the problem is identified. Although assessment--collecting subjective and objective client data--is necessary before developing nursing diagnoses, this assessment does not necessarily have to be a focused assessment.

The nurse is planning to do a physical assessment on a newly admitted client. The assessment will be a review of systems. This means the nurse plans to:

Complete an exam of all body systems.

A client with congestive heart failure has dyspnea while ambulating to the bathroom. The nurse selects the nursing diagnosis of "Activity Intolerance" to address this health problem. Which etiology would be appropriate to select for this nursing diagnosis? Compromised oxygen transport Inadequate motivation Cardiac disease Fluid overload

Compromised oxygen transport

An informatics nurse specialist is working with a team to design a clinical information system. To ensure system usability, the team would make sure that the language meets which criterion? Select all that apply. Conciseness Clarity Jargon-like Familiarity for the user Computer-based

Conciseness Clarity Familiarity for the user

An informatics nurse is demonstrating how to use an updated version of an electronic documentation system to a group of staff nurses. The nurse shows the staff that they can follow the same steps that they used with the previous system but need to add one additional final step to the process. The informatics nurse's actions reflect which concept of usabilty? Simplicity Consistency Naturalness Minimizing cognitive load

Consistency

A client reports not having a bowel movement for 7 days, followed by a day of small, loose stools. How does the nurse define the health problem? Constipation related to irregular evacuation patterns Readiness for Enhanced Nutrition related to constipation Bowel incontinence related to depressive state Diarrhea related to client report of small, loose stools

Constipation related to irregular evacuation patterns This client is experiencing constipation, which is having infrequent or difficult bowel movements, which the nurse believes to be caused by an irregular evacuation pattern--not having a bowel movement for 7 days. There is no evidence that the client needs, much less is ready for, enhanced nutrition related to constipation or that, by implication, that the client's constipation is caused by poor nutrition. There is no evidence that the client is experiencing bowel incontinence, depression, or diarrhea.

A nurse in the emergency room, who is unfamiliar with pediatric clients, assesses the vital signs of a 1-month old infant with a heart rate of 124 and a respiratory rate of 36. What would be the most appropriate measure for the nurse to take to analyze the significance of the infant's vital signs? Report the vital signs and allow the emergency room physician to determine the significance. Consult reference materials to determine the normal vital signs for 1-month old infants. Ask the mother if the infant's heart rate is higher than normal. Perform a complete physical assessment to determine the cause of the elevated vital signs.

Consult reference materials to determine the normal vital signs for 1-month old infants. It is part of nursing practice to interpret the significance of assessment data by comparing it to standards. The nurse should consult reference materials to determine the normal range of vital signs for this client. Deferring to the emergency room physician is unprofessional and may result in harm to the client. Asking the mother if the infant's vital signs are higher than normal is unprofessional practice. A complete physical assessment is not necessary at this time.

A newly graduated nurse is unable to determine the significance of data obtained during an assessment. What would be the nurse's most appropriate action? Consult with a more experienced nurse. Continue to collect assessment data. Document the data for future reference. Contact the client's health care provider.

Consult with a more experienced nurse. A newly graduated nurse does not have the experience to interpret all data. The nurse must recognize when a consult with a more experienced nurse is needed. There is no evidence that the nurse needs to collect more data. The nurse must document the data, but if the data are significant and the nurse does not recognize this and takes no action, it could harm the client. There is no need to contact the health care provider at this time.

While performing an assessment, the nurse recognizes that the nurse's own personal biases may be interfering with the collection of data. What step should the nurse take to ensure that the information is factual and accurate?

Consult with another nurse for that colleague's description of the assessment or observations.

33. Which intervention performed by the nurse is appropriate for assisting a client in meeting physiological needs based on Maslow's Hierarchy of Needs?

Cutting up food and opening drink containers for the client

The nurse is conducting a class for nursing assistants. One of the students asks the nurse why blood pressure, pulse, and temperature are called "vital signs." Which explanations would the nurse offer the student? A. "They are called vital signs because they are the first things the nurse does when admitting a patient to the hospital." B. "They are called vital signs because the blood pressure is very important." C. "The term has just evolved over time without a clear definition for why." D. "Because significant deviation from normal is not compatible with life."

D. "Because significant deviation from normal is not compatible with life."

In what time period did nursing care as we now know it begin? A. pre-civilization B. early civilization to 16th century C. 16th to 17th century D. 18th to 19th century

D. 18th to 19th century

A nurse subscribes to the Basic Needs theory. Which action by the nurse exemplifies this theory? A. Assisting the client in coping with a new diagnosis B. Assisting the client in recognizing signs of infection and changing dressings C. Assisting the client in finding a clean, well-lit residence D. Assisting the client to the bedside commode

D. Assisting the client to the bedside commode

Which challenge associated with technology most affects nurses today? A. The use of electronics (such as a vital sign machine) to monitor clients B. The use of computerized physician order entry (CPOE) C. The identification of superbugs (such as MRSA) D. Client privacy issues within the social media realm

D. Client privacy issues within the social media realm

A nurse researcher develops a foreground question in preparation for conducting a research study. The question is: "In clients with intravenous catheters, how does replacing administration sets every 72 hours (h) compared with other frequent intervals (24h, 48h, or 96h) decrease infection rates?" Applying the PICO framework, which part of the statement reflects the "O"? A. Clients with intravenous catheters B. Replacing administration sets every 72 hours C. Replacing administration sets at other intervals D. Decrease infection rates

D. Decrease infection rates

Nurses who enlist their services to the military are able to do so thanks to the work of which organizer of health care? A. Linda Richards B. Florence Nightingale C. Theodor Fliedner D. Dorothea Dix

D. Dorothea Dix

The nurse is performing an extensive dressing change on a client with burns. The nurse explains each step as it is being performed. The nurse is acting in which role by providing explanation of each step? A. Caregiver B. Client advocate C. Decision-maker D. Educator

D. Educator

The nurse that has been in practice for 15 years is able to use intuition when providing care to clients and also utilizes resources and demands to attain specific goals. According to Author Patricia Benner, in From Novice to Expert (1984), what stage of practice is the nurse displaying? A. Novice B. Competent C. Proficient D. Expert

D. Expert

Of the following client needs, which need takes priority in the nurse's delivery of care? A. Confusion B. Grief C. Anxiety D. Oxygenation

D. Oxygenation

The nurse researcher would like to gather data about the attitudes of young adults on spirituality and health care. What is the most effective form of research on this topic? A. Quantitative research B. Methodologic survey C. Delphi study D. Qualitative research

D. Qualitative research

The second step in implementation of evidence-based practice includes systematic review. To complete a systematic review of the literature, what must the nurse do? A. Ask a question about a clinical practice. B. Provide a statical analysis for studies. C. Recommend best practices for client care. D. Summarize findings from multiple studies that are related to a particular nursing practice.

D. Summarize findings from multiple studies that are related to a particular nursing practice.

A group of nursing students has attended a presentation about the National Student Nurses' Association (NSNA). Which statement by the group indicates that they have understood the information presented? A. The NSNA is run by a group of registered nurses. B. The organization is funded by the national government. C. The NSNA contributes to the improvement of public health. D. The organization provides programs of current professional interest.

D. The organization provides programs of current professional interest

The nurse is preparing to submit a research project to the institutional review board (IRB). The nurse understands that if the IRB were to reject the research project, it would most likely be for which reason? A. The proposed data collection method is invalid. B. The estimated cost of conducting the research is excessive. C. The research is unlikely to be publishable in a peer-reviewed journal. D. The project lacks sufficient safeguards to protect human subjects.

D. The project lacks sufficient safeguards to protect human subjects.

A conceptual framework is defined as: A. an explanation of nursing and nursing practice. B. a set of phenomena and related abstractions. C. a foundation for nursing skills and care. D. a set of concepts and propositions.

D. a set of concepts and propositions.

A student is choosing an educational path and desires a nursing degree with a track for community nursing and leadership and that allows for classes in liberal arts. The student would best be suited in which type of program? A. licensed practical nursing program B. certification in a nursing specialty C. diploma nursing program D. baccalaureate program

D. baccalaureate program

The nurse is caring for a postoperative client. The health care provider has written a prescription for a pain medication, and the prescription gives a dosage range for the amount the nurse may give depending on the severity of the client's pain. This type of functioning within the health care team is called: A. authoritative functioning. B. independent functioning. C. assistive functioning. D. collaborative functioning.

D. collaborative functioning.

An informatics nurse specialist is describing the framework underlying informatics practice. Which component of the framework would the nurse specialist describe as discrete entities without interpretation? Data Knowledge Information Wisdom

Data

The client, who is 8 weeks pregnant as the result of a rape, tells the nurse, "I do not want to have this baby, but I have always believed that abortion is a sin. I don't know what to do." What nursing diagnosis would be most appropriate for the nurse to formulate? Decisional Conflict related to conflict with moral beliefs as evidenced by the client's statement Hopelessness related to inability to decide a course of action as evidenced by the client's statement Complicated Grieving related to mental trauma as evidenced by the client's inability to make a decision Ineffective Coping related to rape trauma syndrome as evidenced by client's inability to make a decision

Decisional Conflict related to conflict with moral beliefs as evidenced by the client's statement The client's statement indicates that it is difficult for the client to reach a decision because of the client's moral beliefs. The client is not expressing hopelessness or demonstrating ineffective coping or complicated grieving. The client may be suffering from rape trauma syndrome, but the assessment data do not lead to that diagnosis.

A nurse is caring for a client admitted with dehydration after completing a triathlon in a hot, dry climate. The nurse identifies an appropriate nursing diagnosis for this client as "Deficient Fluid Volume related to insufficient fluid intake as evidenced by blood pressure of 84/46 mm Hg, heart rate of 145 beats/min, concentrated urine, and client reporting drinking 200 mL of water during the 4-hour event." Which is the problem statement in this nursing diagnosis? Deficient fluid volume Insufficient fluid intake Blood pressure of 84/46 mm Hg, heart rate of 145 beats/min, concentrated urine Hot, dry climate

Deficity fluid volume

Which component of a nursing diagnosis gives additional meaning to the nursing diagnosis? Composition Descriptors Dysfunction Qualifications

Descriptors Descriptors are words used to give additional meaning to a nursing diagnosis through adding conditions and showing relationships between events. The other answers listed are not components of a nursing diagnosis.

An informatics nurse specialist is working as part of a team that will be developing and implementing a new client assessment tool. During which phase of the system development lifecycle would the team be integrating information about workflow patterns, standard terminology, and recommendations for screen layout from supportive research? Analyze and plan Design and build Test Train

Design and build

An informatics nurse specialist is working on a team that is considering a new technological system for the facility. Which aspect would be most important for the team to do as the first step? Determine the need or problem to be solved Identify the use of standard terminology Conduct testing of the system Train those who will be using the system

Determine the need or problem to be solved

While caring for a client admitted with a Clostridium difficileinfection, the nurse notes that the client has had three loose bowel movements in 3 hours. What would be the most appropriate nursing diagnosis to address this health problem? Diarrhea related to infectious processes secondary to C. difficile infection as evidenced by three loose bowel movements in 3 hours Risk for Infection Transmission related to high potential for communicability Fluid Volume Excess related to diarrhea as evidenced by three loose bowel movements in 3 hours Risk for Injury related to urgent need for bowel evacuation

Diarrhea related to infectious processes secondary to C. difficile infection as evidenced by three loose bowel movements in 3 hours

The nurse is assessing a client who was just admitted to the unit following an abdominal hysterectomy. On which assessment finding would the nurse base the priority diagnosis? Dressing intact with slight bloody discharge present Client reports being very sleepy Abdominal area soft with diminished bowel sounds throughout Diminished breath sounds in left lower lobe Skin warm and dry

Diminished breath sounds in left lower lobe

A client undergoing chemotherapy for breast cancer has lost all hair. The client states, "I cannot stand to see myself without hair. I am disgusting." What would be the most appropriate nursing diagnosis for the nurse to use to address this client's problem? Disturbed Body Image related to breast cancer Disturbed Body Image related to loss of hair Disturbed Body Image as evidenced by client's refusal to look at self Disturbed Body Image as evidenced by client's negative comments

Disturbed Body Image related to loss of hair The client has a problem with body image because of the loss of hair. The evidence would be the client's statement. The etiology cannot be a medical diagnosis, so the etiology of breast cancer would be incorrect. The other two statements do not contain an etiology. Nursing diagnoses must identify an etiology to direct the client's care

When assessing a client's respiratory rate, the nurse should take which action?

Do it immediately after the pulse assessment so the client is unaware of it.

A nurse is assessing the lungs of a client and auscultates soft, low-pitched sounds over the base of the lungs during inspiration. What would be the nurse's next action?

Document normal breath sounds.

Which statement describes diastolic blood pressure?

During ventricular relaxation, blood pressure is due to elastic recoil of the vessels

The nursing instructor is teaching nursing students about myocardial contractility and ejection fractions. What diagnostic tests can determine client ejection fractions? Select all that apply.

Echocardiogram Cardiac catheterization Magnetic resonance imaging

A nurse documents the following nursing diagnosis on a client's plan of care: "Fluid Volume Deficit related to gastrointestinal upset from food poisoning as evidenced by vomiting and diarrhea for the past three days, slow skin turgor, and weight loss." The nurse identifies which part of the statement as the etiology? Fluid volume deficit Gastrointestinal upset from food poisoning Slow skin turgor Vomiting

Gastrointestinal upset from food poisoning

A nurse documents the following nursing diagnosis on a client's plan of care: "Fluid Volume Deficit related to gastrointestinal upset from food poisoning as evidenced by vomiting and diarrhea for the past three days, slow skin turgor, and weight loss." The nurse identifies which part of the statement as the etiology? Fluid volume deficit Gastrointestinal upset from food poisoning Slow skin turgor Vomiting

Gastrointestinal upset from food poisoning The etiology identifies the physiological, psychological, sociological, spiritual, and environmental factors believed to be related to the problem as either a cause or a contributing factor. The problem is fluid volume deficit. Vomiting and poor skin turgor are defining characteristics.

A nurse documents the following nursing diagnosis on a client's plan of care: "Readiness for Enhanced Breast-Feeding." The nurse has identified which type of nursing diagnosis? Problem-focused Risk Health promotion Syndrome

Health promotion A health promotion nursing diagnosis is a clinical judgment concerning motivation and desire to increase well-being and to actualize human health potential. These responses are expressed by a readiness to enhance specific health behaviors and can be used in any health state. A problem-focused nursing diagnosis is a clinical judgment concerning an undesirable human response to a health condition or life process that exists in an individual, family, group, or community. A risk nursing diagnosis is a clinical judgment concerning the vulnerability of an individual, family, group, or community for developing an undesirable human response to health conditions or life processes. A syndrome nursing diagnosis is a clinical judgment concerning a specific cluster of

An informatics nurse specialist is recommending the addition of an alert system tool to the facility's patient portal. The tool would be designed to send alerts to the client to schedule routine screenings and immunizations. This recommendation most likely reflects which ANA informatics competency? Health teaching and health promotion Consultation Quality of practice Leadership

Health teaching and health promotion

A nurse is caring for a toddler who has been treated on two different occasions for lacerations and contusions due to the parents' negligence in providing a safe environment. What is an appropriate nursing diagnosis for this client? High Risk for Injury related to abusive parents High Risk for Injury related to impaired home management Child Abuse related to unsafe home environment High Risk for Injury related to unsafe home environment

High Risk for Injury related to unsafe home environment The nursing diagnosis "High Risk for Injury related to unsafe home environment" is appropriate because it contains the NANDA-I nursing diagnosis problem statement and the etiology of the problem. High Risk for Injury related to abusive parents is accusatory and may not be accurate. High Risk for Injury related to impaired home management does not accurately identify the etiology of the problem. Child Abuse is not a NANDA-I approved nursing diagnosis.

A client recently diagnosed with pancreatic cancer tells the nurse, "I don't see any hope for my future." What would be the most appropriate nursing diagnosis for the nurse to formulate to address this health problem? Hopelessness related to difficulty coping secondary to pancreatic cancer diagnosis Disturbed Self-Concept related to pancreatic cancer diagnosis Ineffective Health Maintenance related to being overwhelmed by cancer diagnosis Knowledge Deficit: Cancer treatment options related to new diagnosis

Hopelessness related to difficulty coping secondary to pancreatic cancer diagnosis The client is expressing a lack of hope for the future, which makes "Hopelessness" an appropriate nursing diagnosis. There is no evidence that the client has a disturbed self-concept. There is no evidence that the client is not effectively caring for health. The client does not verbalize a desire to learn about treatment options.

Which best describes the purpose of nursing diagnoses? Identification of client problems that nurses can treat independently Identification of signs and symptoms that identify diseases Identification of client problems that require collaboration with other health care professionals to treat Identification of actual client problems, not including potential problems

Identification of client problems that nurses can treat independently

Which activities does the nurse perform during the diagnosing stage? Select all that apply. Establish plan priorities with the client and family. Identify factors contributing to the client's health problem. Prioritize the client's health problems with input from the client. Validate the identified health problems with the clients. Collect data to monitor quality and effectiveness of nursing practice.

Identify factors contributing to the client's health problem. Prioritize the client's health problems with input from the client. Validate the identified health problems with the clients. During the diagnosis stage, the nurse identifies factors contributing to the client's health problem, validates the identified health problems with the client, and prioritizes the client's health problems with input from the client. The nurse establishes plan priorities with the client and family during the outcome identification and planning. The nurse collects data to monitor the quality and effectiveness of nursing practice during the evaluation stage.

31. Which action should the nurse perform during the planning phase of the nursing process?

Identify measurable goals or outcomes.

When developing a nursing diagnosis for a client, which should the nurse do first? Identify the significant data Cluster the cues Synthesize cue clusters Validate the diagnosis

Identify the significant data

An informatics nurse is assisting with the evaluation of a newly implemented system for electronic documentation of client assessments. The nurse is planning to involve staff nurses in this process. When beginning the evaluation process, the informatics nurse would focus on which area first? Identifying what will be evaluated Developing the question related to what data is to be collected Reviewing available databases related to the topic Collecting the necessary data

Identifying what will be evaluated

Which would be an appropriate nursing diagnosis for a client with cachexia and decreased weight? Anorexia Nervosa Lack of Adequate Nutrition Weight Loss Imbalanced Nutrition: Less than Body Requirements

Imbalanced Nutrition: Less than Body Requirements

Which would be an appropriate nursing diagnosis for a client with cachexia and decreased weight? Anorexia Nervosa Lack of Adequate Nutrition Weight Loss Imbalanced Nutrition: Less than Body Requirements

Imbalanced Nutrition: Less than Body Requirements The most appropriate nursing diagnosis would be Imbalanced Nutrition: Less than Body Requirements. Anorexia Nervosa is a medical diagnosis. Lack of Adequate Nutrition and Weight Loss are not standard terminology for nursing diagnoses.

A nurse is interviewing an older adult client who has experienced a drastic weight loss following a cerebrovascular accident (CVA). The client states, "I have trouble getting groceries because I can no longer drive, so I don't have much food in the house." Based on this evidence, what would be the most appropriate nursing diagnosis? Imbalanced Nutrition: Less than Body Requirements related to difficulty in procuring food Imbalanced Nutrition: Less than Body Requirements related to drastic weight loss Imbalanced Nutrition: Less than Body Requirements related to CVA Imbalanced Nutrition: Less than Body Requirements related to decreased appetite

Imbalanced Nutrition: Less than Body Requirements related to difficulty in procuring food The client relates the drastic weight loss to the inability to bring food into the house. The client's statement is the most appropriate etiology for the nursing diagnosis. Drastic weight loss is the evidence of imbalanced nutrition. CVA is the medical diagnosis. The client could have had a CVA and still have the ability to grocery shop. There is no evidence that the client has lost appetite.

The nurse formulates for a client the nursing diagnosis of: Impaired Physical Mobility related to postoperative pain as evidenced by difficulty ambulating. Which component of this nursing diagnosis is the descriptor? Impaired Physical mobility Postoperative pain Difficulty ambulating

Impaired Descriptors are words used to give additional meaning to a nursing diagnosis. They describe the change in condition, state of the client, or some qualification of the specific nursing diagnosis. In this example, the word "impaired" is a descriptor. Physical mobility is the diagnostic label. Postoperative pain is the related factor or etiology contributing to the problem. Difficulty ambulating is the defining characteristic, or the cue that supports the existence of the problem.

A nurse, who is caring for a client admitted to the patient care unit with acute abdominal pain, formulates the care plan for the client. Which nursing diagnosis is the priority for this client? Impaired Comfort Disturbed Body Image Disturbed Sleep Pattern Activity Intolerance

Impaired Comfort Acute pain in the abdomen disturbs all the systems of the body. Relieving the pain should be the nurse's first priority. According to Maslow, physiologic needs are the highest priority. The client may have Disturbed Body Image, Disturbed Sleep Pattern, or Activity Intolerance, but all these are secondary to pain.

A nurse is caring for a client diagnosed with arthritis. The client is experiencing pain, which is interfering with the client's ability to ambulate. The nurse accurately documents which nursing diagnosis in the client's records? Ineffective Movement related to arthritis Impaired Movements due to pain Impaired Physical Mobility related to pain Ineffective Physical Mobility due to pain

Impaired Physical Mobility related to pain "Impaired Physical Mobility related to pain" is the correct nursing diagnosis because it consists of an accurate descriptor, diagnostic label, and related factor. "Ineffective Movement related to arthritis" is an incorrect entry because the descriptor is incorrect and the diagnostic label is not approved. "Impaired Movements due to pain" is an inaccurate entry because the descriptor is inaccurate and the related factor is not written using approved words. "Ineffective Physical Mobility due to pain" has an erroneous diagnostic label and the related factors are written incorrectly.

An informatics nurse specialist is working as part of a team to develop and implement a new electronic documentation and reporting system at the clinic. The team has analyzed the situation, created a plan based on supporting evidence, created a design, built the program, and has worked out issues with the system. Staff who will be using the system have received education about the system and how it works, as well as how to use it. The team would proceed to which phase of the system development lifecycle next? Test Implement Maintain Evaluate

Implement

A nurse is reading a journal article about health information technology and the need for this technology to demonstrate meaningful use. Which information would the nurse anticipate reading about as reflective of meaningful use? Select all that apply. Improvement in health care quality Increased health disparities Greater client engagement Reduction in privacy breaches of client information Reduced health of populations

Improvement in health care quality Greater client engagement Reduction in privacy breaches of client information

9. A nurse is using a standardized plan of care for a client. Which action would be most important for the nurse to do?

Individualize the plan to the client.

A nurse is caring for a client who has pneumonia. What is an appropriate nursing diagnosis? Impaired Respiration Ineffective Airway Clearance Altered Airway Infection (Pulmonary)

Ineffective Airway Clearance

The nurse is assessing a 3-week-old infant who has not gained weight since birth. The infant's bowel sounds are present in all quadrants and breath sounds are clear to auscultation. The infant's mother reports that the child cries much of the night but sleeps better in the daytime. The mother reports that the child only breastfeeds about four times in a 24-hour period and that the mother doesn't seem to have much milk. Which nursing diagnosis would be of highest priority for this client? a.Readiness for Enhanced Parenting b.Ineffective Breastfeeding c.Risk for Impaired Parenting d.Disturbed Sleep Pattern e.Impaired Comfort

Ineffective Breastfeeding

Which is the best example of a nursing diagnosis? Gastroesophageal Reflux related to low stomach pH as evidenced by foul breath and burning sensation in throat. Ineffective Airway Clearance as evidenced by client not speaking. Ineffective Breastfeeding related to latching as evidenced by nonsustained suckling at the breast. Cellulitis related to infection as evidenced by warm, reddened skin.

Ineffective Breastfeeding related to latching as evidenced by nonsustained suckling at the breast.

A client with diabetes mellitus has been admitted to the hospital in diabetic ketoacidosis. During the admission assessment of the client, the nurse learns that the client is not following the prescribed therapeutic regimen. The client states, "I don't really have diabetes. My doctor overreacts." What is the most appropriate diagnosis for this client's health problem? Risk for Unstable Blood Glucose related to client's reluctance to manage the diabetic regimen Ineffective Health Maintenance related to client's denial of illness Risk for Injury related to client's mismanagement of disease Ineffective Coping related to client's inability to manage the diabetic regimen

Ineffective Health Maintenance related to client's denial of illness

A client with diabetes mellitus has been admitted to the hospital in diabetic ketoacidosis. During the admission assessment of the client, the nurse learns that the client is not following the prescribed therapeutic regimen. The client states, "I don't really have diabetes. My doctor overreacts." What is the most appropriate diagnosis for this client's health problem? Risk for Unstable Blood Glucose related to client's reluctance to manage the diabetic regimen Ineffective Health Maintenance related to client's denial of illness Risk for Injury related to client's mismanagement of disease Ineffective Coping related to client's inability to manage the diabetic regimen

Ineffective Health Maintenance related to client's denial of illness The most appropriate diagnosis is Ineffective Health Maintenance related to client's denial of illness. The data point to the fact that the client is not managing the diabetes, since the client denies that a problem exists. The client is at risk for unstable blood glucose, but the client's denial is the underlying problem. Risk for Injury relates to safety issues. It is also inappropriate documentation to say the client is "mismanaging" the illness. Ineffective Coping could be an appropriate diagnosis, but the client is not "unable" to manage the illness, just unwilling.

A teenager on life support after a diving accident has no brain wave activity. The parents tell the nurse they are sure their child will wake up soon. Which nursing diagnosis would the nurse identify to assist the parents of the child? Interrupted Family Processes related to inability to accept their child's inevitable death as evidenced by the parents' statement that their child will wake soon Interrupted Family Processes related to brain death of their child as evidenced by parents' refusal to accept the inevitable Death Anxiety related to anticipated death of child as evidenced by child having no brain wave activity Death Anxiety related to dysfunctional family processes as evidenced by parents' refusal to acknowledge the child's condition

Interrupted Family Processes related to inability to accept their child's inevitable death as evidenced by the parents' statement that their child will wake soon

The nurse is admitting a client who is unable to identify person, place, or time. To properly analyze these data, what action must the nurse take? Determine the client's medical diagnosis for clarification. Interview the client's family to assess the client's usual level of cognition. Assess the client's vital signs to determine the client's baseline. Ensure precautions are taken to prevent injury to the client.

Interview the client's family to assess the client's usual level of cognition. To properly analyze the assessment data, the nurse must compare them against the client's normal or baseline data. The family is the best informant for a client with cognitive impairment. The medical diagnosis is not necessary to determine whether the client's condition is abnormal for the client. The nurse should obtain the vital signs, but doing so will not give an indication of the client's usual level of cognition. Ensuring the client's safety is an important nursing intervention but will not assist in analyzing these data.

The nurse is reviewing the morning laboratory test results for a client with cardiac problems. Which finding is a priority to report to the healthcare provider?

K+ 3.1 mEq/L

A nurse is planning education about prescription medications for a client newly diagnosed with asthma. What nursing diagnosis would be most appropriate for the nurse to select? Knowledge Deficit: Medications related to new medical diagnosis Ineffective Airway Clearance related to bronchial constriction Noncompliance related to deficient knowledge of a new medical diagnosis Anticipatory Grieving related to chronic illness management

Knowledge Deficit: Medications related to new medical diagnosis To most appropriately address the client's health problem, the nurse should educate the client about the new medications the physician has prescribed to treat the asthma. Ineffective Airway Clearance refers to the physiologic processes of asthma. There is no evidence of noncompliance. There is no indication that the client is having difficulty dealing with the diagnosis.

The nurse is preparing to assess the peripheral pulse of an adult client. Which action is correct?

Lightly compress the client's radial artery using the first, second, and third fingers

A nurse needs to measure the blood pressure of a client with an electronic manometer. Which of the following advantages does an electronic manometer provide over an aneroid manometer or mercury manometer?

No stethoscope is required.

The nurse is assessing an adult who has a pulse rate of 180 beats/min. Which action should the nurse take next?

Notify the health care provider of tachycardia

A nurse documents the following in the client chart: Risk for Decreased Cardiac Output related to myocardial ischemia. This is an example of what aspect of client care? Nursing diagnosis Nursing assessment Medical diagnosis Collaborative problem

Nursing diagnosis The nursing diagnosis statement is worded by stating the client problem (using NANDA-I approved diagnoses) that the nurse is able to treat followed by the etiology of the problem. Nursing assessment refers to the collection of data. A medical diagnosis identifies diseases, whereas nursing diagnoses focus on unhealthy responses to health and illness. Nurses cannot treat medical diagnoses independently. Collaborative problems are the primary responsibility of nurses. Unlike nursing diagnoses, with collaborative problems, the prescription for treatment comes from nursing, medicine, and other disciplines.

14. A nurse is caring for a client with congestive heart failure. The nurse manager informs the nurse that the client was enrolled in a clinical trial to assess whether a 10-minute walk, three times per day, leads to expedited discharge. What type of evaluation best describes what the researchers are examining?

Outcome

12. What are specific measurable and realistic statements of goal attainment?

Outcomes

17. A nurse is writing outcomes for a client who is scheduled to ambulate following hip replacement surgery. Which is a correctly written outcome for this client?

Over the next 24-hour period, the client will walk the length of the hallway assisted by the nurse.

A nurse is measuring the apical pulse of a client. Where should she place the diaphragm of her stethoscope in this assessment?

Over the space between the fifth and sixth ribs on the left midclavicular line

A client has been diagnosed with a recent myocardial infarction. What collaborative problem would be the priority for the nurse to address? PC: Decreased Cardiac Output related to cardiac tissue damage PC: Disturbed Body Image related to decreased activity tolerance PC: Activity Intolerance related to decreased oxygenation capacity PC: Fear related to new diagnosis of myocardial infarction

PC: Decreased Cardiac Output related to cardiac tissue damage All these collaborative problems may be indicated for a client with a recent myocardial infarction; however, priority must be given to life-threatening issues. Decreased cardiac output is the only life-threatening problem among the answer options, so it must be the priority.

A client whose care plan includes a nursing diagnosis of "Risk for Infection related to a disruption of skin integrity secondary to abdominal surgery" is displaying redness, edema, and warmth at the surgical site. What would be the nurse's most appropriate revision of the care plan? PC: Infection related to disrupted skin integrity secondary to abdominal surgery Risk for Infection related to a disruption of skin integrity secondary to abdominal surgery to be treated by an antibiotic Wound Infection related to infectious processes Infection as evidenced by redness, edema, and warmth at the surgical site

PC: Infection related to disrupted skin integrity secondary to abdominal surgery

An informatics nurse specialist is conducting an orientation for the staff of a primary care provider's office about a new web-based tool that they will be implementing. The goal of the tool is to promote patient engagement. The informatics nurse specialist is most likely orienting the staff to which system? Patient portal Telehealth Telemedicine Telecare

Patient portal

An informatics nurse specialist is preparing a presentation for a local community group about advances in technology in health care. Part of the presentation will focus on technological advances to promote greater client participation in managing health. Which component would the nurse likely describe as playing a major role? Patient portal Electronic health record Clinical information system Predictive analytics

Patient portal

An informatics nurse specialist is preparing a presentation about patient portals for a group of primary care providers in an office practice. When describing the tasks that can be accomplished by clients using the portal, which activity would the informatics nurse specialist include? Select all that apply. Paying their bill Checking their lab test results Scheduling appointments Talking with the provider face-to-face Receiving screening reminders

Paying their bill Checking their lab test results Scheduling appointments Receiving screening reminders

A nurse is taking the vital signs of a 9-year old child who is anxious about the procedures. Which nursing action would be appropriate when assessing this child?

Perform the blood pressure measurement last

An informatics nurse is involved in testing an update to a clinical information system. Numerous caregivers are using the system at the same time. The testing is being done to see if the system can handle the large amount of users at one time. The nurse is involved with which type of testing? Unit Function Performance User-acceptance

Performance

An informatics nurse is part of a team that is testing a new electronic health record system. The testing involves large groups of health care providers who will be using the system. During this testing, the system stalls and fails to respond when large numbers of providers are using the system at the same time. Which phase of testing is being conducted? Integration Function Performance User acceptance

Performance

A nurse suspects that a client has a self-care deficit, but needs more data to confirm this diagnosis. What nursing diagnosis would the nurse write for this client? Actual Potential Possible Apparent

Possible

An informatics nurse specialist is extracting data from the facility's electronic health record in an attempt to identify clients at risk for developing catheter-related bloodstream infections. When gathering this data, the nurse specialist is using which technique? Data visualization Predictive analytics Big data Genomics

Predictive analytics

Which nursing diagnosis is written incorrectly as a result of the health problem and etiology being reversed? Pain related to tissue trauma and inflammation Risk for Injury related to lack of knowledge of crutch walking Risk for Disturbed Body Image related to decreased ability to cope with surgical removal of right breast Prolonged Immobility related to impaired skin integrity

Prolonged Immobility related to impaired skin integrity

Which nursing diagnosis is written incorrectly as a result of the health problem and etiology being reversed? Pain related to tissue trauma and inflammation Risk for Injury related to lack of knowledge of crutch walking Risk for Disturbed Body Image related to decreased ability to cope with surgical removal of right breast Prolonged Immobility related to impaired skin integrity

Prolonged Immobility related to impaired skin integrity Impaired Skin Integrity related to prolonged immobility is the correct format. Prolonged immobility contributes (etiology) to impaired skin integrity (problem), but the impaired skin integrity does not contribute to prolonged immobility. The other nursing diagnoses are correct.

An informatics nurse specialist is conducting a seminar on the emergence of telehealth and its importance in health care delivery. The nurse specialist determines that additional discussion is needed when the group identifies which aspect as an advantage of this technology? Promote the development of new health care services Provides a new way to delivery existing health care Helps to save time and money Aids in reducing time spent away from work

Promote the development of new health care services

The nurse is preparing the client to use the hypothermia blanket. How does the nurse measure the client's temperature while the blanket is in use?

Rectal probe continuously

Which error has the nurse made in formulating the following nursing diagnosis: Prolonged Immobility related to impaired skin integrity as evidenced by an open area with a 1-inch diameter on the right buttocks surrounded by a 1-inch margin of redness; wound surface clean and beefy red; no drainage or foul odor detected. Wrote the diagnosis in terms of a need rather than a client response Reversed the health problem and the etiology Omitted the defining characteristics of the client health problem Identified environmental factors rather than client factors as the problem

Reversed the health problem and the etiology The nurse has reversed the health problem and etiology. Impaired Skin Integrity related to prolonged immobility is the correct format. The nursing diagnosis does address a client response rather than need: impaired skin integrity as a response to prolonged immobility. The nursing diagnosis does include defining characteristics: open area on the buttocks, wound surface clean and beefy red, no drainage or foul odor. The nursing diagnosis does not refer to environmental factors.

The care plan for a postoperative client includes a nursing diagnosis of "Risk for Urinary Retention." The nurse determines that the client has been voiding adequately. What is the nurse's most appropriate action? Continue to observe for urinary retention because of the client's postoperative status. Revise the nursing diagnosis because the client's status has changed. Initiate a collaborative problem to address the client's changing status. Consult with the physician about the revision of the nursing diagnosis.

Revise the nursing diagnosis because the client's status has changed.

A client admitted for a surgical procedure tells the nurse, "I am very worried because I am allergic to latex. I want to make sure that everyone knows this." To ensure the safety of the client, which nursing diagnosis should the nurse assign to this client and address in the care plan? Anxiety related to surgical procedure Knowledge Deficit related to surgical procedure Risk for Allergy Response related to latex allergy Risk for Injury related to latex allergy

Risk for Allergy Response related to latex allergy To ensure the safety of the client, the nurse should address the risk for an allergic response due to the client's latex allergy. Anxiety refers to a vague feeling of dread; however, the client is responding with fear to a very real threat. There is no evidence that the client does not understand the surgical procedure. Risk for Injury is not an appropriate diagnosis, because it does not adequately address the specific health problem.

A new chemical plant is being built in the community. The nurse is concerned about the possibility of environmental pollution adversely affecting the health of the residents. What nursing diagnosis would the nurse use to address this concern? Knowledge Deficit related to effects of chemical plant pollution Deficient Community Health related to chemical plant Risk for Community Contamination related to possible environmental pollution Risk for Infection related to community contamination

Risk for Community Contamination related to possible environmental pollution

A new chemical plant is being built in the community. The nurse is concerned about the possibility of environmental pollution adversely affecting the health of the residents. What nursing diagnosis would the nurse use to address this concern? Knowledge Deficit related to effects of chemical plant pollution Deficient Community Health related to chemical plant Risk for Community Contamination related to possible environmental pollution Risk for Infection related to community contamination

Risk for Community Contamination related to possible environmental pollution The nurse has identified a risk diagnosis because of the unknown health effects of the chemical plant on the community. Risk for Community Contamination would address the broad concerns of the nurse. Knowledge Deficit is not appropriate because it has too narrow a focus. Deficient Community Health is not a NANDA-I diagnosis and the etiology must deal with how the plant may possibly affect the community. Risk for Infection has a very narrow focus. The etiology of community contamination has not been proven.

Which is an accurately phrased risk nursing diagnosis? Risk for Impaired Coping as evidenced by client crying Risk for Fluid Volume Excess related to increased oral intake as evidenced by consuming 3 L of soda Risk for Pain After Surgery Risk for Falls related to altered mobility

Risk for Falls related to altered mobility Risk for Falls related to altered mobility is an accurately phrased risk nursing diagnosis. It is a two-part statement that contains the diagnostic statement (Risk for Falls) and risk factors (altered mobility).Two of the options (Risk for Impaired Coping and Risk for Fluid Volume Excess) incorrectly pair actual presenting manifestations, also called defining characteristics (client crying, consuming 3 L of soda), with a risk statement. Another option (Risk for Pain After Surgery) does not include a risk factor.

An older adult client's venous ulcer has become foul-smelling after the client began using strips of a sheet to dress the wound due to running out of sterile dressing supplies. How should the nurse document a nursing diagnosis statement related to this client's circumstances? Risk for Infection related to knowledge deficit Acute Confusion related to appropriate wound care Knowledge Deficit due to risk for infection Risk for sepsis related to local infection.

Risk for Infection related to knowledge deficit Risk for Infection related to knowledge deficit is the correct answer. The client's use of nonsterile items to dress a wound clearly indicates a lack of knowledge. Acute confusion describes a change in cognition, not an inappropriate action. A risk for infection does not cause a knowledge deficit. Indeed sepsis can result from an infection, but infection is a medical diagnosis, not a nursing diagnosis.

Which nursing diagnosis has the priority when caring for an older adult client with Alzheimer disease? Impaired Physical Mobility Risk for Injury Self-Care Deficit Impaired Memory

Risk for Injury Clients with Alzheimer disease are highly prone to injuries. Risk of Injury may also be precipitated by the altered memory. Mortality and morbidity resulting from injury is highest in older age groups. Consequently, it is very important for the nurse to provide a safe and secure environment. Impaired Physical Mobility, Self-Care Deficit, and Impaired Memory are also present but are not the highest priority.

A nurse is educating a client about care to be taken in nephrotic syndrome. The client expresses that the education is of no use because the disease is not curable. What nursing diagnosis should the nurse formulate with regard to the client's concern? Risk for Powerlessness Disturbed Body Image Impaired Comfort Risk for Suicide

Risk for Powerlessness

The nurse is caring for a 14-year-old client who has just delivered a baby. The client reports living with an aunt and having no other family around. The delivery was uncomplicated and the newborn is healthy. Which would be the primary nursing diagnosis for this client?

Risk for impaired parenting

A healthcare provider in a different state is reviewing the x-rays and scans of a patient who lives in another state. The patient's primary care provider has asked the other healthcare provider for their interpretation of the tests and to obtain the provider's opinion about the diagnosis. Which technology is being used? Telehealth Telemedicine Patient portal Diagnostic application

Telehealth

A nurse has a two-way video communication with the specialist involved in the care of a client in a long-term care facility. This is an example of what nursing informatics technology? Client engagement technology Data aggregation technology Telemedicine and mobile technology Population health management technology

Telemedicine and mobile technology

An informatics nurse specialist is working with a team designing an update to a clinical information system being used by the nursing staff. When selecting the language to be used with the system, which characteristic would be most appropriate to address? Select all that apply. Terminology is clear and concise for the nurses Terminology reflects the technological aspects of a system Terminology is familiar to the user Terminology reflects the users work context Terminology is ambiguous to the user

Terminology is clear and concise for the nurses Terminology is familiar to the user Terminology reflects the users work context

A client has been admitted to a hospital due to an acute psychotic episode. Which assessment data would the nurse identify as this client's strengths? (Select all that apply.) The client has been living on the street for 3 weeks. The client is male and 35 years old. The client has ample financial resources. The client refuses to take the ordered medication. The client is willing to attend counseling sessions.

The client has ample financial resources. The client is willing to attend counseling sessions.

The nurse is formulating nursing diagnoses pertaining to a client with pancreatic cancer. Which factors should the nurse identify as strengths of the client? Select all that apply. The client states that no one should ever ask for help from others. The client has been accompanied by family members to every appointment. The client states a belief in a reward in heaven after death. The client has a long history of health problems. The client has demonstrated effective coping skills in the past.

The client has been accompanied by family members to every appointment. The client states a belief in a reward in heaven after death. The client has demonstrated effective coping skills in the past. The client's support by family members, a belief in an afterlife, and demonstration of effective coping skills in the past are indications that the client will be able to cope with this illness. The client's belief in never asking for help will cause excessive isolation from others. The client's long history of health problems may have exhausted the client's physical and mental resources.

A nursing diagnosis of "Ineffective Coping" has been chosen for a client after receiving a diagnosis of prostate cancer. What assessments would the nurse consider as evidence for this diagnosis? (Select all that apply.) The client reports an inability to get adequate restful sleep. The client has difficulty concentrating on the details of treatment options. The client states, "I can't handle all of this." The client asks for information relating to the cancer diagnosis. The client requests the minister of the client's church to visit.

The client has difficulty concentrating on the details of treatment options. The client reports an inability to get adequate restful sleep. The client states, "I can't handle all of this."

The nurse is checking the client's temperature. The client feels warm to touch. However, the client's temperature is 98.8°F (37.1°C). Which statement could explain this?

The client is covered with a couple of thick blankets.

A client diagnosed with advanced lung cancer has a nursing diagnosis of Ineffective Coping. What assessment data would provide evidence to the nurse for this diagnosis? The client asks about hospice services. The client makes funeral plans. The client states, "I am sure the doctors have misdiagnosed me." The client states, "I hope that I am able to attend my daughter's wedding."

The client states, "I am sure the doctors have misdiagnosed me." Denying the illness by stating a belief that the cancer diagnosis is incorrect is evidence that the client is not dealing with the illness. Inquiring about hospice and making funeral plans shows acceptance of the advanced stage of the illness. Stating a hope to attend the daughter's wedding is expressing hope for the future and is evidence of effective coping.

The nurse has identified a nursing diagnosis of "Risk for Impaired Parenting" for a client who has recently learned of being pregnant. What assessment data would be appropriate to lead the nurse to select this diagnosis? The client states, "I am shocked to find out that I am pregnant." The client states, "I do not plan to tell my family about my pregnancy right away." The client states, "I do not know how to take care of a baby." The client states, "I know that I will have to make some changes in my life."

The client states, "I do not know how to take care of a baby." It is not unusual to feel unprepared to care for a baby. However, this warrants the nurse's attention because there is an associated risk of impaired parenting. Being shocked about the pregnancy and being aware of the need to make changes in one's life are normal reactions to finding out about a pregnancy and do not necessarily indicate future problems. The nurse must work with the client about communication with family, but this does not necessarily mean that the client's parenting will be compromised.

The nurse has been providing care to a client during a divorce. The client is now divorced from the spouse, effective 2 weeks ago. The nurse identified a nursing diagnosis of "Readiness for Enhanced Coping." What statement by the client would support this nursing diagnosis? The client states, "I can't believe that he divorced me after 20 years of marriage." The client states, "I feel like I can finally get along with my life now that the divorce is final." The client states, "I am afraid that I have been drinking too much just to fill my time." The client states, "Now that I am divorced, I will have to work two jobs just to pay my bills."

The client states, "I feel like I can finally get along with my life now that the divorce is final." The client's statement of being able to continue with life now that the divorce is final indicates that the client views the finality of the divorce as a relief. Because the client is eager to move on, the nurse would decide that "Readiness for Enhanced Coping" would be appropriate in this case. The client's statement of disbelief, alcohol use, and financial difficulties indicate difficulty in coping with the divorce and would not provide evidence or support for this diagnosis.

20. The nurse is developing goals for a client who has been admitted for an acute myocardial infarction. What goal written by the nurse requires revision?

The client will understand the effects of smoking related to heart disease.

A nurse will assess the oral temperature of a postoperative client. Prior to performing this assessment, which should the nurse identify?

The client's most recent temperature

When administering beta blocker medications, the physician adds an order to hold medication when the client is bradycardic. Which statement explains this order?

The client's pulse rate is below 60 beats per minute.

The nurse caring for a client diagnosed with melanoma has identified a nursing diagnosis of "Ineffective Coping." What subjective assessment data would provide evidence for this nursing diagnosis? The client's report of eating more fruits and vegetables The client's report of reading the Bible and praying daily The client's report of researching treatment options for melanoma The client's report of increased consumption of alcohol

The client's report of increased consumption of alcohol The client's increased consumption of alcohol is an unhealthy coping mechanism. The client's other statements indicate healthy ways of dealing with the illness.

A nurse is assessing a client's blood pressure manually. The nurse should identify the client's systolic blood pressure (SBP) when which event occurs?

The first faint, but clear, sound appears.

The nursing instructor is teaching students about assessment and the importance of having baseline data when caring for clients. The instructor should inform the students that the best place to get baseline data is:

The initial comprehensive client assessment.

Which nursing actions would take place during the diagnosis stage of the nursing process? Select all that apply. The nurse assists the client with ambulation to the bathroom. The nurse asks, "Based on what you have told me, it seems that urinary incontinence is a problem for you. What do you think?" The nurse identifies that the client has effectively coped with health stressors in the past. The nurse identifies that the client who is on strict bed rest is at risk for impaired skin integrity. The nurse determines that the client needs to have a decrease in activity.

The nurse identifies that the client has effectively coped with health stressors in the past. The nurse determines that the client needs to have a decrease in activity. The nurse asks, "Based on what you have told me, it seems that urinary incontinence is a problem for you. What do you think?" The nurse identifies that the client who is on strict bed rest is at risk for impaired skin integrity.

Which strategy could be implemented by the nurse in ensuring the protection of electronic data at health care agencies? The nurses are expected to change their access number and password less frequently. The nurse is being asked to remove the screen saver for data that have been displayed for prolong periods. The nurse gives unlimited access to the multidisciplinary team so that personnel from various departments can retrieve data. The nurse locks out client information, except to those who have been authorized through appropriate security measures.

The nurse locks out client information, except to those who have been authorized through appropriate security measures.

38. A client in the intensive care unit with a nursing diagnosis of Risk for Impaired Skin Integrity has a nursing intervention that states the client is to be turned and repositioned every 2 hours. As the nurse is turning the client to the client's left side, the nurse notices that the client has a non blanching, reddened area over the right trochanter. What would be the most appropriate action for the nurse to take?

The nurse repositions the client to the client's left side and updates the plan of care to turn and reposition the client every hour.

When reviewing the client's history, the nurse notes that the client's last documented bowel movement was 2 days ago. Before the nurse identifies a diagnosis of "Constipation," what assessment must the nurse make? The nurse should assess the client's dietary habits. The nurse should assess the client's bowel sounds. The nurse should determine the client's normal bowel elimination pattern. The nurse should determine the standard bowel elimination pattern for the client's age.

The nurse should determine the client's normal bowel elimination pattern. To validate the diagnosis, the nurse must determine what is normal for the client. Dietary habits may contribute to constipation, but the nurse must first confirm that the client is actually constipated. Likewise, bowel sounds might help explain the cause of constipation, but the nurse should first confirm that the client is constipated. There is no standard elimination pattern; it is highly individualized.

During morning report, the night nurse tells the day nurse that the client refused to allow the technician to draw blood for laboratory testing. What step would be essential for the day nurse to complete before selecting a nursing diagnosis to address this issue? The nurse should determine the length of time the client has been in the hospital. The nurse should determine what laboratory tests are critical at this time. The nurse should determine the reason for the client's refusal. The nurse should determine the client's last laboratory results.

The nurse should determine the reason for the client's refusal. Before addressing the issue, the nurse must determine why the client refused the lab draw. It is essential to know the cause before planning how to address the issue. It is immaterial how long the client has been in the hospital, what laboratory tests are critical, or what the client's last results were.

During a home health care visit, the nurse identifies a nursing diagnosis of Caregiver Role Strain for a parent who is caring for a child dependent on a ventilator. What subjective assessment data would support the nurse's diagnosis? The parent states, "I make sure that I get regular exercise." The parent states, "A member of my church gives me a break twice a week." The parent states, "I cannot allow anyone else to help because they won't do it right." The parent states, "I attend support group meetings when I am able to go."

The parent states, "I cannot allow anyone else to help because they won't do it right." The parent's statement of not allowing anyone to help because "they won't do it right" supports the nursing diagnosis of Caregiver Role Strain. The parent's statement indicates an inability to allow help, which will cause mental and physical strain. The other statements indicate a healthy ability to use coping mechanisms to deal with this difficult situation.

The nurse is developing and documenting a nursing diagnosis for a client. When formulating the nursing diagnosis, what guidelines should the nurse follow? Use accepted terms for the specific facility. Use only terms accepted by NANDA-I to state diagnoses and develop a nursing care plan. Do not use a computerized program to develop nursing diagnoses. Use NANDA-I nursing diagnoses to communicate the purposes of research and client care.

Use accepted terms for the specific facility.

A facility is planning to implement an update to the electronic health record. This update is designed to facilitate documentation of critical changes in clients' condition to save time that nurses spend when documenting these changes. An informatics nurse specialist asks a group of staff nurses to test this update to determine if it is working as it was designed. The nurses are involved in which type of testing? Performance User acceptance Integration Function

User acceptance

The nurse is trying to obtain a temperature and the client continues to bite down on the oral thermometer. The nurse determines a rectal thermometer should be used. What actions demonstrates the nurse's understanding of the client's well-being and safety during this procedure?

Using a digital thermometer, the nurse inserts the covered, lubricated probe 1.5 in (3.75 cm) into the rectum for 1 minute.

The nurse is performing a physical assessment on a newly admitted client. During the assessment, the nurse notices the client grimacing and holding the abdomen. When the nurse asks the client whether the client is in pain, the client answers, "No." What is the best thing for the nurse to do next?

Validate the data.

The nurse caring for a morbidly obese client formulates the possible nursing diagnosis, "Imbalanced Nutrition: More than Body Requirements related to excessive food intake as evidenced by morbid obesity." In order to assure the accuracy of the diagnosis, which further step must the nurse take? Validate with the client that excessive food intake is the cause of the client's obesity. Determine what weight loss programs the client has utilized in the past. Research the client's medical history to determine the client's usual weight. Interview the client to assess the client's motivation to lose weight.

Validate with the client that excessive food intake is the cause of the client's obesity.

The nurse is assessing a client who was just admitted to the unit following an abdominal hysterectomy. On which assessment finding would the nurse base the priority diagnosis? Dressing intact with slight bloody discharge present Client reports being very sleepy Abdominal area soft with diminished bowel sounds throughout Diminished breath sounds in left lower lobe Skin warm and dry

Validate with the client that excessive food intake is the cause of the client's obesity. The nurse must discuss the diagnosis with the client to ascertain whether or not the diagnosis is correct. There are other causes of obesity, such as a decrease in activity secondary to surgery. In order to plan effective interventions, it is important to determine the correct etiology. Determining the weight loss programs used by the client and the client's motivation to lose weight are important in planning interventions once the cause is determined. The client's usual weight is not relevant; the obesity may be longstanding.

An informatics nurse specialist is engaged in evaluating a clinical information system. At which time would the nurse specialist anticipate the need to obtain help from a statistician? During the literature search When determining the data collection method When preparing how to display the data Just before documenting the outcome evaluation

When determining the data collection method

42. The nurse has established client outcomes and outcome criteria. What should the nurse do next?

Write a client plan of care

36. The nurse reviews an interdisciplinary plan of care to determine the day's care guidelines and outcomes for a client who had a left hip replacement. The type of plan of care the nurse is reviewing is:

a clinical pathway.

It is very important to assess for the quality of someone's respirations as well as describe what is heard with auscultation. Which describes stridor?

a harsh, inspiratory sound that may be compared to crowing

A nurse sees the client grimace and documents that the client is in pain, without interviewing the client to obtain further cues. The nurse has: an impaired cluster interpretation. a lack of cues, or premature closure. an ineffective database. an inaccurate evaluation.

a lack of cues, or premature closure. The lack of adequate cues is called premature closure, which is the case in this situation, as the nurse only has one cue. There is no "cluster" of cues to interpret, so impaired cluster interpretation would not be accurate. It is not so much that the nurse's database is ineffective as it is that the database lacks sufficient data. Evaluation is a separate phase in the nursing process and does not pertain to diagnosis.

The nurse is caring for a client who is experiencing a collaborative problem. The nurse should plan the client's care based on an understanding that this problem is characterized by: an emergent condition that requires rapid nursing response. a risk or wellness human response to health problems. a result of disease, trauma, treatment, or diagnostic studies. a convenient means for communication among team members.

a result of disease, trauma, treatment or diagnostic studies.

The nurse is collecting health data and avoids using closed-ended questions. Which are examples of closed-ended questions? Select all that apply. a) "Does it hurt when I touch you here?" b) "Do you smoke cigarettes?" c) "What plans do you have after you are discharged?" d) "Is there any chance you might be pregnant?" e) "What sorts of things do you do for fun?" f) "Are you ready to get out of bed?"

a) "Does it hurt when I touch you here?" b) "Do you smoke cigarettes?" d) "Is there any chance you might be pregnant?" f) "Are you ready to get out of bed?"

Which statement by a nurse case manager regarding this nurse's role in client care is most accurate? a) "Even though I do not provide care to clients, my work is very important." b) "Moving away from client care is a necessary step to advancing my career." c) "I provide a critical service that is necessary for financial reimbursement." d) "I provide indirect care to my clients by coordinating their treatment with other disciplines."

a) "Even though I do not provide care to clients, my work is very important."

The nurse is attending a conference on evidence-based practice. Which statement by the nurse indicates further education is needed? a) "I must conduct research to validate the usefulness of my nursing interventions." b) "I can learn about evidence-based practice by reading professional nursing journals." c) "Nursing interventions should be supported by a sound scientific rationale." d) "The Agency for Healthcare Research and Quality is a resource for evidence-based practice."

a) "I must conduct research to validate the usefulness of my nursing interventions."

Which statement by a nurse case manager regarding this nurse's role in client care is most accurate? a) "I provide indirect care to my clients by coordinating their treatment with other disciplines." b) "Even though I do not provide care to clients, my work is very important." c) "I provide a critical service that is necessary for financial reimbursement." d) "Moving away from client care is a necessary step to advancing my career."

a) "I provide indirect care to my clients by coordinating their treatment with other disciplines."

The nurse is caring for a client who is being treated following a drug overdose. The client states, "My life is over, I cannot stop using heroin." Which statement would the nurse employ to strengthen the nurse-client relationship? a) "Perhaps we can talk about your feelings some more." b) "I understand why you feel this way, it is difficult to kick an addiction." c) "Would you like to discuss how long you have been using drugs?" d) "Have you reflected on what causes you to use heroine?"

a) "Perhaps we can talk about your feelings some more."

A client is reluctant to undergo surgery and is discussing it with the nurse. Which response by the nurse would reflect an authoritarian approach? a) "Surgery is your only option. You need this operation." b) "It's your choice about the surgery. What do you understand about the situation?" c) "If you don't have the surgery you may not live. Your family needs you." d) "Your grandchildren would be very upset if they lost their grandfather."

a) "Surgery is your only option. You need this operation."

A nurse is providing care to several assigned clients and decides to delegate the task of morning vital signs to unlicensed assistive personnel. The nurse would assume responsibility and refrain from delegating this task for which client? a) A client with a high fever receiving intravenous fluids, antibiotics, and oxygen b) An older adult with pneumonia who is being discharged to the son's home tomorrow c) An adult client who is being treated for kidney stones d) A middle-aged client who had abdominal surgery 3 days ago and is ambulating in the hall

a) A client with a high fever receiving intravenous fluids, antibiotics, and oxygen Explanation: The client with a high fever receiving intravenous fluids, antibiotics, and oxygen is the least stable of the clients listed and should be assessed by the nurse.

Which type of nursing intervention is oxygen administration and why is it considered to be so? a) A dependent nursing intervention, because oxygen is considered a drug that requires a physician's order b) A collaborative nursing intervention, because it is ordered by the respiratory therapist c) An independent nursing intervention, because nurses have the necessary skill to administer oxygen d) An interdependent intervention, because physicians, nurses, and respiratory therapists have the necessary skill to administer oxygen

a) A dependent nursing intervention, because oxygen is considered a drug that requires a physician's order

Which nursing intervention is most likely to be allowed within the parameters of a protocol or standing order? a) Administering a glycerin suppository to a constipated client who has not responded to oral stool softeners b) Changing a client's intravenous (IV) fluid from normal saline to 5% dextrose c) Administering a beta-adrenergic blocker to a new client whose blood pressure is high on admission assessment d) Changing a client's advance directive after the prognosis has significantly worsened

a) Administering a glycerin suppository to a constipated client who has not responded to oral stool softeners

Which is a characteristic of a person-centered or helping relationship? a) An unequal sharing of information b) A focus on the needs of the helping person c) The accountability of the person being helped for the outcomes of the relationship d) Spontaneous occurrence with random individuals

a) An unequal sharing of information

The nurse is caring for a client who is a victim of sexual assault. Which action would the nurse take to develop a trusting rapport with the client? a) Approach the client with empathy and understanding and allow the client to share feelings without being judged. b) Exhibit a professional demeanor while examining the client and obtaining specimens, asking questions that are not intrusive. c) Use strategic pauses to allow the client to provide information that will be used to help officials in their investigation. d) Practice active listening by allowing the client to express fears and concerns then restating in the nurse's own words to demonstrate understanding.

a) Approach the client with empathy and understanding and allow the client to share feelings without being judged.

The nurse is assessing the glossopharyngeal nerve on a client diagnosed with a cerebrovascular accident. Which action should the nurse take? a) Ask client to move tongue side to side b) Lightly touch around the jawline using a piece of cotton wool then c) a blunt pin d) Ask the client to open the mouth while applying resistance Observe the uvula while the client say "Ahhhh"

a) Ask client to move tongue side to side

A 55-year-old female client was admitted to the medical unit 2 days ago with liver failure secondary to alcohol use. She's on bed rest with bathroom privileges and has just been up to use the toilet. While helping the client to stand so she can wipe herself, the nurse notices a few drops of blood on top of the semiliquid, clay-colored stool in the toilet. What action should the nurse take next? a) Ask the client if she has noted any blood in her stools lately. b) Nothing. The nurse shouldn't alarm her unnecessarily. c) Ask the client if her gums bled this morning when she brushed her teeth. d) Ask the client if she feels dizzy.

a) Ask the client if she has noted any blood in her stools lately.

The nurse is preparing to do a focused assessment of the abdomen on a client following an abdominal hysterectomy. Which intervention is most important for the nurse to do prior to the physical assessment? a) Ask the client to empty her bladder. b) Warm the equipment. c) Place the client in a semi-Fowler's position. d) Measure height and weight.

a) Ask the client to empty her bladder.

A 44-year-old male client arrives unconscious to the emergency department with a head injury sustained in a fall from a 6-ft (2-m) ladder. Which action by the nurse is the most important to take? a) Assess pupil shape and reactivity to light. b) Assess the client's orientation to person, place, and time. c) Assess the client's arterial blood gases level. d) Assess blood pressure and apical heart rate.

a) Assess pupil shape and reactivity to light.

One hour after receiving pain medication, a postoperative client reports intense pain. What is the nurse's appropriate first action? a) Assess the client to determine the cause of the pain. b) Consult with the physician for additional pain medication. c) Discuss the frequency of pain medication administration with the client. d) Assist the client to reposition and splint the incision.

a) Assess the client to determine the cause of the pain.

The nurse has assisted the client to ambulate for the first time. After returning the client to bed, what is the nurse's priority intervention? a) Assess the client's response to the ambulation. b) Inform the client when ambulation is scheduled next. c) Discuss the client's feelings about the illness. d) Document the client's ambulation.

a) Assess the client's response to the ambulation.

Which nursing action can be categorized as a surveillance or monitoring intervention? a) Auscultating of bilateral lung sounds b) Providing hygiene c) Administering a paracetamol tablet d) Use of the therapeutic communication skills

a) Auscultating of bilateral lung sounds

A nurse is completing a health history with a newly admitted client. During the interview, the client presents with an angry affect and states, "If my doctor did a good job, I would not be here right now!" What is the nurse's best response? a) Be silent and allow the client to continue speaking when ready. b) Smile and say, "Don't worry, I am sure the physician is doing a good job." c) Stand and say, "I can see this interview is making you uncomfortable, so we can continue later." d) Nod and say, "I agree. If I were you, I would get a new doctor."

a) Be silent and allow the client to continue speaking when ready.

The nurse is caring for an 88-year-old male admitted 2 days ago for dehydration. The nurse brings the client his breakfast tray and notes that the client appears to be having difficulty understanding what she is saying to him today. Which nursing action is most appropriate? a) Check the client's ear canals for cerumen. b) Ask the client if he left his earplugs in his ears. c) Use facial expressions and sign language to communicate. d) Speak to the older adult client in a high-frequency tone of voice.

a) Check the client's ear canals for cerumen.

The nurse is asking admission interview questions and the client has explained the reason for seeking care. What is the most appropriate way to document the response? a) Client states, "I feel winded all of the time and yesterday I started spitting up a lot of phlegm." b) Client reports breathlessness and productive cough. c) Client reports respiratory distress and frequent spitting. d) Client describes shortness of breath and increased sputum production.

a) Client states, "I feel winded all of the time and yesterday I started spitting up a lot of phlegm."

During an assessment of a newly admitted client the nurse asks the client many questions. The nurse begins the assessment by asking, "How many times have you been hospitalized this year for your back pain?" This is an example of which type of question? a) Closed question b) Sequencing question c) Open-ended question d) Reflective question

a) Closed question

half of what the doctor ordered." How would the nurse most effectively meet this client's need? a) Collaborate with other disciplines to determine the best way to meet the client's medication requirements. b) Reinforce to the client and family the necessity of taking all medication as ordered to stabilize the client's condition. c) Inform the physician of the need to prescribe a less expensive medication for the client's condition. d) Instruct the client that some pharmaceutical companies have programs to help with medication expenses.

a) Collaborate with other disciplines to determine the best way to meet the client's medication requirements.

Discharge plans for a client with a mental health disorder include living with family members. The nurse learns that the family is no longer willing to allow the client to live with them. What is the nurse's most appropriate action? a) Collaborate with other disciplines to revise the discharge plans. b) Instruct the client to make alternate living arrangements. c) Communicate with the physician about additional orders. d) Inform the family that it is not possible to change the discharge plans.

a) Collaborate with other disciplines to revise the discharge plans.

The nurse is caring for a vegetarian who has iron deficiency anemia. The standardized nutritional plan for a client with anemia calls for the client to increase consumption of animal protein. How should the nurse plan to meet this client's nutritional needs? a) Collaborate with the nutritionist to modify the nutritional plan. b) Instruct the client that consumption of animal protein is necessary to cure the anemia. c) Meet with the client's family to emphasize the importance of nutritional modification. d) Arrange for animal protein to be disguised in the client's meal.

a) Collaborate with the nutritionist to modify the nutritional plan.

A new client is admitted to the hospital and requires a comprehensive admission assessment. What should the nurse include in this assessment? Select all that apply. A new client is admitted to the hospital and requires a comprehensive admission assessment. What should the nurse include in this assessment? Select all that apply. a) Complete set of vital signs b) Functional ability evaluation c) Goals with outcome criteria d) Description of client education e) Collection of subjective data

a) Complete set of vital signs b) Functional ability evaluation e) Collection of subjective data

The nurse assigned to care for a client who has received a sedative has asked the unlicensed assistive personnel (UAP) to help the client to the toilet. The nurse demonstrates proper delegation skills by performing which actions? Select all that apply. a) Confirming that the UAP has repeatedly completed similar tasks b) Giving a report on the client to the UAP and answering questions c) Being available for questions from the UAP d) Transferring accountability and responsibility for the client to the UAP e) Confirming that the UAP has successfully passed this skill competency

a) Confirming that the UAP has repeatedly completed similar tasks b) Giving a report on the client to the UAP and answering questions c) Being available for questions from the UAP e)Confirming that the UAP has successfully passed this skill competency

The nurse is providing care for a male client age 69 years who has been admitted to the hospital for the treatment of pneumonia. Auscultation of the client's lungs reveals the presence of discontinuous, popping sounds during inspiration over the lower lung fields. What should the nurse document as being present? a) Crackles b) Sonorous wheeze c) Sibilant wheeze d) A friction rub

a) Crackles

Cranial nerve function is important for normal sensory functioning. Which cranial nerve is important for the sense of smell? a) Cranial nerve I b) Cranial nerve III c) Cranial nerve IV d) Cranial nerve II

a) Cranial nerve I

A grating feel and noise with joint movement, particularly in the temporomandibular joint, is called what? a) Crepitus b) Arthritis c) Fremitus d) Inflammation

a) Crepitus

The nurse must give instructions before discharge to a 13-year-old in a sickle cell crisis. Three of the client's friends from school are visiting. In order to assure effective instruction, what should the nurse plan to do? a) Delay the instruction until the visitors leave. b) Give the visitors instructions to leave in 10 minutes. c) Ask the client if the client has any questions. d) Leave written information for the client to read later.

a) Delay the instruction until the visitors leave.

The nurse is auscultating an apical pulse on a 39-year-old client admitted with pneumonia. In counting the apical pulse, the nurse recognizes which characteristic about heart sounds? a) Each lub-dub is one beat. b) The lub-dub sounds occur within 2 seconds of each other. c) Each lub-dub is two beats. d) Heart sounds are caused by the opening of heart valves.

a) Each lub-dub is one beat.

The nurse in a burn intensive care unit (BICU) is caring for a 3-year-old child who was burned with scalding hot water. The client has burns covering 75% of the body. The client's condition is critical but stable. At 1000, the nurse reassesses the client and finds that the client is agitated and pulling at the endotracheal tube. Which is the nurse's priority intervention for this client at this time? a) Ensuring that the endotracheal tube is secure b) Changing the dressing to prevent infection c) Providing medication for agitation d) Repositioning to prevent pressure injuries

a) Ensuring that the endotracheal tube is secure

A client is admitted to the emergency department. He is bleeding from a cut on his head and his skin color is pale, with diaphoresis. What nursing action should be performed first? a) Evaluate the blood pressure and pulse b) Provide a warm, quiet, dimly lit room c) Assess the cause of the client's wound d) Interview to obtain the health history

a) Evaluate the blood pressure and pulse

Priority setting is based on the information obtained during reassessment and is used to rank nursing diagnoses. Each factor contributes to priority setting except which? a) Finances of the client b) The client's condition c) Time and resources d) Feedback from the family

a) Finances of the client

The nurse testing a client's eyes asks the client to focus on a finger from 60 cm away and moves the client's eyes through the six cardinal positions of gaze. Using this procedure, which cranial nerves is this nurse testing? Select all that apply. a) IV: Trochlear b) VII: Facial c) VI: Abducens d) V: Trigeminal e) III: Oculomotor f) II: Optic

a) IV: Trochlear c) VI: Abducens e) III: Oculomotor

A nurse is caring for a 44-year-old female who had a left total hip arthroplasty 3 days ago. Her postoperative course has been uneventful except for a urinary tract infection that developed yesterday for which she is receiving cefaclor 500 mg PO bid. The client tells the nurse that the backs of her legs and buttocks are "itching like crazy." Which action should the nurse take first? a) Inspect the area of itchy skin. b) Check her chart for allergy information. c) Review her medication record. d) Review her medical history.

a) Inspect the area of itchy skin.

The nurse and the physical therapist discuss the therapy schedule and goals for a client on a rehabilitation unit. What type of communication is occurring between the nurse and the therapist? a) Interpersonal b) Small-group c) Intrapersonal d) Organizational

a) Interpersonal

A nurse is teaching a client about the importance of checking the skin for changes that might suggest skin cancer. After describing the typical lesions associated with melanoma, the nurse determines that the teaching was successful when the client identifies which characteristic? Select all that apply. a) Irregular edges b) Single color c) Larger than 1/4 inch in diameter d) Symmetrical shape e) Change in the mole

a) Irregular edges c) Larger than 1/4 inch in diameter e) Change in the mole

A nurse auscultates the right carotid artery in an older adult client and identifies a bruit. What does this assessment finding mean? a) It is distended. b) It is normal. c) It is inflamed. d) It is dissecting.

a) It is distended.

A nurse assesses breath sounds for clients presenting at a local clinic with difficulty breathing. Which sounds would the nurse document as normal? Select all that apply. a) Medium-pitched, medium-intensity blowing sounds, auscultated over the first and second interspaces anteriorly and the scapula posteriorly b) Soft, low-pitched, whispering sounds heard over most of the lung fields c) Musical or squeaking sounds, or high-pitched continuous sounds auscultated during inspiration and expiration d) Sonorous or coarse sounds with a snoring quality auscultated during inspiration and expiration e) Blowing, hollow sounds auscultated over the larynx and trachea f) Bubbling, crackling, or popping sounds auscultated during inspiration and expiration

a) Medium-pitched, medium-intensity blowing sounds, auscultated over the first and second interspaces anteriorly and the scapula posteriorly b) Soft, low-pitched, whispering sounds heard over most of the lung fields e) Blowing, hollow sounds auscultated over the larynx and trachea

A client reports severe abdominal pain that started about an hour after eating lunch. Assessment reveals absent bowel sounds and rebound tenderness in the right lower quadrant. What does the nurse suspect these findings may indicate? Select all that apply. a) Peritonitis b) Salpingitis c) Duodenal ulcer d) Early bowel obstruction e) Paralytic ileus f) Food poisoning

a) Peritonitis e) Paralytic ileus

After learning about a client's limited financial resources and limited insurance benefits, the home care nurse modifies nursing interventions related to a client's care instructions. The nurse modifies the plan of care based upon which client variable? a) Psychosocial background b) Developmental stage c) Research findings d) Current standards of care

a) Psychosocial background

An indwelling urinary catheter has been ordered for a client experiencing urinary retention after surgery. When the nurse enters the room to place the catheter, the client reports voiding in the bathroom. Which is the nurse's most appropriate action? a) Reassess whether the client still needs the urinary catheter. b) Inform the client that the catheter will no longer be necessary. c) Insert the urinary catheter as ordered to relieve the urinary retention. d) Instruct the client that the catheter is essential to check for urinary retention.

a) Reassess whether the client still needs the urinary catheter.

An indwelling urinary catheter has been ordered for a client experiencing urinary retention after surgery. When the nurse enters the room to place the catheter, the client reports voiding in the bathroom. Which is the nurse's most appropriate action? a) Reassess whether the client still needs the urinary catheter. b) Inform the client that the catheter will no longer be necessary. c) Insert the urinary catheter as ordered to relieve the urinary retention. d) Instruct the client that the catheter is essential to check for urinary retention.

a) Reassess whether the client still needs the urinary catheter.

The nurse is performing an initial admission assessment from a client. What subjective data gathered from the client will the nurse document? Select all that apply. a) Reports of abdominal pain of 4 on a 0 to 10 point scale b) Client informs the nurse there is a floater in the left eye c) Hypoactive bowel sounds in all four quadrants d) The client states, "I feel nauseated." e) Peripheral pulses +3 f) Skin warm and dry

a) Reports of abdominal pain of 4 on a 0 to 10 point scale b) Client informs the nurse there is a floater in the left eye d) The client states, "I feel nauseated."

The nurse is preparing to give the client a bath early in the morning. The client states, "I prefer to take my bath at night. It helps me sleep." What is the nurse's most appropriate action? a) Reschedule the client's bath to the evening shift. b) Ask the client for permission to give the bath in the morning. c) Determine whether the nurses have time to give the client's bath at night.

a) Reschedule the client's bath to the evening shift.

The nurse is preparing to give the client a bath early in the morning. The client states, "I prefer to take my bath at night. It helps me sleep." What is the nurse's most appropriate action? a) Reschedule the client's bath to the evening shift. b) Ask the client for permission to give the bath in the morning. c) Tell the client that the physician has ordered sleep medication if necessary. d) Determine whether the nurses have time to give the client's bath at night.

a) Reschedule the client's bath to the evening shift.

Nursing interventions for the client after prostate surgery include assisting the client to ambulate to the bathroom. The nurse concludes that the client no longer requires assistance. What is the nurse's best action? a) Revise the care plan to allow the client to ambulate to the bathroom independently. b) Continue assisting the client to the bathroom to ensure the client's safety. c) Consult with the physical therapist to determine the client's ability. d) Instruct the client's family to assist the client to ambulate to the bathroom.

a) Revise the care plan to allow the client to ambulate to the bathroom independently.

Which components are included in the integumentary system? Select all that apply. a) Skin b) Sweat glands c) Arteries d) Muscles e) Nails f) Hair

a) Skin b) Sweat glands e) Nails f) Hair

The nurse is using a bed scale to weigh a client, and the client becomes agitated as the sling rises in the air. What would be the priority nursing intervention in this situation? a) Stop lifting the client and reassure him. b) Reassure the client that the procedure will only take a few minutes. c) Enlist the help of another nurse to hold the client steady during the procedure. d) Administer a sedative to the client and try again when the sedative takes effect.

a) Stop lifting the client and reassure him.

Which respiratory sound indicates an upper airway obstruction? a) Stridor b) Dyspnea c) Fremitus d) Wheeze

a) Stridor

While auscultating a client's lung sounds, the nurse notes crackles in the left lower lobe, which were not present at the start of the shift. The nurse is engaged in which type of nursing intervention? a) Surveillance b) Maintenance c) Educational d) Psychomotor

a) Surveillance

The nurse pinches the skin under the clavicle and it tents. What conclusion should the nurse determine from this assessment? a) The client is dehydrated. b) The client is overhydrated. c) The skin has normal turgor. d) The skin is less elastic with aging.

a) The client is dehydrated.

The registered nurse is working with an unlicensed assistive personnel. Which client should the nurse not delegate to the unlicensed assistive personnel? a) The client with continuous pulse oximetry who requires pharyngeal suctioning. b) The client who requires assistance dressing in preparation for discharge. c) The client who needs vital signs taken following infusion of packed red blood cells. d) The client who is pleasantly confused and requires assistance to the bathroom.

a) The client with continuous pulse oximetry who requires pharyngeal suctioning.

The client is having difficulty breathing. The respiratory rate is 44 and the oxygen saturation is 89% (0.89 L). The nurse raises the head of the bed and applies oxygen at 3 L/min per nasal cannula. How does the nurse determine the effectiveness of the interventions? Select all that apply. a) The client's oxygen saturation level increases. b) The client's respiratory rate decreases. c) The client's family asks if the client is going to be okay. d) The client is watching television. e) The client states, "I can breathe easier now."

a) The client's oxygen saturation level increases. b) The client's respiratory rate decreases. e) The client states, "I can breathe easier now."

A client recently was diagnosed with Bell's palsy and is back to the clinic for a follow-up visit. What would the nurse observe during the assessment of cranial nerve VII if the client's symptoms are resolving? a) The movement and appearance would appear symmetrical as the client smiles, frowns, and raises the eyebrows. b) The client is able to turn the head to the side and shrug the shoulders against resistance. c) The palate and pharynx move as the client says "ah." d) The client's tongue remains midline when it protrudes from the mouth.

a) The movement and appearance would appear symmetrical as the client smiles, frowns, and raises the eyebrows.

Which actions are examples of nursing actions listed in the ANA's Nursing: Scope and Standards of Practice for Standard 5: Implementation? Select all that apply. a) The nurse utilizes community resources and systems to implement the plan. b) The nurse incorporates new knowledge to initiate changes in nursing practice if the desired outcomes are not achieved. c) The nurse documents implementation and any modifications, including changes or omissions, of the identified plan. d) The nurse utilizes evidence-based interventions and treatments specific to the diagnosis or problem. e) The nurse demonstrates quality by documenting the application of the nursing process in a responsible, accountable, and ethical manner. f) The nurse develops expected outcomes that provide direction for the continuity of care.

a) The nurse utilizes community resources and systems to implement the plan. d) The nurse utilizes evidence-based interventions and treatments specific to the diagnosis or problem. e) The nurse documents implementation and any modifications, including changes or omissions, of the identified plan.

To provide effective nursing care, the nurse should engage in what type of communication with the client and significant others? a) Therapeutic communication b) Metacommunication c) Intrapersonal communication d) Purposive communication

a) Therapeutic communication

For which purposes would observing silence be appropriate? Select all that apply. a) To allow the client time to formulate an answer after asking the client a question b) To allow the nurse time to think of something to say when the nurse doesn't know the answer to a question c) To allow the client time to reflect on the client's thoughts d) To allow the client time to reflect on communication that has occurred e) To allow the client time to compose oneself when the client is upset

a) To allow the client time to formulate an answer after asking the client a question c) To allow the client time to reflect on the client's thoughts d) To allow the client time to reflect on communication that has occurred e) To allow the client time to compose oneself when the client is upset

A nurse during orientation notices that the preceptor gives all subcutaneous injections on a 45-degree angle. When the new nurse asks the preceptor the rationale for the practice the preceptors states, "This is how I do it, and this is how you will do it." The new nurse recognizes this behavior to be: a) aggressive. b) nurturing. c) assertive. d) passive.

a) aggressive.

A nurse is attempting to complete an admission database. While taking the history, the nurse notices the client appears uncomfortable and slightly tachypneic. The nurse should: a) allow the client to set the pace. b) tell the client to rest and allow a family member to answer. c) ask questions as quickly as possible. d) use only open-ended questions.

a) allow the client to set the pace.

An older adult client who has had a colostomy for over 10 years states, "I won't need any teaching about colostomies. I understand how to change the bag and care for my colostomy, but I'm not sure how to best clean my stoma." What does this statement indicate? a) an incongruent relationship b) a non-therapeutic relationship c) a confused relationship d) an evaluative relationship

a) an incongruent relationship

An older adult client who has had a colostomy for over 10 years states, "I won't need any teaching about colostomies. I understand how to change the bag and care for my colostomy, but I'm not sure how to best clean my stoma." What does this statement indicate? a) an incongruent relationship b) a non-therapeutic relationship c) an evaluative relationship d) a confused relationship

a) an incongruent relationship

A nurse communicating with a client states, "I will be changing your dressing, but we have plenty of time to talk first." She is already wearing sterile gloves and a mask and is busy working with her back to the client. The nurse is conveying: a) an incongruent relationship. b) a functional focus. c) a therapeutic relationship. d) a congruent relationship.

a) an incongruent relationship.

A client has sustained head trauma. The nurse uses the Full Outline of Un-responsiveness (FOUR) coma scale to determine the presence of increased intracranial pressure and client outcomes. What component(s) of the assessment will the nurse document? Select all that apply. a) eye response b) respiration c) motor response d) Shape e) brainstem reflexes f) heart rate

a) eye response b) respiration c) motor response e) brainstem reflexes

The mother of a toddler is deciding if she wants to allow her child to receive the recommended immunizations. The clinic nurse responds, "If you don't immunize your child you are jeopardizing the health of other children." What type of approach does this response indicate? a) guilt inducement or approval/disapproval b) dictatorial or bossing c) authoritarian or belittling d) advocacy or enforcing rights

a) guilt inducement or approval/disapproval

The term metacommunication is best defined as: a) interpersonal bridge between verbal and nonverbal communication. b) documenting a conversation between the client and nurse. c) contextual factors that impede communication patterns. d) congruent relationships in the spoken topics.

a) interpersonal bridge between verbal and nonverbal communication.

A male client has always prided himself in maintaining good health and is consequently shocked at his recent diagnosis of diabetes. The nurse has asked the client, "How do you think your diabetes is going to affect your lifestyle?" The nurse has utilized which of the following interviewing techniques? a) open-ended question b) validating question c) closed question d) reflective question

a) open-ended question

Which assessment measure would the nurse use to assess the location, shape, size, and density of a tumor? a) percussion b) palpation c) observation d) auscultation

a) percussion

A nurse is caring for a client who presents with a skin infection. While obtaining the client's medical history, it is determined that the client is an intravenous drug user. To foster effective communication, the nurse should: a) remain honest, open, and frank. b) consult with the social worker regarding inpatient drug rehabilitation. c) ask if the client realizes the infection is a direct result of the drug use. d) ask the client for a urine specimen for urine drug use screenings.

a) remain honest, open, and frank.

Implementation of the plan of care is most successful when: a) the nurse includes family members and other health care professionals b) the nurse takes on care and decision making for the client c) the nurse avoids further collecting of data until the evaluation phase d) the nurse recognizes documentation will occur during another phase

a) the nurse includes family members and other health care professionals

Which questions may help the nurse assess his or her ability to relate to various groups in society? Select all that apply. a. "Can I welcome this person sincerely?" b. "Can I have dinner with this person comfortably?" c. "Can I genuinely try to help this person and be comfortable enough to listen?" d. "Can I allow my son/daughter to date this person?" e. "Do I have the experience to help this person?"

a. "Can I welcome this person sincerely?" c. "Can I genuinely try to help this person and be comfortable enough to listen?" e. "Do I have the experience to help this person?"

The nurse determines that a client has not met the goal of consuming at least 80% of each meal served by a designated date. Which response(s) by the nurse would be appropriate regarding this lack of goal attainment? Select all that apply. a. "Do you think it is possible that you will be able to eat 80% of the food served here?" b. "What kinds of things have we been doing to increase your appetite?" c. "Maybe it would be better if we delete this goal and work on something else." d. "Do you think you could meet the goal if we check on it in one week or so?" e. "Are you trying as much as possible to eat 80% of each meal?"

a. "Do you think it is possible that you will be able to eat 80% of the food served here?" b. "What kinds of things have we been doing to increase your appetite?" d. "Do you think you could meet the goal if we check on it in one week or so?"

Which nursing intervention reflects culturally appropriate care when addressing a client? a. "Good morning, Mr. Smith. I am your nurse, John." b. "You can sit in this chair, Sally." c. "Thank you for coming to the clinic today." d. "I see you are here because you have a sinus infection."

a. "Good morning, Mr. Smith. I am your nurse, John."

The nursing student is discussing the need for a care plan with the instructor. What is the most appropriate explanation by the instructor for nursing care plan development? a. "The care plan is required for every client by The Joint Commission." b. "The care plan shows the medical diagnosis for the client." c. "The care plan is the only way for nurses to document what they do." d. "The care plan provides additional documentation about the work of the nurse."

a. "The care plan is required for every client by The Joint Commission."

The mother of a Black newborn asks the nurse about the bluish-black areas she noticed around the infant's lower back and buttocks. What is the nurse's best response? a. "These areas are normal and should disappear by early childhood." b. "It will be best if you have these areas treated with laser surgery." c. "These spots will normally fade in about 2 weeks." d. "This discoloration occurs in some infants and is usually permanent."

a. "These areas are normal and should disappear by early childhood."

A nurse on the unit fails to help a colleague ambulate a client even though there is time to do so. Which are appropriate responses by the nurse who required assistance with the client? Select all that apply. a. "We all have to work together as a team to provide quality care for our clients." b. "Never mind, I will get someone else to help." c. "This client is in need of our assistance, and everyone who is free should come together for improved client outcomes." d. "Please come and help and work together with me as a team." e. "If you don't assist me with client care, you may as well go home."

a. "We all have to work together as a team to provide quality care for our clients." c. "This client is in need of our assistance, and everyone who is free should come together for improved client outcomes." d. "Please come and help and work together with me as a team."

The unlicensed assistive personnel reports to the nurse that the client is refusing to eat the food on the meal tray. The nurse observes the client eating the food brought in by family members. How should the nurse respond? a. "What type of food did your family prepare for you, and does it have special meaning?" b. "You can only eat the food that we serve you." c. "Do you understand that you are on a strict diet and any variation can cause you harm?" d. "I will need to get permission from your health care provider for you to eat the food your family brought in."

a. "What type of food did your family prepare for you, and does it have special meaning?"

The nurse is documenting an assessment that was completed at 9:30 p.m. The facility uses military time for documentation. What entry should the nurse make for the time care was given? a. 2130 b. 1930 c. 930 p.m. d. 0930

a. 2130

The nurse is taking a client history. With which client is direct eye contact appropriate? a. 32-year-old white woman b. 44-year-old woman of Asian descent c. 55-year-old Native American/First Nations woman d. 60-year-old woman of Arab descent

a. 32-year-old white woman

The nurse is performing an admission assessment on a young client admitted to the unit. Which are considered objective data? Select all that apply. a.38-year-old man b.Weight: 195 lb (89 kg) c."My leg hurts." d."I am afraid something serious is wrong." e.Height: 6 ft (1.82 m)

a. 38-year-old man b. Weight: 195 lb (89 kg) e. Height: 6 ft (1.82 m)

A nurse is assessing a client admitted to the hospital with reports of difficulty urinating, bloody urine, and burning on urination. What is a priority assessment for this client? a. A focused assessment of the specific problems identified b. Obtaining a detailed assessment of the client's sexual history c. Conducting a thorough systems review to validate data on the client's record d. A full assessment of the urinary system

a. A focused assessment of the specific problems identified

A large university hospital has commissioned a multidisciplinary group to review client records following discharge to evaluate client outcomes and the character and quality of nursing care that clients receive. Which type of evaluation process will take place? a. A nursing audit b. An accreditation inspection c. A structure evaluation d. A process evaluation

a. A nursing audit

When developing a nursing plan of care and associated client outcomes, what should the nurse recognize? Select all that apply. a. A plan of care should be comprehensive and ongoing, covering and being updated during all phases of care. b. All plans of care are the same for clients with certain medical diagnoses. c. Only the client is involved in outcome setting, not the family. d. Outcomes can be short- and long-term. e. Outcome setting allows for individualization of the plan of care.

a. A plan of care should be comprehensive and ongoing, covering and being updated during all phases of care. d. Outcomes can be short- and long-term. e. Outcome setting allows for individualization of the plan of care.

Which statement is true regarding addressing a priority problem? a. A priority problem requires a nursing intervention before another problem is addressed. b. Addressing priority problems involves skipping interventions. c. The priority of problems is established and continued according to the nursing plan of care. d. Prioriy problems are identified at predetermined intervals throughout the shift.

a. A priority problem requires a nursing intervention before another problem is addressed.

What is the priority assessment for the nurse when developing a plan of care for a client from a poverty culture? a. Access to care b. Access to health insurance c. Access to affordable housing d. Access to financial assistance

a. Access to care

Which are psychomotor outcomes? Select all that apply. a. Accurately drawing up insulin b. The client will safely ambulate using a walker. c. The client will identify signs and symptoms of infection. d. The client will rate pain as a 2 on a 0 to 10 pain rating scale. e. The client will report increased confidence in testing blood glucose level.

a. Accurately drawing up insulin b. The client will safely ambulate using a walker.

The nurse is developing a plan of care for a client with a fractured femur who is in traction and will be restricted to bed for some time. Which domain should the nurse consider when developing a nursing diagnosis based on this client's musculoskeletal health problems? a. Activity and rest b. Health promotion c. Nutrition d. Self-perception

a. Activity and rest

The nurse is caring for an underweight client diagnosed with a new food allergy to wheat, rye, and oats and with a nursing diagnosis of Imbalanced Nutrition: less than body requirements. What is the most appropriate intervention for this client? a. Administer a 2,500-calorie (10,460-kJ) diet, excluding wheat, rye, and oats b. Administer a daily multivitamin c. Monitor for allergies d. Weigh client as needed

a. Administer a 2,500-calorie (10,460-kJ) diet, excluding wheat, rye, and oats

A client reports to the nurse quitting smoking 6 months ago after being diagnosed with lung cancer. The nurse recognizes this change in behavior is which type of outcome? a. Affective b. Cognitive c. Psychomotor d. Physiologic

a. Affective

A nurse is conducting an ethnographic interview with a client. Which step would the nurse do first? a. Ask an open-ended, general question b. Request clarification of a key term c. Identify clues to what may be immportant d. Document the client's view of self

a. Ask an open-ended, general question

When talking with a client, the nurse notes that the client keeps backing up. What would be the most appropriate response? a. Ask the client about personal space preferences. b. Back away from the client. c. Ask the client why he or she is backing away. d. Move closer to the client.

a. Ask the client about personal space preferences.

A client is administered an anxiolytic. Which nursing action demonstrates the nurse evaluating the client? a. Asking whether the client feels less anxious 30 min after administering the medicine b. Assigning the client a new nursing diagnosis based on the client's controlled anxiety c. Devising a plan for the client to practice anti-anxiety exercises at home d. Collecting data about the client's history with anxiety

a. Asking whether the client feels less anxious 30 min after administering the medicine

The nurse is performing an assessment on a client who reports having a rash on the back that is red and raised. What would be the most appropriate nursing action? a. Assess the client's back visually. b. Document the rash in the client's chart. c. Establish a nursing diagnosis of Altered Skin Integrity. d. Report it to the health care provider.

a. Assess the client's back visually.

When completing a transcultural assessment of communication, which assessment by the nurse is most appropriate? a. Assessment of eye contact, personal space, and social taboos b. Assessment of religious beliefs and prayer schedules c. Assessment of racial identification and cultural affiliation d. Assessment of income level to determine poverty status

a. Assessment of eye contact, personal space, and social taboos

A nurse is caring for a postoperative client 1 day after a total abdominal hysterectomy. Which nursing intervention best demonstrates caring in this situation? a. Assisting the client to sit up in a chair b. Assessing the abdominal incision c. Monitoring vital signs d. Notifying the health care provider of lab results

a. Assisting the client to sit up in a chair

Which behavior by the nurse is stereotyping? a. Avoiding older adult clients because their care is time consuming b. Openly ridiculing the practice of acupuncture c. Explaining to others that Western medicine is always superior d. Grouping care assignments to allow ample time to care for complex clients

a. Avoiding older adult clients because their care is time consuming

The nurse is preparing to evaluate the goals set for a newborn and mother. What physiologic goals will the nurse evaluate for effectiveness? Select all that apply. a. By 4/6/20, the newborn will demonstrate 2 hours of sleep prior to breastfeeding at night. b. Before discharge, the parents of the baby will verbalize decreased anxiety about taking care of a newborn. c. By 4/6/20, the parents will list appropriate resources in case questions arise after discharge. d. By 4/6/20, the mother will demonstrate a pain rating of 0 on a 0 to 10 scale. e. Before discharge, the baby with a birth weight of 7 lb, 6 oz (3.3 kg) will have reached a target weight of 8 lb (3.6 kg). f. Before discharge, the parents will demonstrate confidence in bathing and feeding their baby.

a. By 4/6/20, the newborn will demonstrate 2 hours of sleep prior to breastfeeding at night. d. By 4/6/20, the mother will demonstrate a pain rating of 0 on a 0 to 10 scale. e. Before discharge, the baby with a birth weight of 7 lb, 6 oz (3.3 kg) will have reached a target weight of 8 lb (3.6 kg).

While working as part of an interdisciplinary group developing a client's plan of care, a nurse asks the question, "Can you give me an example?" The nurse is demonstrating which standard for judging thinking? a. Clarity b. Accuracy c. Precision d. Relevance

a. Clarity

Which is the best source of information for the nurse when collecting data for an assessment? a. Client b. Primary physician c. Charge nurse d. Medical record

a. Client

Which is the best source of information for the nurse when collecting data for an assessment? a.Client b.Primary physician c.Medical record d. Charge nurse

a. Client

Which statement related to the evaluation of outcome attainment for a client is correct? a. Collecting data related to outcome attainment requires the nurse to know when to collect the data, based upon established time criteria. b. The nurse should initially evaluate the plan of care at the time of the client's discharge. c. Celebrating outcome achievement with a client often interferes with attainment of future goals. d. Evaluation of the client's attainment of outcome goals is determined by the nurse and physician.

a. Collecting data related to outcome attainment requires the nurse to know when to collect the data, based upon established time criteria.

Which activities does the nurse engage in during the evaluation phase? Select all that apply. a. Collects data to determine whether desired outcomes are met b. Assesses the effectiveness of planned strategies c. Adjusts the time frame to achieve the desired outcomes d. Involves the client and family in formulating desired outcomes e. Initiates activities to achieve the desired outcomes

a. Collects data to determine whether desired outcomes are met b. Assesses the effectiveness of planned strategies c. Adjusts the time frame to achieve the desired outcomes

A nurse providing care to a client questions judgments and considers other ways of thinking about the client's situation. Which behavior is the nurse demonstrating in the care of the client? a. Critical reflectivity b. Thoughtful practice c. Reflection in action d. Reflective skepticism

a. Critical reflectivity

The emergency department nurse is caring for a client injured in a motor vehicle collision. The client recently immigrated to the country. The nurse should implement interventions aimed at addressing which issue? a. Culture shock b. Ethnocentrism c. Generalization d. Ageism

a. Culture shock

Upon moving to another country, a college student is very confused by many local customs. He is especially bothered by the custom of men and women eating in separate areas and it makes him angry and resentful of the new culture. What are the feelings experienced by this student? a. Culture shock b. Stereotyping c. Ethnocentrism d. Cultural assimilation

a. Culture shock

A nurse is evaluating a client to determine outcome achievement. The nurse determines that the client's outcome was partially met. When documenting the evaluative statement, the nurse records which other information? a. Data that support the decision of the outcome being partially met b. The client's verbal agreement of the outcome not being met c. The reason the outcome was only partially met d. The revision to the initial outcome identified

a. Data that support the decision of the outcome being partially met

Which action exemplifies the purpose of evaluation in the nursing process? a. Decide whether to continue, modify, or terminate client care. b. Develop a prioritized list of nursing diagnoses. c. Develop an individualized plan of client care. d. Determine the client's health status, self-care ability, and need for nursing.

a. Decide whether to continue, modify, or terminate client care.

The nurse manager on an orthopedic unit has determined that the nurses are not keeping the nursing diagnoses up-to-date on client care plans and, in turn, are not using the plan of care. What is a feasible approach to correcting this problem? a. Develop a process for periodic review of care plans that focuses on deleting and updating the nursing diagnoses. b. Request that a staff development nurse instruct the nurses on concept mapping to use instead of care planning. c. Provide an in-service on interviewing and physical assessment skills; discuss the importance of these skills with the staff. d. Delegate the updating of nursing diagnoses for all clients on the unit to one nurse for each shift.

a. Develop a process for periodic review of care plans that focuses on deleting and updating the nursing diagnoses.

A client's spouse has asked that the client be cared for exclusively by female nurses. How should the nurse incorporate this request into the care plan? a. Document the request and make all reasonable efforts to honor it b. Assess the couple's rationale for making the request c. Determine whether the request is based on a valid reason or cultural preference d. Document the request as a knowledge deficit and address the couple's educational needs

a. Document the request and make all reasonable efforts to honor it

The nurse manager is holding a staff meeting and indicates that the unit is looking at a 3% budget cut for the coming year. The nurse manager asks the staff what they see as priorities for the unit, and solicits suggestions from the staff as to what budget areas might be reduced. Which standard for establishing and sustaining healthy work environments does this action represent? a. Effective decision making b. Micromanagement c. Appropriate staffing d. Meaningful recognition

a. Effective decision making

A hospital client has an aggressive fungal infection in the right eye that necessitates evisceration (removal of the eye). Consequently, the client requires twice-daily packing and dressing changes to the orbit. Which of the nurse's actions in the care of this client most clearly demonstrates interpersonal skills? a. Ensuring the client's privacy during dressing changes and providing an explanation during the procedure b. Documenting the condition of the client's orbit and the procedure of the dressing change in an accurate and timely manner c. Understanding the anatomy and physiology of the affected parts of the client's body d. Maintaining aseptic technique when performing the dressing change

a. Ensuring the client's privacy during dressing changes and providing an explanation during the procedure

Which are appropriate actions for protecting clients' identities? Select all that apply. a. Have conversations about clients in private places where they cannot be overheard. b. Document all personnel who have accessed a client's record. c. Place light boxes for examining X-rays with the client's name in private areas. d. Orient computer screens toward the public view. e. Ensure that clients' names on charts are visible to the public.

a. Have conversations about clients in private places where they cannot be overheard. b. Document all personnel who have accessed a client's record. c. Place light boxes for examining X-rays with the client's name in private areas.

The nurse is reassessing a client with leukemia who has received several packed red blood cell transfusions over the past week. Which question should the nurse ask the client to evaluate the treatment? a. Have you had any fevers? b. Have you experienced any tenderness in your joints? c. Have you noticed any bruising? d. Have you experienced any headaches?

a. Have you had any fevers?

Upon assessment of an older adult, the nurse notes the client's skin to have a yellow color. The nurse interprets this finding as a result of which health condition? a. Hepatitis b. Diverticulitis c. Cellulitis d. Appendicitis

a. Hepatitis

A nurse is evaluating the plan of care for a client in the clinic. Which actions should the nurse perform, as classic elements of evaluation? Select all that apply. a. Identifying evaluative criteria and standards b. Documenting only the facts related to the plan of care c. Collecting data to determine whether criteria and standards are being met d. Interpreting and summarizing findings e. Terminating, continuing, or modifying the plan of care

a. Identifying evaluative criteria and standards c. Collecting data to determine whether criteria and standards are being met d. Interpreting and summarizing findings e. Terminating, continuing, or modifying the plan of care

Which actions should the nurse take before making an entry in a client's record? Select all that apply. a. Identifying the form appropriate to be used for documenting b. Locating clients' files within an electronic health record system c. Choosing the charting format that the nurse prefers d. Reviewing the agency's list of approved abbreviations e. Checking that clients' names are not identified within the chart forms

a. Identifying the form appropriate to be used for documenting b. Locating clients' files within an electronic health record system d. Reviewing the agency's list of approved abbreviations

Which statement best conveys the role of intuition in nurses' problem solving? a. Intuition can be a clinically useful adjunct to logical problem solving. b. Intuition is an unreliable mode of thinking that should be avoided. c. In experienced nurses, intuition can be a valid replacement for scientific problem solving. d. Intuition is reliable when those nurses implementing it have a special "gift."

a. Intuition can be a clinically useful adjunct to logical problem solving.

Which statement regarding critical thinking in nursing is true? a. It is a systematic way of thinking. b. It shows trends and patterns in client status. c. It makes judgments based on conjecture. d. It supplies validation for reimbursement.

a. It is a systematic way of thinking.

What type of learning best takes place in the nursing laboratory? a. Kinesthetic learning b. Auditory learning c. Concrete learning d. Collaborative learning

a. Kinesthetic learning

A nurse is working in a clinic that serves a community with a high population of immigrants. Which nursing assessment is the priority? a. Language assessment b. Blood sugar assessment c. Spiritual assessment d. Blood pressure assessment

a. Language assessment

Quality improvement in care delivery requires which components? Select all that apply. a. Leadership commitment b. Continuous improvement c. Total client care by the nursing unit d. Focus on data collection e. Focus on the mission of the organization

a. Leadership commitment b. Continuous improvement d. Focus on data collection e. Focus on the mission of the organization

A nurse is assessing a client and determines that the client belongs to a minority group. Based on the nurse's understanding about minority groups, the nurse would anticipate that the client would likely experience which effects? Select all that apply. a. Less power b. Health disparities c. Greater advantages d. Improved access to care e. Increased economic privileges

a. Less power b. Health disparities

A nurse is preparing to interview a client who is newly admitted to the unit. Which strategies will help establish a quiet, relaxed, and comfortable environment during the interview? Select all that apply. a.Maintaining a proper distance from the client b. Leaving the television on c. Providing a proper seating arrangement d. Leaving the door to the room open e. Keeping the heat on high

a. Maintaining a proper distance from the client c. Providing a proper seating arrangement

A nurse is evaluating whether a client has achieved the outcomes identified in the plan of care. The nurse determines that one of the outcomes was not met. Which action would be appropriate for the nurse to try to facilitate outcome achievement? Select all that apply. a. Make the outcome more realistic for the client. b. Adjust the time frame for achievement. c. Continue to focus on the outcome as written. d. Reassess to determine whether the nursing diagnosis identified is the problem. e. Revise nursing interventions to match the client's needs.

a. Make the outcome more realistic for the client. b. Adjust the time frame for achievement. d. Reassess to determine whether the nursing diagnosis identified is the problem. e. Revise nursing interventions to match the client's needs.

A nurse is providing care to a client who is from a different culture. Which aspect about culture would be most important for the nurse to integrate into the client's care? a. Not all members of the same culture act and think alike. b. Culture is relatively static and unchanging. c. Individuals learn culture in a purposeful manner. d. Individuals can easily describe their culture.

a. Not all members of the same culture act and think alike.

Which are assessment techniques the nurse uses when performing a physical examination? Select all that apply. a. Palpation b. Percussion c. Inspection d. Auscultation e. Documentation

a. Palpation b. Percussion c. Inspection d. Auscultation

A group of nurses on the orthopedic floor of a hospital wish to improve their clinical performance. The nurse manager suggests a program in which the nurses will evaluate each other and provide feedback for improved performance. This program is termed: a. Peer review b. Quality and Safety Education for Nurses (QSEN) c. Hospital Consumer Assessment of Healthcare Providers and Systems (HCAHPS) d. American Association of Critical-Care Nurses (AACN)

a. Peer review

A nurse identifies an area where client care has been compromised. What steps should the nurse take to improve performance? Select all that apply. a. Plan a strategy using indicators. b. Assess the change. c. Discover a problem. d. Ask the client if there is a problem. e. Implement a change.

a. Plan a strategy using indicators. b. Assess the change. c. Discover a problem. e. Implement a change.

The nurse has measured from the tip of the client's nose to the earlobe and then down to the xiphoid process before inserting a nasogastric (NG) tube and attaching it to low suction. Which components of the nursing process has the nurse demonstrated? a. Planning; implementing b. Assessing; diagnosing c. Diagnosing; implementing d. Implementing; evaluation

a. Planning; implementing

Which principle should guide the nurse's documentation of entries on the client's health care record? a. Precise measurements should be used rather than approximations. b. Nurses should not refer to the names of physicians. c. Correcting fluid is used rather than erasing errors. d. Documentation does not include photographs.

a. Precise measurements should be used rather than approximations.

A nurse documents the following data in the client record according to the SOAP format: Client reports unrelieved pain; client is seen clutching the side and grimacing; client pain medication does not appear to be effective; Call in to primary care provider to increase dosage of pain medication or change prescription. This is an example of what charting method? a. Problem-oriented method b. PIE charting method c. Focus charting method d. Source-oriented method

a. Problem-oriented method

The Joint Commission is conducting an accreditation visit at the hospital. What is the focus of the evaluation being conducted? a. Quality assurance b. Magnet status c. Peer review d. Quality Improvement

a. Quality assurance

The nurse manager observes one of the unit nurses failing to wash hands on entering a client room. Hospital protocol is to wash hands before and after entering a client room. This scenario is an example of which approach to quality assurance? a. Quality by inspection b. Quality as opportunity c. Quality by perception d. Quality as initiative

a. Quality by inspection

Which statement best conveys the relationship between race and ethnicity? a. Race denotes physical characteristics, while ethnicity is rooted in a common heritage. b. Race and ethnicity can be considered to be synonymous in the context of health care. c. Race and ethnicity are both culturally determined concepts. d. Race is based on an individual's cultural history and is independent of ethnicity.

a. Race denotes physical characteristics, while ethnicity is rooted in a common heritage.

What information should the nurse document in the medication record when administering a non-narcotic pain medication? Select all that apply. a. Reason given b. Dose c. Time d. Vital signs e. Effectiveness of medication

a. Reason given b. Dose c. Time e. Effectiveness of medication

The nurse determines that the client is not meeting some of the expected outcomes in the plan of care. What are the next steps in the process? Select all that apply. a. Reevaluate each step of the nursing process. b. Identify contributing factors. c. Collect additional data. d. Delete all of the expected outcomes and write new ones. e. Add or alter nursing diagnoses.

a. Reevaluate each step of the nursing process. b. Identify contributing factors. c. Collect additional data. e. Add or alter nursing diagnoses.

In the clinical setting, a nurse is working on developing higher-level reflection skills. With which activity would the nurse most likely be engaged? a. Recalling a sequence of events b. Identifying a positive situation c. Thinking about relationships involved d. Reevaluating experience in light of ideas

a. Reevaluating experience in light of ideas

The nurse hears an unlicensed assitive personel (UAP) discussing a client's allergic reaction to a medication with another UAP in the cafeteria. What is the priority nursing action? a. Remind the UAP about the client's right to privacy. b. Notify the client relations department about the breach of privacy. c. Document the UAP's conversation. d. Report the UAP to the nurse manager.

a. Remind the UAP about the client's right to privacy.

Which behaviors demonstrated by the client would the nurse consider reflections of the client's pride in ethnicity? Select all that apply. a. Requesting native cuisine b. Listening to folk music and dance c. Asking to wear unique clothing d. Crying when given a diagnosis of cancer e. Requesting assistance when transferring from bed to chair

a. Requesting native cuisine b. Listening to folk music and dance c. Asking to wear unique clothing

During hospitalization, the client has developed shortness of breath with edema. What action should the nurse take? a. Revise the plan of care. b. Review the nursing care plan. c. Involve the family in changes. d. Implement changes in the current interventions.

a. Revise the plan of care.

The nurse is caring for a client who has an elevated temperature. When calling the health care provider, the nurse should use which communication tools to ensure that communication is clear and concise? a. SBAR b. MAR c. SOAP d. PIE

a. SBAR

Which statement is true of the nursing process? a. Scientific problem solving can occur within the nursing process. b. It is a valid alternative to using intuition to respond to nursing situations. c. It is more appropriate in medical surgical settings than community health care. d. Trial-and-error problem solving is incongruent with the nursing process.

a. Scientific problem solving can occur within the nursing process.

A nurse overhears another nurse make a statement that indicates racism. The nurse makes this determination based on which characteristic indicative of social value? a. Skin color b. Size c. Language d. Dress

a. Skin color

A newly hired young nurse overheard the charge nurse talking with an older nurse on the unit. The charge nurse said, "All these young nurses think they can come in late and leave early." What cultural factor can the new nurse assess from this conversation? a. Stereotyping b. Cultural blindness c. Cultural conflict d. Cultural imposition

a. Stereotyping

A nurse is working as part of a quality assurance team that uses the American Nurses Association model. The team is evaluating the resources of the facility as well as the physical facilities and equipment. Which type of evaluation is the team engaged in? a. Structure evaluation b. Process evaluation c. Outcome evaluation d. Quality by inspection

a. Structure evaluation

The charge nurse is reviewing SOAP format documentation with a newly hired nurse. What information should the charge nurse discuss? a. Subjective data should be included when documenting. b. Abnormal laboratory values are common items that are documented. c. Objective data are what the client states about the problem. d. The plan includes interventions, evaluation, and response.

a. Subjective data should be included when documenting.

Which are areas of focus in quality improvement? Select all that apply. a. Systems b. Processes c. Nurses d. Data use e. Individuals

a. Systems b. Processes d. Data use

When the nurse assesses the client's blood glucose level, what is the term for the type of skill the nurse is using? a. Technical b. Cognitive c. Ethical d. Interpersonal

a. Technical

A client has returned to the clinic for a postoperative visit. The nurse reviews the plan of care and may choose to take which actions based on the client's previous responses to the current plan of care? Select all that apply. a. Terminate the plan of care if the client has achieved outcomes. b. Modify the plan of care if the client has encountered difficulty with achieving outcomes. c. Explain the plan of care to significant others and advise them of the expectation that the client will achieve outcomes within a reasonable amount of time. d. Continue the plan of care if more time could result in achievement of outcomes.

a. Terminate the plan of care if the client has achieved outcomes. b. Modify the plan of care if the client has encountered difficulty with achieving outcomes. d. Continue the plan of care if more time could result in achievement of outcomes.

The younger nurses on a unit, who seem to adapt easily to the new technology presented, are perceived as threatening by two nurses who have worked on the unit for years. The older nurses begin to ridicule the younger nurses, saying, "You might be able to work a computer, but we know how to provide real care." How should the charge nurse respond? a. The charge nurse should discuss the concept of cultural conflict and help both parties see their respective value to the unit. b. The charge nurse should demonstrate cultural blindness and pretend that the issue does not exist. c. The charge nurse should understand that this is stereotyping in the form of racism and intervene immediately. d. The charge nurse should recognize that this is cultural imposition and the younger nurses are forcing new technology on the older nurses.

a. The charge nurse should discuss the concept of cultural conflict and help both parties see their respective value to the unit.

Which are cognitive client outcomes? Select all that apply. a. The client describes how to perform progressive muscle relaxation. b. The client reports cycling 30 minutes three times each week. c. The client correctly ambulates with a walker. d. The client lists the side effects of digoxin. e. The client identifies signs and symptoms of hypoglycemia.

a. The client describes how to perform progressive muscle relaxation. d. The client lists the side effects of digoxin. e. The client identifies signs and symptoms of hypoglycemia.

The nurse is caring for several clients of different cultures. Which client situation would the nurse recognize as the client with highest risk of culture shock? a. The client from Ethiopia states, "All these machines attached to me scare me and I need to get them off." b. The client from Mexico states, " I am having difficulty reading these pamphlets you gave me." c. The client from France states, "I miss my family back home." d. The client from England states, "I do not understand what all these new medications are for."

a. The client from Ethiopia states, "All these machines attached to me scare me and I need to get them off."

A client has been recently diagnosed with diabetes after receiving emergency treatment for a hyperglycemic episode. Which of the client's actions indicates that the client has achieved a cognitive outcome in the management of this new health problem? a. The client is able to explain when and why the client needs to check the blood glucose level. b. The client has maintained blood glucose levels within acceptable range in the days prior to discharge. c. The client can demonstrate the correct technique for using a new glucometer. d. The client expresses a desire to change the way that the client eats and exercises.

a. The client is able to explain when and why the client needs to check the blood glucose level.

The home health nurse is conducting a health history interview with a client that immigrated 3 months ago. What action observed by the nurse would indicate that the client is adapting to the majority culture? a. The client reads books in the language of the new country. b. The client prepares traditional meals of the native country. c. The client lives in a neighborhood that has a population of immigrants from the native country. d. The client prefers to watch television in the native language.

a. The client reads books in the language of the new country.

A nurse convinces a client who is a Jehovah's Witness that receiving blood products is more important than the legalistic components of religion. What client reaction may be expected following this mandated change? a. The client states, "I feel like I abandoned my religion." b. The client states, "I am glad that nurse told me what to do." c. The client states, "I can't get over my feelings of legalism as a Jehovah's Witness." d. The client states, "Why isn't blood administration forced on all who need that treatment?"

a. The client states, "I feel like I abandoned my religion."

Which client outcome is an example of a physiologic outcome? a. The client's pulse oximetry reading is 97% on room air 30 minutes after removal of a nasal cannula. b. The client reports walking for 30 minutes each day. c. The client demonstrates active range-of-motion exercises with left upper extremity. d. The client explains how to administer a vaginal cream.

a. The client's pulse oximetry reading is 97% on room air 30 minutes after removal of a nasal cannula.

Which nursing action reflects evaluation? a. The nurse assesses the client's response to pain medication. b. The nurse performs colostomy irrigation. c. The nurse identifies that the client has wound drainage. d. The nurse sets an anxiety level of 3 or less with the client.

a. The nurse assesses the client's response to pain medication.

A nurse evaluates clients prior to discharge from a hospital setting. Which action is the most important act of evaluation performed by the nurse? a. The nurse evaluates the client's goal/outcome achievement. b. The nurse evaluates the plan of care. c. The nurse evaluates the competence of nurse practitioners. d. The nurse evaluates the types of health care services available to the client.

a. The nurse evaluates the client's goal/outcome achievement.

Which nursing actions reflect the evaluation stage of the nursing process? Select all that apply. a. The nurse identifies that a client's pain is not being adequately treated. b. The nurse sets an anxiety level of 3 or less with the client. c. The nurse performs tracheostomy care using sterile technique. d. The nurse documents the client's response to suctioning. e. The nurse determines the client did not lose the expected 2 lb (0.90 kg).

a. The nurse identifies that a client's pain is not being adequately treated. d. The nurse documents the client's response to suctioning. e. The nurse determines the client did not lose the expected 2 lb (0.90 kg).

Which statements are true of factors that influence client responses and outcome achievement and of how the nurse should use them? Select all that apply. a. The nurse should draw on positive factors to deal with other variables in the future. b. The nurse should reinforce negative behaviors to achieve desired outcomes. c. A client's strong motivation to learn appropriate health behaviors is an example of a positive factor. d. A nurse uses only positive factors to manipulate client outcomes. e. The nurse should identify which factors are helpful to attaining outcomes and manipulate them to achieve goals.

a. The nurse should draw on positive factors to deal with other variables in the future. c. A client's strong motivation to learn appropriate health behaviors is an example of a positive factor. e. The nurse should identify which factors are helpful to attaining outcomes and manipulate them to achieve goals.

A client is admitted to a hospital unit with scleroderma. The nurse is unfamiliar with this condition. What is the nurse's best source of information about this condition? a.The nursing and medical literature b.The client's physician c.The client's chart d.The client

a. The nursing and medical literature

A nurse is documenting care for clients in a hospital setting. Which documenting errors may potentially increase the nurse's risk for legal problems? Select all that apply. a. There are lines between the entries. b. The content is not in accordance with professional standards. c. Dates and times of entries are omitted. d. The documentation is not countersigned. e. The content includes descriptions of situations that are out of the ordinary. f. The content reflects client needs.

a. There are lines between the entries. b. The content is not in accordance with professional standards. c. Dates and times of entries are omitted.

Which is a cultural norm of the health care system? a. There is the use of a systematic approach and problem-solving methodology. b. There is a tolerance of tardiness, disorderliness, and disorganization. c. There are rigid procedures attending birth and death. d. The omnipotence of technology is yet to be recognized.

a. There is the use of a systematic approach and problem-solving methodology.

A client has requested a translator to help understand the questions that the nurse is asking during the client interview. The nurse knows that what is important when working with a client translator? a. Translators may need additional explanations of medical terms. b. Talking loudly helps the translator and the client understand the information better. c. It is always okay to not use a translator if a family member can do it. d. Talking directly to the translator facilitates the transfer of information.

a. Translators may need additional explanations of medical terms.

The nursing process is based upon the process of problem solving. The nurse attempts to obtain a blood pressure on the client's right arm, then on the left arm, then on the left leg, and finally on the right leg, where the blood pressure is obtained. What type of problem solving did the nurse use? a. Trial-and-error problem solving b. Intuitive thinking c. Scientific problem solving d. Critical thinking

a. Trial-and-error problem solving

A nurse has explained her intention to conduct a Weber test and a Rinne test. Which pieces of equipment will the nurse require? a. Tuning fork-Snellen chart-Otoscope-Ophthalmoscope b. Snellen chart-Otoscope-Ophthalmoscope c. Otoscope-Ophthalmoscope d. Ophthalmoscope

a. Tuning fork-Snellen chart-Otoscope-Ophthalmoscope

A new graduate is working at a first job. Which statement is most important for the new nurse to follow? a. Use abbreviations approved by the facility. b. Document lengthy entries using complete sentences. c. Only document changes in the client's status. d. Use PIE charting, even if it is not the institution's charting method.

a. Use abbreviations approved by the facility.

A client was recently hospitalized. To process insurance payment, the insurance company requested access to the client's payment information. What is the most appropriate response to maintain client privacy? a. Use minimum disclosure policy to release the information. b. Release the full medical record to expedite payment. c. Refer the insurance agency directly to the client. d. Do not release any information to the insurance company.

a. Use minimum disclosure policy to release the information.

The nurse is providing discharge teaching for a client who is from a different culture. The nurse notes that the client will look away from the nurse and does not maintain eye contact. What would be the most appropriate action by the nurse, with regard to culturally competent care? a. Utilize a key informant and continue with the teaching, verifying the client's understanding through open-ended questions. b. Tell the client that it is important to pay attention to the teaching and ask if the client would like an interpreter. c. Change positions to promote eye contact with the client, asking open-ended questions to ensure understanding. d. Continue with teaching and leave written instructions for the client to review.

a. Utilize a key informant and continue with the teaching, verifying the client's understanding through open-ended questions.

Which documentation by the nurse best supports the PIE charting system? a. Vomiting 250 mL undigested food, antiemetic given, no further vomiting b. Vomiting 250 mL undigested food, states abdominal pain, blood pressure 114/68 mm Hg c. Blood pressure 88/42 mm Hg, 500 mL IV fluids given, no statements of nausea d. States nauseated, vomiting 250 mL undigested food, hypoactive bowel sounds, antiemetic given

a. Vomiting 250 mL undigested food, antiemetic given, no further vomiting

A client has been admitted to the unit for chest pain. A nurse told the family that they could not be with the client. The family became very upset, and now the client wants to leave. What is the most culturally appropriate response by the charge nurse? a. "Would you feel more comfortable with your family with you?" b. "If you leave, you will be asked to sign a form indicating that you are leaving against medical advice." c. "Sometimes family can cause stress, and we try to maintain a stress-free care environment." d. "Why do you think you need to leave?"

a. Would you feel more comfortable with your family with you?"

A nurse is arranging for home care for clients and reviews the Medicare reimbursement requirements. Which client meets one of these requirements? a. a client who is homebound and needs skilled nursing care b. a client who is not making progress in expected outcomes of care c. a client whose rehabilitation potential is not good d. a client whose status is stabilized

a. a client who is homebound and needs skilled nursing care

The nurse, while admitting an older adult client, charts, "The client does not respond when I speak while standing on the client's right side." This statement is an example of: a. a cue. b. a misinterpretation. c. duplicate data. d. an inference.

a. a cue.

An older adult client who only speaks the nondominant language has been admitted to the emergency department after suffering a fall and suspected hip fracture in the home. Who is the best person to perform translation services for the client? a. a hospital translator b. a family member c. a trusted friend d. a bilingual hospital employee

a. a hospital translator

With input from the staff, the nurse manager has determined that bedside reporting will begin for all client handoff at shift change to improve client safety and quality. When performing bedside reporting, what information should the nurse include? Select all that apply. a. any abnormal occurrences with the client during the shift b. what the client watched on television during the shift c. identifying demographics, including diagnosis d. what time the nurse will return for the next shift e. current orders

a. any abnormal occurrences with the client during the shift c. identifying demographics, including diagnosis e. current orders

Which scenario represents a nurse demonstrating the critical thinking process? a. assessing whether physician help is needed b. assessing why a physician encounter form is missing from the record c. collaborating with the respiratory therapist and physical therapist to address a complication d. using power for more control and freedom over the daily tasks

a. assessing whether physician help is needed

A nurse is conducting focused data collection and recognizes the existence of cues. The nurse is most likely involved in which phase of the nursing process? a. Assessment b. Diagnosis c. Planning d. Implementation

a. assessment

Which components should the nurse include when documenting a critical pathway? Select all that apply. a. care plan b. expected outcomes c. timeline d. subjective data e. significant deviations

a. care plan b. expected outcomes c. timeline

A client has had major abdominal surgery and just returned to the unit from the operating room. The nursing priority is to: a. complete the postoperative assessment. b. evaluate the abdominal dressing for drainage. c. administer pain medication. d. expect the client to be drowsy, and let the client rest.

a. complete the postoperative assessment.

A nurse is caring for a client with bacterial pneumonia and a temperature of 104°F (40.0°C). Yesterday, the client's temperature was 102°F (38.9°C). The health care provider on call prescribes cool compresses for the client to help lower the fever. The client insists that the nurse bring warm blankets because they will help the client to recover more quickly. The nurse recognizes that the client's request is an example of: a. cultural ritual. b. cultural competence. c. cultural stereotyping. d. ethnocentrism.

a. cultural ritual.

The focus of a hospital's current quality assurance program is a comparison of the health status of clients on admission and with that at the time of discharge. This form of quality assurance is characteristic of: a. outcome evaluation. b. structure evaluation. c. process evaluation. d. nursing audit.

a. outcome evaluation.

What dual purpose does an audit serve? a. quality assurance and reimbursement b. communication and evaluation c. education and confidentiality d. knowledge and quality

a. quality assurance and reimbursement

The spouse of a client asks the nurse whether the spouse may bring in a cream from home to apply to the client's skin. The spouse says, "Whenever anyone gets sick, we always use this cream." The nurse interprets this as: a. ritual. b. ethnocentrism. c. stereotyping. d. subculture.

a. ritual.

One of the outcomes that has been identified in the care of a client with a new suprapubic catheter is that he will demonstrate the correct technique for cleaning his insertion site and changing his catheter prior to discharge. When should this outcome be evaluated? a. throughout the client's hospital admission b. when the client is discharged c. during the first home health care visit d. once the primary care physician has written a discharge order

a. throughout the client's hospital admission

A nurse is caring for clients in a predominantly black community. What values or beliefs are commonly shared by members of this culture? Select all that apply. a. Oriented to the present b. Frequently highly religious c. Youth valued over age d. Praise of self or others considered poor manners e. Respect given according to sex (male) f. Clergy members highly respected

a.Oriented to the present b.Frequently highly religious f.Clergy members highly respected

A new client comes to the primary care clinic and asks for help treating head lice. The nurse assesses that the client lives in low-income housing, and nine other people live with her in a one-bedroom apartment. Which of the following is a primary nursing concern? a. The client does not have running water. b. The client has no hope for the future. c. The client receives government assistance. d. The client does not have air-conditioning.

a.The client does not have running water.

In addressing health promotion for a client who is a member of another culture, the nurse should be guided by which principle? a. The client may have a very different understanding of health promotion. b. Health promotion is a concept that is largely exclusive to Western cultures. c. A culture's conceptualization of health promotion is a result of that culture's level of socioeconomic development. d. The nurse should avoid performing health promotion education if this is not a priority in the client's culture.

a.The client may have a very different understanding of health promotion.

A nurse identifies the following: "The client will report a pain rating of 4 or less within 30 to 45 minutes of receiving prescribed analgesic." The nurse has identified: a. outcome. b. subjective data. c. nursing diagnosis. d. intervention.

a.outcome

A nurse obtaining the most important information first during an assessment of a client is primarily an example of the nurse being: a.purposeful. b.factual. c.complete. d.able to prioritize.

able to prioritize.

A client comes to a community mental health clinic for a psychiatric evaluation at the family's request. During the initial interview, the client tells the nurse about painting the streets to beautify the city, lecturing subway riders about germ control, and banning smoking in order to clean up the environment. The client is irritable and easily distracted by the slightest sound. Which stage of mania is the client exhibiting?

acute mania

An 80-year-old client has a body temperature of 97°F (36°C). Which condition best accounts for this client's temperature reading?

advanced age

The client is self-monitoring blood pressure at home and reports that every reading is 150/90 mmHg. What is the priority nursing intervention?

ask the client to demonstrate self-blood pressure assessment

A nurse is teaching a young female client about breast cancer prevention. The client asks at what age she needs to begin having mammograms. What is the nurse's best response? a) "Your physician will decide when it is best for you to begin having mammograms based on your family history." b) "According to the American and Canadian Cancer Societies, your first mammogram should be done at age 40 and then yearly after that." c) "Why do you want to know? Do you have a history of breast or ovarian cancer in your family?" d) "Don't worry about that yet; you are still young. You will not need a mammogram until you are in your 40s."

b) "According to the American and Canadian Cancer Societies, your first mammogram should be done at age 40 and then yearly after that."

An older adult client admitted 4 days ago is being treated for chronic obstructive pulmonary disease (COPD) and now appears confused. What question will the nurse ask to determine the client's level of orientation? a) "How are you feeling?" b) "Can you tell me where you are right now?" c) "Have you been more confused?" d) "Do you know what day this is?"

b) "Can you tell me where you are right now?"

The nurse at the neighborhood family clinic is teaching a 55-year-old client with hypertension and a family history of heart disease about reduction of risk factors. It is most important for the nurse to make which statement to the client? a) "You should decrease your intake of fried foods." b) "Take your blood pressure medications exactly as your doctor prescribed them." c) "It is important for you to do 30 minutes of exercise three times a week." d) "You need to sign up for the clinic's stop smoking program."

b) "Take your blood pressure medications exactly as your doctor prescribed them."

The nurse is performing an assessment for a 12-months-old child and observes pronation of the child's feet. The parent asks the nurse what is wrong with the child's feet. What is the bestresponse by the nurse? a) "It is uncommon for a child to have a deformity of the feet like this. I will call the health care provider." b) "This is an age-related variation for the child and should go away after about 30 months." c) "We will make a referral to the pediatric orthopedic clinic to determine what is wrong with the feet." d) "Your child may likely have to have serial casting done to correct the deformity of the feet."

b) "This is an age-related variation for the child and should go away after about 30 months."

A nurse has developed strong rapport with the spouse of a client who has been receiving rehabilitation following a debilitating stroke. The spouse has just been informed that the client is unlikely to return home and requires care that can only be provided in a facility with constant nursing care. The client's spouse tells the nurse, "I can't believe it's come to this." How should the nurse best respond? a) "Why do you think that the care team has made this recommendation?" b) "This must be very difficult for you to hear. How do you feel right now?" c) "Do you understand that everyone here has your spouse's best interest at heart?" d) "What would help you accept that this is best for both of you?"

b) "This must be very difficult for you to hear. How do you feel right now?"

A client with a cardiac dysrhythmia was recently prescribed metoprolol and is at a follow-up appointment at the cardiologist's office. The client tells the nurse, "I feel depressed, tired, and I have no desire to exercise." To determine a cause-and-effect relationship, the nurse should ask: a) "Have you tried exercising at all in the last week or two?" b) "Were you tired and depressed before starting the new medication?" c) "Do you check your blood pressure and pulse before you take your medication?" d) "Tell me about the foods you are eating."

b) "Were you tired and depressed before starting the new medication?"

The nurse is caring for a client admitted with a head injury. Which question should the nurse ask to determine the client's remote memory? a) "What did you eat for dinner last night?" b) "What are the month, date, and the year of your birth?" c) "What is meant by 'an ounce of prevention is worth a pound of cure'?" d) "What are the three objects I told you earlier?"

b) "What are the month, date, and the year of your birth?"

The client is being discharged, and the nurse observes the client crying. What is the nurse's most appropriate response? a) "Are you scared because you are going home?" b) "Would you like to talk about anything before you go home?" c) "Let's discuss your discharge plan." d) "What is your pain level at this time?"

b) "Would you like to talk about anything before you go home?"

A nurse is completing a health history with a client being admitted for a mastectomy. During the interview the client states, "I do not know what to do. I am not sure if I really need this surgery." Which response by the nurse demonstrates active listening? a) "You seem unsure, please let me know if you decide to postpone the surgery until you are no longer unsure." b) "You seem unsure. Tell me your concerns about your surgery." c) "I understand your confused, what do you think you should do?" d) "I understand you are not sure about having the surgery. Why do you think you really do not need the surgery?"

b) "You seem unsure. Tell me your concerns about your surgery."

A nurse assesses a client's nails. What is a normal finding? a) Skin-toned cuticles b) 160-degree angle of nail attachment c) Capillary refill of 5 seconds d) Concave nails

b) 160-degree angle of nail attachment

During assessment of the lower extremities, the nurse notes that the bilateral lower extremities are pink, intact, warm, and soft to touch, as well as normal in contour with a 4-mm depression in the skin after pressing that returns after 2 seconds. Which is the correct interpretation and documentation of this result? a) 1+ pitting edema noted on bilateral lower extremities b) 2+ pitting edema noted on bilateral lower extremities c) brawny edema noted over bilateral lower extremities d) bilateral lower extremities within normal limits

b) 2+ pitting edema noted on bilateral lower extremities

A nurse states the following to another nurse who is constantly forgetting to wash hands between clients: "It looks like you keep forgetting to wash your hands between clients. It's really not safe for your clients. Let's think of some type of reminder we can use to help you remember." This communication is an example of what type of speech? a) Aggressive b) Assertive c) Nonassertive d) Therapeutic

b) Assertive

The nurse is preparing to administer a blood pressure medication to a client. To ensure the client's safety, what is the priority action for the nurse to take? a) Tell the client to report any side effects experienced. b) Assess the client's blood pressure to determine if the medication is indicated. c) Determine the client's reaction to the medication in the past. d) Ask the client to verbalize the purpose of the medication.

b) Assess the client's blood pressure to determine if the medication is indicated.

An unlicensed assistive personnel (UAP) has worked on the postpartum unit for many years. The UAP has been oriented well and provides excellent client care. What duties could the professional nurse appropriately delegate to the UAP? Select all that apply. a) Initial assessment of the mother after birth of the infant b) Assisting the client with personal hygiene needs and ambulation c) Assisting and teaching the client to breastfeed the infant d) Providing routine discharge instructions related to infant care e) Transporting the infant to the mother's room according to hospital policy

b) Assisting the client with personal hygiene needs and ambulation e) Transporting the infant to the mother's room according to hospital policy

A 66-year-old female client is reporting abdominal pain. The nurse assesses the client's abdomen by first inspecting the abdomen. What should the nurse do next? a) Measure abdominal girth. b) Auscultate the abdomen. c) Percuss the abdomen. d) Palpate the abdomen.

b) Auscultate the abdomen.

An older client presents to the clinic with reports of dyspnea upon exertion and when lying down as well as feeling tired all the time. The nurse notes that the client's ankles and feet are swollen. What cardiac assessment technique would the nurse use? a) Inspection b) Auscultation c) Percussion d) Palpation

b) Auscultation

The nurse is performing a respiratory assessment for a client and hears a high-pitched, harsh "blowing" sound, with sound on expiration being longer than inspiration. How will the nurse document this finding? a) Adventitious breath sounds b) Bronchial breath sounds c) Bronchovesicular breath sounds d) Vesicular breath sounds

b) Bronchial breath sounds

A nurse is performing auscultation. The nurse would use the bell of the stethoscope to auscultate which sounds? a) Breath sounds b) Bruits c) Normal heart sounds d) Bowel sounds

b) Bruits

A gerontologic nurse is inspecting the genitalia of an older adult male client. Which assessment findings are of the most concern? Select all that apply. a) Decreased penis size b) Bulge to the left inguinal area c) Scant amount of pubic hair d) Scant yellow discharge e) Less firmness of the testes

b) Bulge to the left inguinal area d) Scant yellow discharge

After instituting interventions to increase oxygenation, the client shows no signs of improvement. What is the nurse's priority action? a) Determine the client's code status in case of an emergency. b) Communicate with the physician for additional orders. c) Reassess the client for improvement in 30 minutes. d) Document the interventions and the result.

b) Communicate with the physician for additional orders.

A client is diagnosed with diabetes. The client's adult child offers to serve as an interpreter, because the client does not speak the dominant language. Which is the best action for the nurse to take? a) Ask a fellow nurse who knows some words in the client's language to help. b) Contact a professional interpreter. c) Allow the client's child to interpret. d) Involve a friend who speaks both the dominant and the client's languages.

b) Contact a professional interpreter.

The nurse detects a weak, thready pulse found from a client palpating peripheral pulses. What condition does the nurse suspect the client is experiencing? a) Inflammation of a vein b) Decreased cardiac output c) Hypertension and circulatory overload d) Impaired kidney function

b) Decreased cardiac output

A nurse is assessing the lungs of a client and auscultates soft, low-pitched sounds over the base of the lungs during inspiration. What would be the nurse's next action? a) Assess for asthma. b) Document normal breath sounds. c) Suspect an inflamed pleura rubbing against the chest wall. d) Recommend testing for pneumonia.

b) Document normal breath sounds.

Which is the nurse's priority question to consider prior to delegating a task to an unlicensed assistive personnel (UAP)? a) How can I supervise the completion of this task? b) Does this task fall within the scope of a UAP? c) What is the client's condition? d) How can I explain the task to the UAP?

b) Does this task fall within the scope of a UAP?

Which quality in a nurse helps the nurse to become effective in providing for a client's needs while remaining compassionately detached? a) Sympathy b) Empathy c) Kindness d) Commiseration

b) Empathy

The nurse weighs the client using a portable bed scale. The obtained weight is 10 lb (4.5 kg) more than the nurse expected. What action does the nurse take next? a) Obtain a second scale to verify the measurement. b) Ensure equipment is not hanging into the sling. c) Notify the health care provider of the abnormal finding. d) Document the weight in the medical record.

b) Ensure equipment is not hanging into the sling.

Mr. Martinez is a 55-year-old male who was brought to the emergency department (ED). He reports abdominal pain in his right lower quadrant (RLQ) and nausea without vomiting. The nurse performs a physical assessment on the client and documents the following: Neurologic status: awake and alert; Cardiovascular: radial pulses 90, bounding, and equal; Skin: warm and dry; Respirations: 24 and regular; Gastrointestinal: abdominal pain with rebound tenderness in RLQ; Musculoskeletal: sitting up in bed with knees bent. Identify which findings involved the assessment technique of palpation. Select all that apply. a) Neurologic: awake and alert b) Gastrointestinal: abdominal pain with rebound tenderness in RLQ c) Respirations: 24 and regular d) Cardiovascular: radial pulses 90, bounding, and equal e) Skin: warm and dry f) Musculoskeletal: sitting up in bed with knees bent

b) Gastrointestinal: abdominal pain with rebound tenderness in RLQ d) Cardiovascular: radial pulses 90, bounding, and equal e) Skin: warm and dry

The nurse is caring for a 44-year-old female client with a diagnosis of deep vein thrombosis (DVT) in her left lower leg. What assessment method should the nurse perform first? a) Lightly palpate the left leg, assessing for edema. b) Inspect the left lower leg for areas of redness. c) Assess for pain by deeply palpating the left leg. d) Palpate the popliteal and posterior tibial pulses of both legs.

b) Inspect the left lower leg for areas of redness.

A nurse auscultates the right carotid artery in an older adult client and identifies a bruit. What does this assessment finding mean? a) It is normal. b) It is distended. c) It is inflamed. d) It is dissecting.

b) It is distended.

The physician has ordered that the client should ambulate 3 times a day. The nurse enters the room to ambulate the client and the client reports pain. What is the nurse's most appropriate action? a) Ambulate the client and medicate later. b) Medicate the client and wait to ambulate later. c) Emphasize to the client the importance of following the treatment plan. d) Explain to the client the benefits of ambulation.

b) Medicate the client and wait to ambulate later.

Sanjay Patel is a 10-year-old boy from India with mahogany-colored skin. He arrives at the school nurse's office and tells the nurse that he was stung by a wasp on the arm yesterday, and he thinks it might be infected. The nurse performs which action in order to assess the wasp sting site for inflammation? a) Palpate the area of the sting with ungloved fingertips to assess the skin for increased warmth. b) Palpate the area with the back of the hand for increased warmth, then touch the other arm for comparison. c) Shine a light on the area of the sting and observe the skin for signs of redness (erythema). d) Take the child's oral and axillary temperatures and compare the values.

b) Palpate the area with the back of the hand for increased warmth, then touch the other arm for comparison.

The emergency room (ER) has a strict protocol regarding intramuscular (IM) injection technique. A nurse working in the ER has learned of a new technique to decrease pain with IM injections from the nursing literature and would like to use it. What is the most appropriate way for the nurse to implement the technique? a) Begin using the technique to determine whether it is effective. b) Petition to change the protocol based on the new evidence. c) Ask the ER physician to order IM injections with the new technique. d) Research the protocols at other area emergency rooms.

b) Petition to change the protocol based on the new evidence.

The nurse is reporting to an oncoming nurse about the care of a client using the SBAR format. The nurse informs the oncoming nurse that the client should continue to have neurolgoical checks every 2 hours and the nurse should report any alterations to the health care provider. In which section should this information be relayed? a) Assessment b) Recommendation c) Background d) Situation

b) Recommendation

The registered nurse (RN) is delegating the task of assisting a postoperative client to the bathroom to the unlicensed assistive personnel (UAP). The nurse witnessed the UAP correctly perform the task on previous occasions and knows the UAP is competent to perform the task. The nurse has communicated how to get the client out and back into bed and told the UAP not to allow the client to bear weight on the left leg. The nurse validated that the activity was completed and gave the UAP feedback. Which delegation guideline did the nurse omit? a) Right task b) Right circumstance c) Right person d) Right supervision

b) Right circumstance

The nurse has performed a Romberg test in the context of a client's neurologic assessment. The client has failed the test. The nurse should consequently identify what nursing diagnosis? a) Acute Confusion b) Risk for Falls c) Disturbed Thought Processes d) Chronic Confusion

b) Risk for Falls

While observing a new nurse inserting an indwelling urinary catheter, the preceptor observes a break in sterile technique. What is the preceptor's first action? a) Report the new nurse's error to the nurse manager for corrective action. b) Tell the new nurse that a break in sterile technique has occurred and the procedure must be stopped. c) Assign the new nurse to view videos on sterile catheter insertion. d) Allow the new nurse to continue with the insertion and discuss the error later away from the client.

b) Tell the new nurse that a break in sterile technique has occurred and the procedure must be stopped.

Mrs. Harris was admitted to the psychiatric unit 3 days ago with a diagnosis of major depressive disorder. The client answers assessment questions with barely audible "yes" or "no" responses and tells the nurse that she has been depressed for a long time. She wants the door closed and the curtains drawn to darken her room. She refuses visitors, eats only 25% of her meals, and tells the nurse that the food makes her nauseous. The nurse observes the client biting her fingernails. She cries often and sleeps a lot. The nurse documents which client actions as objective assessment data? Select all that apply. a) The client states that she has been depressed for a long time. b) The client bites her fingernails. c) The client says that the food makes her nauseous. d) The client sleeps a lot. e) The client eats 25% of her meals. f) The client answers questions in a barely audible voice.

b) The client bites her fingernails. d) The client sleeps a lot. e) The client eats 25% of her meals. f) The client answers questions in a barely audible voice.

The nurse is planning instruction on wound care to an adult client. What variables would cause the nurse to alter the education plan? Select all that apply. a) The client is male. b) The client is blind. c) The client is an architect. d) The client denies the need for education. e) The client is married.

b) The client is blind. d) The client denies the need for education.

The client has a diagnosis of Risk for Injury related to falls. How would the nurse know if the intervention was successful? a) The client calls for assistance to get out of bed. b) The client is free of falls. c) The client is taught safety precautions. d) The client verbalizes risks for injury.

b) The client is free of falls.

A 33-year-old male client returns to the medical-surgical unit following a thyroidectomy. Which assessment finding requires an immediate intervention by the nurse? a) The client reports thirst. b) The client makes noises when he breathes. c) The client reports pain at the surgical site. d) The client is sleepy from the anesthesia.

b) The client makes noises when he breathes.

A nurse is completing a neurologic assessment of an 84-year-old client. Which principle should guide the nurse's interpretation of the results? a) The client will likely have difficulty expressing or understanding abstract concepts. b) The client's reaction time will likely be slower than that of a younger adult. c) The client will experience lapses in short- and long-term memory. d) The client's arm and leg strength will be more asymmetric than that of a younger client.

b) The client's reaction time will likely be slower than that of a younger adult.

A nurse is communicating the plan of care to a client who is cognitively impaired. Which nursing actions facilitate this process? Select all that apply. a) If there is no response, the nurse does not repeat what is said and takes a break. b) The nurse maintains eye contact with the client. c) The nurse keeps communication simple and concrete. d) The nurse gives lengthy explanations of the care that will be given. e) The nurse shows patience with the client and gives the client time to respond. f) The nurse communicates in a busy environment to hold the client's attention.

b) The nurse maintains eye contact with the client. c) The nurse keeps communication simple and concrete. e) The nurse shows patience with the client and gives the client time to respond.

A nurse who is experienced caring only for well babies is assigned to the neonatal intensive care unit (NICU) because of a shortage of nurses in the NICU. The nurse is assigned to an infant on a ventilator who will require blood transfusions during the shift. What is the nurse's most appropriate course of action? a) The nurse should recognize the necessity of the assignment and provide care to the best of the nurse's ability. b) The nurse should inform the charge nurse that the nurse does not have the experience to properly care for this client. c) The nurse should ask another nurse who was previously assigned to the client for instruction. d) The nurse should request that the blood transfusions be delayed until the next shift.

b) The nurse should inform the charge nurse that the nurse does not have the experience to properly care for this client.

The nurse is assessing the ear canal and tympanic membrane of a client using an otoscope. Which finding would the nurse document as normal? a) The ear canal is smooth and white. b) The tympanic membrane is translucent, shiny, and gray. c) The ear canal is rough and pinkish. d) The tympanic membrane is reddish.

b) The tympanic membrane is translucent, shiny, and gray.

When collecting data on a client, the nurse implements which nonverbal communication form as one of the most effective to express feelings? a) Body posture b) Touch c) Eye contact d) Gait

b) Touch

The nurse should use the bell of the stethoscope during auscultation of: a) a client's bowel sounds. b) a client's heart murmur. c) a client's breath sounds. d) a client's apical heart rate.

b) a client's heart murmur.

Carl Rogers (1961) studied the process of therapeutic communication. Through his research, the elements of a "helpful" person were described. They include all of the following except which choice? a) comfortable sense of self b) analysis c) positive regard d) empathy

b) analysis

A nurse is on lunch break in the hospital cafeteria and sits at a table near a group of physicians eating their lunch. One of the physicians, who is in charge of the nurse's clients, points at the nurse and states, "That guy needs to get fired." The best response by the nurse would be to: a) return to the nurse's home unit and ask to meet with the charge nurse. b) ask to speak to the physician in private and address the disrespectful remark. c) write a written account of what transpired and contact an attorney. d) call the nursing supervisor to address the situation at hand.

b) ask to speak to the physician in private and address the disrespectful remark.

An evening shift nurse is caring for a client scheduled for a colon resection in the morning. The client tells the nurse that the client is afraid of waking up during surgery. The best response by the nurse is to: a) look directly at the client and state, "You are afraid of waking up during surgery." b) ask why the client thinks the client will wake up during surgery. c) state "everyone is afraid of that." d) ask the surgeon to come to the bedside to reassure the client.

b) ask why the client thinks the client will wake up during surgery.

To obtain subjective data about a newly admitted client's sleep pattern, the nurse: a) determines how frequently the client naps. b) asks the client what promotes sleep. c) documents the client's affect and yawning. d) inspects the client's eyes for redness.

b) asks the client what promotes sleep

A client states during the interview that he has pain in his lower back. He states it is a 10 on a scale of 0 to 10 when he is asked to turn. The nurse should: a) have the client turn from side to side and assess pain. b) avoid a position change that requires turning. c) have the client lay on his right side, then palpate the area. d) elevate the legs, bending at the knee while the client is supine.

b) avoid a position change that requires turning.

When a client enters the acute care facility, the nurse should perform a: a) spiritual health assessment. b) comprehensive health assessment. c) physical health assessment. d) focused health assessment.

b) comprehensive health assessment.

A nurse is caring for a client admitted to the hospital for dehydration. The physical findings consistent with this diagnosis that the client's general appearance can nonverbally communicate to the nurse include: a) pallor and diaphoresis. b) easy wrinkling of the skin and sunken eyes. c) slow heart rate and prolonged capillary refill. d) cold intolerance and brittle nails.

b) easy wrinkling of the skin and sunken eyes.

A nurse is performing a wet to dry dressing change on a client's lower abdomen. The nurse should be aware that he or she will be encroaching on which zone? a) public zone b) intimate zone c) personal zone d) social zone

b) intimate zone

To assess a client's visual accommodation, the nurse has the client: a) sit still while a penlight is shined at the pupil. b) look at a close object, then at a distant object. c) look straight ahead with one eye covered. d) stand 20 feet from the Snellen chart.

b) look at a close object, then at a distant object.

A client is being treated for chronic obstructive pulmonary disease. The nurse auscultates the client's lungs following a period of coughing. The findings of this assessment are an example of: a) subjective data. b) objective data. c) baseline data. d) comprehensive data.

b) objective data.

A nurse is evaluating a client's orientation after he was brought into the ER following a car accident. What is indicated by "Oriented x3"? a) oriented to person, place, and situation b) oriented to person, place, and time c) oriented to person, situation, and time d) oriented to hospital, person, and date

b) oriented to person, place, and time

A nurse is caring for a client who presents with a skin infection. While obtaining the client's medical history, it is determined that the client is an intravenous drug user. To foster effective communication, the nurse should: a) consult with the social worker regarding inpatient drug rehabilitation. b) remain honest, open, and frank. c) ask the client for a urine specimen for urine drug use screening. d) ask if the client realizes the infection is a direct result of the drug use.

b) remain honest, open, and frank.

A nurse is interviewing a client for the establishment of long-term care insurance. During the interview, the nurse asks questions regarding the client's past medical history. In this case, the nurse plays the role in the process of communication of the: a) target. b) sender. c) decoder. d) receiver.

b) sender

A nurse is caring for a postoperative client 1 day after coronary artery bypass surgery. Which nursing interventions demonstrate the skill of assessment? Select all that apply. a) helping the client to bathe and brush their teeth b) taking the client's blood pressure c) inspecting the abdominal incision d) reviewing morning lab results e) assisting the client to sit up in a chair

b) taking the client's blood pressure c) inspecting the abdominal incision d) reviewing morning lab results

Palpation is the use of hands and fingers to gather information through touch. Different parts of the hand are more suitable for different tactile sensations. Which part of the hand is bestfor sensing temperature? a) the palm b) the dorsum c) the fingertips d) the knuckles

b) the dorsum

The nurse is caring for a client who is postoperative 24 hours from an appendectomy. The client is hesitant to get out of bed. How should the nurse respond? a. "You need to get up and walk to prevent complications." b. "Can you describe what you are feeling when you try to move?" c. "I will come back later and help you get up." d. "Would like to wait until your family arrives to get out of bed?"

b. "Can you describe what you are feeling when you try to move?"

The nurse has just attended a seminar on concepts of cultural diversity. Which statement made by the nurse would require further education? a. "Culture helps to define identity within specific groups of people." b. "Culture cannot be influenced, and you are born with your culture." c. "Language is the primary way that people share their culture." d. "Culture can be seen in attitudes and institutions of certain populations."

b. "Culture cannot be influenced, and you are born with your culture."

A nurse is transfusing multiple units of packed red blood cells. After the second unit is transfused, the nurse auscultates bilateral crackles at the bases of the client's lungs and the client reports dyspnea. The nurse telephones the health care provider and provides an SBAR report. Which statement represents the final step in this type of communication? a. "I am calling because the client receiving blood has developed dyspnea and had crackles." b. "I think the client would benefit from intravenous furosemide." c. "It seems like this client has fluid volume overload." d. "This client has a medical history of heart failure."

b. "I think the client would benefit from intravenous furosemide."

After conducting the initial assessment of a new resident of a long-term care facility, the nurse is preparing to terminate the interview. Which question is the most appropriate conclusion to the interview? a. "What practices have you found especially helpful in other settings?" b. "Is there anything else we should know in order to care for you better?" c. "What do you envision for your care while you're here at the facility?" d. "What are your expectations from us and from yourself in your care?"

b. "Is there anything else we should know in order to care for you better?"

The health care provider tells the client, "You are experiencing an MI," and leaves the room. The client asks the nurse what an MI stands for. What response by the nurse is most accurate? a. "Mitochondria inflammation." b. "Myocardial infarction." c. "Muscle infection." d. "Myopia instability."

b. "Myocardial infarction."

Which statement by a nurse best indicates an accurate understanding of the different types of assessments? a. "How much time the nurse has and how the client is feeling determine which type of assessment to perform." b. "The purpose for the assessment offers guidance for which type and how much data to collect." c. "The physician informs the nurse of which type of assessment to perform for each client." d. "It is up to the nurse to decide which assessment to perform."

b. "The purpose for the assessment offers guidance for which type and how much data to collect."

The nurse is collecting the health history of a client and notes the client is apprehensive in answering questions. The client states, "My spiritual healer will be here soon." What is the best response by the nurse? a. "I will leave the questionnaire here. Please fill it out when your spiritual healer arrives." b. "We can wait until your spiritual healer arrives and work together to answer these questions." c. "I can wait until your spiritual healer arrives, but you are the only one who can answer these questions." d. "These questions need to be answered so we can provide you with the best care."

b. "We can wait until your spiritual healer arrives and work together to answer these questions."

The nurse calls the health care provider due to changes in the client's status. Using the SBAR, the nurse is about to address Recommendation. Which statement appropriately supports this part of the SBAR? a. "The client's temperature has been 102°F (38.9°C) for the last 6 hours." b. "Will you prescribe a complete blood count to check the white blood cell count and a culture?" c."The client was admitted today with a urinary tract infection." d. "I am concerned that the client might be exhibiting sepsis."

b. "Will you prescribe a complete blood count to check the white blood cell count and a culture?"

Which are examples of subjective data? Select all that apply. a.A nurse observes redness and swelling at an intravenous site. b. A client describes pain as an 8 on the pain assessment scale. c. A client's blood pressure is elevated following physical activity. d. A nurse observes a client wringing the hands before signing a consent for surgery. e. A client feels nauseated after eating breakfast. f. A client reports being cold and requests an extra blanket.

b. A client describes pain as an 8 on the pain assessment scale. e. A client feels nauseated after eating breakfast. f. A client reports being cold and requests an extra blanket.

Which is an example of a subjective finding that the nurse would likely obtain when performing a review of systems (ROS)? a. A blood glucose level of 108 mg/dL b. A client report of shooting pain up the left leg c. Grip weakness in the right hand d. Crackles in bilateral lung bases

b. A client report of shooting pain up the left leg

Which situations observed by a nurse should the nurse report to the nurse manager for quality assurance? Select all that apply. a. A nurse cleans a stethoscope between clients. b. A nurse assesses a client after sneezing into the nurse's hands. c. A nurse administers medications to the wrong client. d. A nurse delays answering call lights to an abusive client. e. A nurse refuses to provide care to a client with HIV.

b. A nurse assesses a client after sneezing into the nurse's hands. c. A nurse administers medications to the wrong client. d. A nurse delays answering call lights to an abusive client. e. A nurse refuses to provide care to a client with HIV.

Which is the purpose of a focused assessment? a.Gives a comprehensive volume of data b.Adds depth to existing information c.Provides breadth for future comparisons d.Suggests possible problems

b. Adds depth to existing information

The nurse is caring for a client who presents with polydipsia, polyphagia, and polyuria. The client's laboratory test results reveal an increased HgbA1C level, which could indicate increased blood glucose levels. What is the next step for the nurse to take based on the nursing process? a. Administer a prescribed medication to decrease the client's blood glucose level. b. Analyze the data and create an individualized nursing diagnosis. c. Follow up with the client later to determine whether the client's laboratory test results improve. d. Identify outcomes for the client with the client's input.

b. Analyze the data and create an individualized nursing diagnosis.

A nurse technician is assigned to take clients' vital signs. When making rounds, the nurse notices that one client's vital signs are very different from what they were at the beginning of the shift. What is most appropriate for the nurse to do about these findings? a. Ask the nurse technician whether the vital signs are correct. b. Assess the client's vital signs again. c. Call the health care practitioner for new orders. d. Document the vital signs in the client's chart.

b. Assess the client's vital signs again.

Which scenario is an example of cultural competence in nursing? a. Assessing the rate at which an illness causes death in a culture b. Attending a conference for cultural diversity c. Attending one's own church d. Assuming the provider and the client share beliefs and values

b. Attending a conference for cultural diversity

For nursing students to be successful in their educational endeavors, they must: a. Come to class having memorized the material in the textbook b. Be actively involved with the material in the text c. Save all questions for the end of the class d. Read the material after the class for understanding

b. Be actively involved with the material in the text

Which statements are true about the implementation phase of the nursing process? Select all that apply. a. All interventions carried out during this phase must be accompanied by a physician's order. b. Care provided during implementation should be documented in the client's chart. c. Implementation is only carried out by nursing professionals. d. Implementation is the process of carrying out the plan of care. e. This phase promotes wellness and restores health.

b. Care provided during implementation should be documented in the client's chart. d. Implementation is the process of carrying out the plan of care. e. This phase promotes wellness and restores health.

A nurse is conducting a cultural assessment of a client. Which person would the nurse identify as the expert? a. Older family member b. Client c. Nurse d. Health care provider

b. Client

Which note includes all elements of a SOAP note? a. Client with nausea, vomiting, diarrhea, most likely secondary to gastroenteritis. Will give an antiemetic and reassess. b. Client reports nausea, including one episode of nausea yesterday. Also with diarrhea. Mucous membranes are moist, good turgor. Blood pressure of 130/85 mm Hg, heart rate of 92 beats/min. Nausea and vomiting of unknown etiology. Will give an antiemetic and reassess within 1 hour for effectiveness. c. Client reports nausea, vomiting, and diarrhea × 3 days. Denies any sick contacts or recent travel. Mucous membranes moist, blood pressure of 130/85 mm Hg, heart rate of 92 beats/min. d. Client reports nausea and vomiting × 3 days. Vital signs stable. Most likely due to gastroenteritis.

b. Client reports nausea, including one episode of nausea yesterday. Also with diarrhea. Mucous membranes are moist, good turgor. Blood pressure of 130/85 mm Hg, heart rate of 92 beats/min. Nausea and vomiting of unknown etiology. Will give an antiemetic and reassess within 1 hour for effectiveness.

A client comes into the clinic for a routine postoperative visit. While the nurse is assessing the level of pain, the client states that there is occasional discomfort but that pain levels have improved daily since returning home from the hospital. What should the nurse's response be regarding the client's plan of care? a. Terminate the plan of care. b. Continue the plan of care. c. Promptly modify the plan of care. d. Suggest increasing the pain medication.

b. Continue the plan of care.

Recording prioritized outcomes in the plan of care ensures which benefit? a. Each nurse can select which priorities to accomplish. b. Continuity of care can be provided to the client. c. The client will reach the goals of the care plan. d. The nurse knows what the client wants.

b. Continuity of care can be provided to the client.

A nurse is reviewing a client's plan of care. What would the nurse determine is a problem related to the assessment phase of the nursing process? a. Nursing diagnoses are too vague. b. Database does not reflect changes in the client condition. c. Plan of care is vague and only contains information that nurses would utilize without a plan of care. d. There is inefficient use of nursing resources.

b. Database does not reflect changes in the client condition.

A nurse overhears a coworker telling a somewhat offensive joke to a client. Which nursing action is indicated? a. Report what was overheard to the charge nurse. b. Discuss the occurrence with the coworker. c. Apologize to the client for the coworker's behavior. d. Investigate whether the coworker and client have a previous relationship.

b. Discuss the occurrence with the coworker.

The client is admitted to the hospital with a ruptured ovarian cyst. The client has expressed that it is very important that the spouse be present to receive all medical information. Using the concepts of culturally competent care, which is the best response? a. Explain to the client that the client is required to make all decisions related to the client's own health care. b. Document the client's request in the nursing care plan. c. Bring the client's spouse into the hallway to discuss surgical options for the client. d. Explain to the client that it is not a good idea to have the spouse in the room when discussing such a private matter.

b. Document the client's request in the nursing care plan.

A client reports hearing voices in the head that tell the client to do bad things. When the nurse enters the client's room, the client is talking out loud to someone but there is nobody in the room. How should the nurse record this assessment? a. Document that the client is talking back to the voices in the client's head. b. Document this assessment based on the client's behaviors. c. Do not document this assessment because the client could be using a wireless device to talk to family. d. Do not document this assessment because it is subjective.

b. Document this assessment based on the client's behaviors.

A nurse is providing care to a client who has limited understanding of the dominant language. Which strategy is best for the nurse to use to ensure that the client obtains the needed health information? a. Ask a bilingual family member to translate b. Enlist the services of a qualified language interpreter c. Ask another nurse who speaks the client's language to interpret d. Use a laboratory aide who is from the same country as the client

b. Enlist the services of a qualified language interpreter

A nurse is caring for a postoperative client after knee arthroplasty. The nurse plans to help the client ambulate but is aware that the client may feel threatened by physical closeness because the client is from a culture that tends to prefer more personal space when interacting with others. Using the principles of culturally competent care, what would be the most appropriate nursing action? a. Let the client ambulate slowly on his or her own when stable. b. Explain the purpose and need for assistance during ambulation. c. Instruct family members to assist in ambulating the client. d. Ambulate the client explaining it is an expected outcome of their treatment.

b. Explain the purpose and need for assistance during ambulation.

What must the nurse do to identify actual or potential health problems? a.Evaluate care implemented b.Gather data from sources c.Call the physician d.Meet with significant others

b. Gather data from sources.

The nurse is caring for a client who perceives time differently. What action should the nurse take for this client? a. Set all interventions to be done at specific times. b. Maintain flexibility when the client requests interventions at specific times. c. Have the client set all times for the interventions. d. Perform interventions at random times during shift.

b. Maintain flexibility when the client requests interventions at specific times.

A client comes to the emergency department with flulike symptoms. The nurse records the vital signs and listens to the client's lung sounds. Vital signs and lung sounds are examples of which type of data? a.Intuitive b.Objective c.Subjective d.Hunches

b. Objective

The nurse is assessing the self-care capabilities of a client who will be discharged from the hospital. Which are barriers this client will likely face in meeting established nursing goals pertaining to self-care? Select all that apply. a. Referrals to specialists b. Poor communication skills c. Inadequate emotional coping skills d. Debilitating illness e. Family's lack of interest in the plan of care

b. Poor communication skills c. Inadequate emotional coping skills d. Debilitating illness e. Family's lack of interest in the plan of care

A nurse is participating as a team member involved in the facility's evaluation process. The facility is conducting a retrospective evaluation. Which methods should the nurse expect to use to collect data? Select all that apply. a. Direct observation of client care b. Post-discharge client questionnaires c. Client interviews during the client's stay d. Chart review e. Telephone interviews of discharged clients

b. Post-discharge client questionnaires d. Chart review e. Telephone interviews of discharged clients

A nurse is evaluating the outcome of the plan of care after teaching a client how to prepare and administer an insulin pen. Which type of outcome is the nurse addressing? a. Cognitive b. Psychomotor c. Affective d. Physiologic

b. Psychomotor

After incorrectly administering digoxin to a client, a nurse admits the error to the nurse manager and peers to prevent them from making the same mistake. This is an example of which approach to quality assurance? a. Quality by inspection b. Quality as opportunity c. Quality by supervision d. Quality as repetition

b. Quality as opportunity

For the second time this week, a nurse reports to the nurse manager failing to perform an ordered dressing change due to a lack of time. The nurse manager recognizes that the nurse normally is very punctual and known to provide good care for clients and that the unit census has been very high this week. However, the nurse manager knows that quality care must be provided and reports this occurrence. Which approach to quality assurance does this scenario represent? a. Quality by inspection b. Quality as opportunity c. Quality by design d. Quality as promotion

b. Quality as opportunity

A client refuses to allow any healthcare worker of Asian descent to provide care. This client is demonstrating what practice? a. Ethnocentrism b. Racism c. Stereotyping d. Ethnic identification

b. Racism

The nurse is caring for an obese client who needs to be turned every 2 hours. Which action by this nurse is an example of reflection-for-action? a. During the first attempt to turn the client, the nurse realizes the need for assistance and calls the front desk for help. b. Reflecting on prior experience and best practice, the nurse includes assistance with turning in the client's plan of care. c. After turning the client alone, the nurse realizes that the nurse should have insisted on having help. d. The nurse decides to turn the client every 4 hours because everyone is too busy to help.

b. Reflecting on prior experience and best practice, the nurse includes assistance with turning in the client's plan of care.

Which term refers to a purposeful activity that leads to action, improvement of practice, and better client outcomes? a. Memorization b. Reflection c. Assessment d. Evaluation

b. Reflection

A geriatric client is observed smoking a cigarette and lowering the oxygen nasal prongs away from the nostrils. Which is the priority action of the nurse? a. Instruct the client to switch to chewing tobacco b. Remind the client to avoid smoking during oxygen therapy c. Accept the client has lung cancer and will die one way or another d. Request a prescription for a smoking cessation program for the client

b. Remind the client to avoid smoking during oxygen therapy

Which action should the nurse take when client data indicate that the stated goals have not been achieved? a. Collect more data for the database. b. Review each preceding step of the nursing process. c. Implement a standardized plan of care. d. Change the nursing orders.

b. Review each preceding step of the nursing process.

The nurse is caring for a 14-year-old client who has just delivered a baby. The client reports living with an aunt and having no other family around. The delivery was uncomplicated and the newborn is healthy. Which would be the primary nursing diagnosis for this client? a.Acute Pain b.Risk for Impaired Parenting c.Ineffective Breastfeeding d.Ineffective Infant Feeding Pattern e.Risk for Loneliness

b. Risk for Impaired Parenting

When recording data regarding the client's health record, the nurse mentions the analysis of the subjective and objective data, in addition to detailing the plan for care of the client. Which of the following styles of documentation is the nurse implementing? a. narrative charting b. SOAP charting c. FOCUS charting d. PIE charting

b. SOAP charting

At the beginning of prenatal care, the goal for the client was to gain 25 lb (11.25 kg) by the end of the pregnancy. At 30 weeks of pregnancy, the client has only gained 1 lb (0.45 kg). Which statemen(s) would help the nurse most appropriately interpret these data? a. It is too early to evaluate if the goal has been achieved. The client has 10 more weeks of pregnancy. b. The client is not achieving the goal. The nurse should determine the reasons the client has not been gaining weight. c. The client is progressing toward achieving the goal. The plan should be continued. d. The client has partially achieved the determined goal. The nurse should revise the goal to reflect a more realistic outcome.

b. The client is not achieving the goal. The nurse should determine the reasons the client has not been gaining weight

A nurse caring for a client with a respiratory condition notices the client's breathing pattern is getting more irregular and the rate has greatly increased from 18 to 32 breaths per minute. The nurse notes that this client's vital signs are assessed once every shift, but believes the assessment should be done more frequently. Who is responsible for increasing the frequency of this client's assessments? a.The nursing supervisor b.The nurse c.The case manager d.The physician

b. The nurse

A nurse is interviewing a new client admitted to the hospital for surgery. Which action would the nurse perform in the introductory phase of the interview? a. The nurse asks the client if there is anything else that needs to be divulged b. The nurse assesses the client's comfort and ability to participate in the interview. c. The nurse recapitulates the interview, highlighting important points. d. The nurse gathers all the information needed to form the subjective database.

b. The nurse assesses the client's comfort and ability to participate in the interview.

Which example may illustrate a breach of confidentiality and security of client information? a. The nurse informs a colleague that she should not be discussing client information in the hospital cafeteria. b. The nurse provides information over the phone to the client's family member who lives in a neighboring state. c. The nurse accesses client information on the computer at the nurses' station, then logs off before answering a client's call bell. d. The nurse provides information to a professional caregiver involved in the care of the client.

b. The nurse provides information over the phone to the client's family member who lives in a neighboring state.

The nurse is comparing a client's current status to baseline data obtained upon admission to long-term care facility 6 months previously. Which tool should the nurse use to make this form of assessment? a.Patient centered assessment method (PCAM) b.Time-lapsed assessment c.Initial assessment d.Emergency assessment

b. Time-lapsed assessment

A new graduate nurse states that it does not make sense to have to perform such an extensive assessment on clients when they are not feeling well. Which response by the nurse preceptor is an appropriate explanation for conducting a comprehensive physical assessment on clients? Select all that apply. a. To keep the nurse focused and with the client b. To appraise the client's health status c. To improve nurses' skills through practice d. To establish a database for nursing interventions e. To identify any health problems

b. To appraise the client's health status. d. To establish a database for nursing interventions e. To identify any health problems

The nurse is admitting a client to the acute care unit with a diagnosis of dehydration. The client's skin turgor is poor and the mucous membranes are pale and dry. What is the rationale for the next phase in the nursing process? a. To decide whether to continue, modify, or terminate client care b. To develop a prioritized list of current and possible health problems c. To develop an individualized plan of client care d. To determine the client's health status, self-care ability, and need for nursing.

b. To develop a prioritized list of current and possible health problems

The nurse is caring for a client who is terminally ill and recently immigrated to the country. The nurse understands that in order to provide quality end-of-life care for the client, what would be the priority action by the nurse? a. Make every effort to involve the client and the client's family with the end-of-life care. b. Understand the client's personal and cultural views regarding death and dying. c. Arrange for end-of-life care to be provided by personnel familiar with the client's culture. d. Share the client's concerns regarding the dying process with the interdisciplinary care team.

b. Understand the client's personal and cultural views regarding death and dying.

While doing an assessment, the nurse identifies questionable data. Which should the nurse do first? a.Inform the client that the data are not correct. b.Validate the questionable data. c.Inform the physician of the questionable data. d.Disregard the questionable data.

b. Validate the questionable data.

A nurse is performing an admission assessment on a client who is scheduled for an elective surgery the next morning. When taking vital signs, the nurse finds that the client's temperature is 39.4°C (103°F). What should be the nurse's priority action? a.Verbally report the finding to the charge nurse at the change of shift. b.Verbally report the finding immediately to the client's physician. c.Inform the unlicensed assistive personnel to document the finding. d.Reassess the client's temperature in 2 hours and chart this data.

b. Verbally report the finding immediately to the client's physician.

A client is admitted to the health care facility with hypoglycemia. After the client is stable, the nurse discovers that the client has not had the prescribed medicines. The client believes that eating saffron will keep blood sugar under control. What is the most appropriate response by the nurse? a. "Saffron does not have any effect on blood sugar level." b. "Why don't you take the medicines, too, and benefit from both?" c. "Yes, I agree that you should continue taking saffron for diabetes." d. "Let me inform the health care provider that you are not taking your medicines."

b. Why don't you take the medicines, too, and benefit from both?"

Which documentation tool will the nurse use to record the client's vital signs every 4 hours? a. a medication record b. a flow sheet c. acuity charting forms d. a 24-hour fluid balance record

b. a flow sheet

The client demonstrates stair climbing using a quad cane. This is an example of: a. a physiologic outcome. b. a psychomotor outcome. c. an affective outcome. d. a cognitive outcome.

b. a psychomotor outcome.

Once a nurse has collected and interpreted the data on a client's outcome achievement, the nurse then makes a judgment and documents a statement summarizing those findings. This statement is called: a. a criterion. b. an evaluative statement. c. a standard. d. evidence-based practice.

b. an evaluative statement.

During the initial assessment of a newly admitted client, the nurse has clustered data as follows: range of motion with gait, bowel sounds with usual elimination pattern, and chest sounds with respiratory rate. The nurse is most likely organizing assessment data according to: a. human needs. b. body systems. c. functional health patterns. d. human response patterns.

b. body systems.

The nurse is planning to do a physical assessment on a newly admitted client. The assessment will be a review of systems. This means the nurse plans to: a. focus on only the systems that the client is comfortable with. b. complete an exam of all body systems. c. perform a review of the problem areas. d. examine certain body systems.

b. complete an exam of all body systems.

A staff nurse has asked the nursing student to perform an intervention that the nursing student has not been educated to perform? what is the appropriate approach for the nursing student to take? a. perform the procedure and inform your instructor of the results b. consult with your nursing instructor before performing the procedure c. delegate the intervention to an unlicensed assistive personnel member d. review the procedure in the procedure manual before performing the intervention

b. consult with your nursing instructor before performing the procedure

The purpose of obtaining a nursing history is to: a. assist the physician to establish a medical diagnosis. b. identify actual and potential health problems. c. focus on objective physical data specific to the client. d. minimize the time required to establish a nursing diagnosis.

b. identify actual and potential health problems.

A nurse on a night shift entered an older adult client's room during a scheduled check and discovered the client on the floor beside the bed, the result of falling when trying to ambulate to the washroom. After assessing the client and assisting into the bed, the nurse has completed an incident report. What is the primary purpose of this particular type of documentation? a. following up the incident with other members of the care team b. identifying risks and ensuring future safety for clients c. protecting the nurse and the hospital from litigation d. gauging the nurse's professional performance over time

b. identifying risks and ensuring future safety for clients

The nurse enters the room of the client diagnosed with a cerebral hemorrhage and immediately states, "This client is getting worse." This is an example of the experienced nurse using: a. acute observation ability. b. intuitive problem identification. c. illogical thinking. d. an assumption to guide practice.

b. intuitive problem identification.

A client newly diagnosed with diabetes has been sent home after in-depth education regarding the diabetes management plan. Because the client is newly diagnosed, the nurse included in the plan of care a risk for unstable glucose. What is the most appropriate short-term outcome for this nursing diagnosis in the client? The client will: a. log all meals in a diary for the next 6 weeks. b. maintain a blood sugar between 70 mg/dL (3.89 mmol/L) and 110 mg/dL (6.11 mmol/L). c. maintain a normal HgbA1C. d. not exhibit signs and symptoms of hypoglycemia/hyperglycemia.

b. maintain a blood sugar between 70 mg/dL (3.89 mmol/L) and 110 mg/dL (6.11 mmol/L).

A nurse is examining alternatives and judging the worth of evidence as part of preparing the plan of care for a client. The nurse would most likely be involved in which phase of the nursing process? a. Diagnosis b. Planning c. Implementation d. Evaluation

b. planning

A 34-year-old client of Asian descent has been hospitalized for the past 3 days with a diagnosis of hepatitis B. The nurse is planning a head-to-toe assessment of the client and understands that the characteristics of an acute hepatitis infection are jaundice, nausea and vomiting, joint pain, rashes, and elevations in serum liver function tests. Where would be the best location for the nurse to observe jaundice in this client? a. neck and chest b. sclera of the eye c. mucous membranes of the mouth d. face and hands

b. sclera of the eye

During the interview component of the health assessment, how does the nurse convey to the client that the information is important? a.Nodding frequently during the interview b.Sitting at eye level with the client c.Limiting questions to those with yes or no answers d. Standing next to the client while interviewing

b.Sitting at eye level with the client

While giving a client a bath, the nurse observes the color of the client's skin as having a yellowish tinge. Which question would the nurse ask the client? a) "How long have you had these spots?" b) "Have you had diarrhea or constipation lately?" c) "Do you use acetaminophen or drink alcohol?" d) "How much caffeine do you drink per day?"

c) "Do you use acetaminophen or drink alcohol?"

A client was recently diagnosed with metastatic lung cancer. The nurse finds the client crying in the room. Which statement made by the nurse best demonstrates the use of empathy? a) "Don't worry, I have seen lots of people with cancer do fine." d) "I am sorry to hear you have cancer. I would be upset too, is there anything I can do?"

c) "I see you are upset. Would you like to talk?"

Which nurse would most likely be the best communicator? a) An advanced practice nurse b) A nurse who is proficient in sign language c) A nurse who easily develops a rapport with clients d) A nurse who is bilingual

c) A nurse who easily develops a rapport with clients

A nurse is planning care for an adult client with severe hearing impairment who uses sign language and lip reading for communication and who has a new diagnosis of cancer. Which nursing action is most appropriate when establishing the plan of care for this client? a) Talk with the client's children to determine needs. b) Consult the oncology nurse specialist. c) Arrange for a sign language interpreter when discussing treatment. d) Use a text-telephone device (TTD) for daily communication..

c) Arrange for a sign language interpreter when discussing treatment.

A client requires a change and reapplication of a colostomy bag. The nurse has never changed an ostomy bag before. What is the nurse's best course of action? a) Ask the client how the bag is changed. b) Read the policy and procedure manual. c) Ask a skilled nurse to assist with the procedure. d) Determine the necessity of the bag change.

c) Ask a skilled nurse to assist with the procedure.

A nurse is completing a health history on a client who has a hearing impairment. Which action should the nurse take first to enhance communication? a) Use facial and hand gestures b) Contact a person skilled in sign language c) Assess how the client would like to communicate d) Provide paper and pencil for written communication

c) Assess how the client would like to communicate

The nurse is palpating the skin of a 30-year old client and documents that when picked up in a fold, the skin fold slowly returns to normal. What would be the next action of the nurse based on this finding? a) Document a normal skin finding on the client chart. b) Assess the client for cardiovascular disorders. c) Assess the client for dehydration. d) Report the finding as a positive sign for cystic fibrosis.

c) Assess the client for dehydration.

The nurse is preparing to administer a blood pressure medication to a client. To ensure the client's safety, what is the priority action for the nurse to take? a) Tell the client to report any side effects experienced. b) Ask the client to verbalize the purpose of the medication. c) Assess the client's blood pressure to determine if the medication is indicated. d) Determine the client's reaction to the medication in the past

c) Assess the client's blood pressure to determine if the medication is indicated.

The home health nurse caring for a client with limited eyesight notes that the client's route to the bathroom is cluttered. What is the most effective way for the nurse to ensure the client's long-term safety? a) Remove all the cluttered objects from the pathway to the client's bathroom. b) Instruct the client about the need to keep the walkway to the bathroom clear. c) Assist the client to identify strategies to promote safety in the home. d) Assign a home health aide to perform housekeeping duties.

c) Assist the client to identify strategies to promote safety in the home.

The nursing is caring for several clients. Which intervention can the nurse direct the unlicensed assistive personnel (UAP) to perform? a) Take the vital signs of the client who just returned from surgery. b) Feed a client who is eating for the first time following an ischemic stroke. c) Bathe a client with stable angina who has a continuous IV infusing. d) Assist the client who is ambulating the first time since hip replacement surgery.

c) Bathe a client with stable angina who has a continuous IV infusing.

A nurse and an older adult client with chronic back pain are beginning to communicate. What activity should the nurse focus on at this point? a) Sharing the nurse's own family and personal history of back pain b) Reassuring the client that back surgery will likely alleviate the pain completely c) Being sensitive to the client's emotional barriers d) Explaining in detail all of the pain management options available

c) Being sensitive to the client's emotional barriers

Which parties are essential for the nurse to include in the implementation of a client's plan of care? a) Client, surgeon, and physician b) Client, physical therapist, and nursing staff c) Client, family, and physician d) Client, physician, and hospital director

c) Client, family, and physician

The nurse is caring for a client who is recovering from a cerebrovascular accident. When reviewing the client's orders, the nurse notes that one of the physicians wrote orders to ambulate the client, whereas another physician ordered strict bed rest for the client. How would the nurse most appropriately remedy this conflict? a) Collaborate with the physical therapist to determine the client's ability b) Assess the client to determine whether the client is capable of ambulation c) Communicate with the physicians to coordinate their orders d) Instruct the client to ask the physicians for clarifications of instructions.

c) Communicate with the physicians to coordinate their orders

A client has terminal cancer and the primary care provider has ordered a diagnostic imaging test. The client does not want the test performed so the nurse agrees to dialogue with the primary care provider on the client's behalf. The nurse's actions are what type of intervention? a) Surveillance b) Supportive c) Coordinating d) Technical

c) Coordinating

A grating feel and noise with joint movement, particularly in the temporomandibular joint, is called what? a) Inflammation b) Fremitus c) Crepitus d) Arthritis

c) Crepitus

The nurse detects a weak, thready pulse found from a client palpating peripheral pulses. What condition does the nurse suspect the client is experiencing? a) Inflammation of a vein b) Hypertension and circulatory overload c) Decreased cardiac output d) Impaired kidney function

c) Decreased cardiac output

The nurse is preparing a client for surgery when the client tells the nurse that the client no longer wants to have the surgery. How should the nurse most appropriately respond? a) Notify the physician of the client's refusal. b) Ask the client to discuss the decision with family members. c) Discuss with the client the reasons for declining surgery. d) Review with the client the risks and benefits of surgery.

c) Discuss with the client the reasons for declining surgery.

The client is an 18-month-old in the pediatric intensive care unit. The client is scheduled to have a subgaleal shunt placed tomorrow, and the client's mother is quite nervous about the procedure. The nurse tells the client's mother, "The surgeon has done this a million times. Your son will be fine." This is an example of what type of nontherapeutic communication? a) Giving advice b) Being moralistic c) False reassurance d) Rescue feelings

c) False reassurance

A nurse is administering metformin to a client who has a new onset of diabetes mellitus type 2. Which step should the nurse consider a priority on the nursing care plan? a) Restrict intake of foods and fluids. b) Monitor for noncompliance. c) Monitor for lactic acidosis d) Administer B12 injections

c) Monitor for lactic acidosis

A client has been reporting persistent headaches. Which is an example of subjective data? a) The client is alert and oriented to person, place, and time. b) Temperature is 104.1°F (40.05°C) c) Pain is 4 out of 10 on a pain scale. d) The client appears lethargic.

c) Pain is 4 out of 10 on a pain scale.

A nurse is inspecting the external genitalia of a female client. Which assessment finding is of the most concern? a) Dark pink vulva b) Whitish vaginal discharge c) Pink labia lesions d) Coarse brown hair

c) Pink labia lesions

A nurse who has been caring for a client for the past few days is preparing the client for discharge and termination of the nurse-client relationship. Which activity would the nurse be carrying out during this phase of the relationship? a) Attending to physical health care needs b) Developing solutions that will be enacted c) Reviewing health changes d) Establishing trust and rapport

c) Reviewing health changes

Nursing interventions for the client after prostate surgery include assisting the client to ambulate to the bathroom. The nurse concludes that the client no longer requires assistance. What is the nurse's best action? a) Consult with the physical therapist to determine the client's ability. b) Continue assisting the client to the bathroom to ensure the client's safety. c) Revise the care plan to allow the client to ambulate to the bathroom independently. d) Instruct the client's family to assist the client to ambulate to the bathroom.

c) Revise the care plan to allow the client to ambulate to the bathroom independently.

The nurse is discussing diabetes mellitus with the family members of a client recently diagnosed. To promote the health of the family members, what would be the most important information for the nurse to include? a) The cellular metabolism of glucose b) The severity of the client's disease c) Risk factors for and prevention of diabetes mellitus d) Medications used to treat diabetes mellitus

c) Risk factors for and prevention of diabetes mellitus

The Joint Commission (TJC) encourages clients to become active, involved, and informed participants on the health care team. What nursing action follows TJC recommendations for improving client safety by encouraging them to speak up? a) The nurse encourages clients to advocate for themselves instead of choosing a trusted family member or friend. b) The nurse assures the client who questions a medication that it is the right medication prescribed for him or her and administers the medicine. c) The nurse encourages the client to participate in all treatment decisions as the center of the health care team. d) The nurse explains each procedure twice to prevent client questions from wasting time.

c) The nurse encourages the client to participate in all treatment decisions as the center of the health care team.

Which nursing actions would most likely help improve communication with clients and achieve a more effective helping relationship? Select all that apply. a) The nurse never admits a lack of knowledge to the client to avoid undermining the client's confidence in the helping relationship. b) The nurse feels free to use words that might have different interpretations when using the same language as the client. c) The nurse makes statements that are as simple as possible, gearing conversation to the client's level. d) The nurse controls the tone of voice so that it conveys exactly what is meant. e) The nurse remains focused on the topic at hand and does not allow the client to diverge to another topic. f) The nurse takes advantage of any available opportunities to communicate information to clients in routine caregiving situations.

c) The nurse makes statements that are as simple as possible, gearing conversation to the client's level. d) The nurse controls the tone of voice so that it conveys exactly what is meant. f) The nurse takes advantage of any available opportunities to communicate information to clients in routine caregiving situations.

Which best describes an element of the nurse-client relationship? The nurse and client have a conversation for mutual companionship, enjoyment and interaction. a) The nurse and client have a conversation with the goal of forming b) a more intimate relationship. c) The nurse self-discloses only what is necessary for the client's benefit. d) The client shares life events and activities for the nurse's benefit.

c) The nurse self-discloses only what is necessary for the client's benefit.

A nurse who is experienced caring only for well babies is assigned to the neonatal intensive care unit (NICU) because of a shortage of nurses in the NICU. The nurse is assigned to an infant on a ventilator who will require blood transfusions during the shift. What is the nurse's most appropriate course of action? a) The nurse should ask another nurse who was previously assigned to the client for instruction. b) The nurse should request that the blood transfusions be delayed until the next shift. c) The nurse should inform the charge nurse that the nurse does not have the experience to properly care for this client. d) The nurse should recognize the necessity of the assignment and provide care to the best of the nurse's ability.

c) The nurse should inform the charge nurse that the nurse does not have the experience to properly care for this client.

Which statement accurately describes the concept of feedback as it pertains to the process of communication? a) The sender sends a clear message that is understood by the receiver. b) The receiver listens to the sender in an unassuming way. c) The sender and the receiver use one another's reactions to produce further messages. d) The sender's message is translated into a code, using verbal and nonverbal communication.

c) The sender and the receiver use one another's reactions to produce further messages.

A dialysis nurse is educating a client on caring for the dialysis access that was inserted into the client's right arm. The nurse assesses the client's fears and concerns related to dialysis, the dialysis access, and care of the access. This information is taught over several sessions during the course of the client's hospitalization. Which phase of the working relationship is bestdescribed in this scenario? a) The orientation phase b) The introduction phase c) The working phase d) The termination phase

c) The working phase

A nurse is preparing to auscultate a client's abdomen for the presence of bowel sounds. Which is the appropriate action of the nurse? a) Uncover the client to expose the chest and abdomen. b) Assist the client to a sitting position. c) Warm the diaphragm of the stethoscope. d) Palpate the abdomen before auscultating.

c) Warm the diaphragm of the stethoscope.

The Glasgow Coma Scale is a standardized assessment tool for a person's level of consciousness. Which client would this scale not be appropriate for? a) a client in the Intensive Care Unit after having a stroke yesterday b) a client with a brain tumor who is in the hospital because of respiratory depression c) a client in the Intensive Care Unit for acute pancreatitis asking for pain medications d) a client recovering from brain surgery for repair of an aneurysm

c) a client in the Intensive Care Unit for acute pancreatitis asking for pain medications

A nurse touches the client's hand while discussing the client's diagnosis. This action is: a) a dynamic process. b) a translation. c) a communication channel. d) an auditory channel.

c) a communication channel.

Each of the following facilitates a therapeutic nurse-client relationship except: a) active listening. b) rephrasing. c) closed-ended questions. d) reflection.

c) closed-ended questions.

Which of the following are examples of nonprofessional involvement? Select all that apply. a) asking a patient in hospice care to describe their relationship with various family members b) discussing today's weather forecast c) discussing your recent breakup with your boyfriend with a patient who is also going through a difficult breakup d) asking a patient if they would like to go out for dinner after they are discharged

c) discussing your recent breakup with your boyfriend with a patient who is also going through a difficult breakup d) asking a patient if they would like to go out for dinner after they are discharged

A unit-based infection control task force was developed in an attempt to reduce catheter-acquired infections. The group consists of 10 team members. During the past three meetings, one person dominated the meeting and did not allow other members ample time to speak. The best way to address the team dysfunction is to: a) pick a team leader who is not the dominant member. b) have group members issue a written warning to the dominant member. c) have group members confront the dominant member to promote the needed team work. d) plan a meeting where the dominant person cannot attend.

c) have group members confront the dominant member to promote the needed team work.

The primary purpose of nursing implementation is to: a) improve the client's postoperative status. b) identify a need for collaborative consults. c) help the client achieve optimal levels of health. d) implement the critical pathway for the client.

c) help the client achieve optimal levels of health.

The term metacommunication is best defined as: a) contextual factors that impede communication patterns. b) congruent relationships in the spoken topics. c) interpersonal bridge between verbal and nonverbal communication. d) documenting a conversation between the client and nurse.

c) interpersonal bridge between verbal and nonverbal communication.

To assess an adult client's hearing, the nurse performs the Rinne test by activating the tuning fork and placing it first at the: a) top of the head. b) affected ear. c) mastoid process. d) front of the ear.

c) mastoid process.

An experienced nurse has been working with a client with heart failure. The client's lungs were clear to auscultation during the morning assessment; however, the afternoon assessment revealed bibasilar crackles and tachypnea. The nurse calls to give SBAR report to the covering health care provider. In the final step of the report the nurse should: a) discuss the client's situation and request a chest x-ray to assess lung function. b) provide detailed findings of the head-to-toe assessment. c) recommend 40 mg of furosemide be administered because the client had improvement with past administration. d) detail the client's past medical history and active medication orders.

c) recommend 40 mg of furosemide be administered because the client had improvement with past administration.

A client made a formal request to review his or her medical records. With review, the client believes there are errors within the medical record. What is the most appropriate nursing response? a. "According to HIPAA, medical records cannot be changed." b. "HIPAA legislation only allows access to review the medical record." c. "According to HIPAA legislation, you have a right to request changes to inaccurate information." d. "HIPAA legislation allows for you to change any information."

c. "According to HIPAA legislation, you have a right to request changes to inaccurate information."

A nurse manager is discussing a nurse's social media post about an interesting client situation. The nurse states, "I didn't violate client privacy because I didn't use the client's name." What response by the nurse manager is most appropriate? a. "All aspects of clinical practice are confidential and should not be discussed." b. "The information being posted on social media is inappropriate. Make sure to discuss information about clients privately with friends and family." c. "Any information that can identify a person is considered a breach of client privacy." d. "You may continue to post about a client, as long as you do not use the client's name."

c. "Any information that can identify a person is considered a breach of client privacy."

The nurse is caring for a client who is admitted for hypertension (HTN). The nurse notes that the client has not been eating the food provided, and family members have brought in homemade food. What would be the best response by the nurse? a. "You should consider eating the food provided, which is healthier." b. "Do you understand the specific diet for your HTN?" c. "Can you tell me what foods you prefer to eat and what your family is bringing you?" d. "It is ok to eat what your family brought you, as long as we see what it is."

c. "Can you tell me what foods you prefer to eat and what your family is bringing you?"

The nurse is teaching a black client about common health conditions. Which statement by the client most directly addresses a health problem with an increased incidence in this population group? a. "I need to watch the amount of sugar that I eat." b. "It is important to monitor my blood pressure." c. "Getting a mammogram in my thirties is important." d. "Increasing dairy will improve my bones."

c. "It is important to monitor my blood pressure."

The staff nurse overhears the charge nurse, who is of Italian heritage, talking to the unlicensed assistive personnel. Which statement made by the charge nurse is an example of ethnocentrism? a. "People who are Irish are usually alcoholics." b. "Asian people are bad drivers." c. "Italians are best at everything." d. "Hispanics are usually lazy."

c. "Italians are best at everything."

Which statement by the nurse would indicate to the charge nurse that there is need for further teaching on the purposes of medical records? a. "I can share the clients' medical records with the health care team." b. "The clients' medical records provide data for legal evidence." c. "The clients' medical records are an obstruction to research and education." d. "The clients' health records should be used to promote reimbursement from insurance companies"

c. "The clients' medical records are an obstruction to research and education."

The nurse is assessing a 1-year-old baby. The mother states, "I'm not sure if he has a fever. I have such a hard time with my glass thermometer. It's so hard to read." The nurse's best response would be: a. "Yes, they are hard to read, but with practice, it gets easier." b. "Why would you use that for your baby?" c. "There is some danger in using a glass thermometer and the mercury it contains. You might consider buying a new type of device." d. "Bring it with you next time and I will teach you the proper way to use your thermometer."

c. "There is some danger in using a glass thermometer and the mercury it contains. You might consider buying a new type of device."

The nurse is interviewing a client to obtain the health history. Which question would the nurse ask first? a. Are you having any pain?" b. "What medications do you normally use?" c. "What brings you here today?" d. "Do you have any allergies?"

c. "What brings you here today?"

A mother brings her child into the clinic for follow up after beginning treatment for attention deficit hyperactivity Disorder (ADHD). One of the outcomes was for the child to complete homework within a 1-hour time interval. The mother reports that it still takes 1 1/2 hours but that is dramatically reduced from the 3 hours or more before beginning treatment. What is the best response for the nurse to make to the child? a. "That's okay. I will just change the plan of care." b. "You will have to do better next time." c. "You have done a great job by focusing on your homework and doing it in much less time. Do you think by your next visit that you can get it down to an hour?" d. "Mom, can you sit with him to make sure he meets the outcome?"

c. "You have done a great job by focusing on your homework and doing it in much less time. Do you think by your next visit that you can get it down to an hour?"

A nurse caring for clients of different cultures in a hospital setting attempts to make eye contact with clients when performing the initial assessment. What assumption might the nurse make based on common cultural practices? a. A Native American/First Nations man stares at the floor while talking with the nurse. Assumption: The client is embarrassed by the conversation. b. A Hasidic Jewish man listens intently to a male physician, making direct eye contact with him, but refuses to make eye contact with a female nursing student. Assumption: Jewish men consider women inferior to men. c. A Muslim-Arab woman refuses to make eye contact with her male nurse. Assumption: She is being modest. d. A Black man rolls his eyes when asked how he copes with stress in the workplace. Assumption: He may feel he has already answered this question and has become impatient.

c. A Muslim-Arab woman refuses to make eye contact with her male nurse. Assumption: She is being modest.

Which clinical situation is addressed by the provisions of the Health Insurance Portability and Accountability Act (HIPAA)? a. A client wishes to appeal her insurance company's refusal to reimburse for a diagnostic test. b. A client has asked for a second opinion regarding treatment options for her diagnosis of ovarian cancer. c. A client has asked a nurse if he can read the documentation that his physician wrote in his chart. d. A client who resides in Indiana has required hospitalization during a vacation in Hawaii.

c. A client has asked a nurse if he can read the documentation that his physician wrote in his chart.

A community health nurse is providing care to a group of Hispanic people living in an area that is predominantly populated by white people. What are the Hispanic people in this community an example of? a. A subculture b. A subgroup c. A minority d. A majority

c. A minority

Which scenario is an example of a time-lapse reassessment? a. A nurse in a long-term skilled nursing facility assesses a new resident's baseline health status. b. Seeing a client down on the floor, the nurse assesses the client's airway, breathing, and circulation, calls for help, and begins a quick neurological exam. c. A nurse assesses a client with mobility issues to see how the client is doing with fall prevention strategies they practiced before. d. A nurse just coming on shift performs a focused physical assessment on each client, based on the client's diagnosis.

c. A nurse assesses a client with mobility issues to see how the client is doing with fall prevention strategies they practiced before.

A nurse is following a clinical pathway that guides the care of a client after knee surgery. When the nurse observes the client vomiting, it creates a deviation from the clinical pathway. What should the nurse identify this event as? a. An audit b. A never event c. A variance d. A sentinel event

c. A variance

A client has been diagnosed with pneumonia and is experiencing chest pain when taking a deep breath. What are the priority nursing diagnoses? Select all that apply. a. Risk for Infection b. Anxiety c. Acute Pain d. Ineffective Airway Clearance e. Feeding Self-Care Deficit

c. Acute Pain d. Ineffective Airway Clearance

Which client situation most likely warrants a time-lapse nursing assessment? a. A client is being admitted to a general medicine unit after spending several days in the intensive care unit. b. A nurse is auscultating the lungs and measuring the oxygen saturation of a client who has pulmonary edema. c. An older adult resident of an extended-care facility is being assessed by a nurse practitioner during the nurse's scheduled monthly visit. d. The nurse has responded to the call light of a hospital client who is reporting shortness of breath and chest pain.

c. An older adult resident of an extended-care facility is being assessed by a nurse practitioner during the nurse's scheduled monthly visit.

What would be a nursing priority when assessing a client who weighs 250 lb (112.50 kg) and stands 5 ft, 3 in (1.58 m) tall? a. Obtain an electrocardiogram daily. b. Assess cholesterol levels. c. Assess blood pressure with a large cuff. d. Begin client education regarding a low-fat diet.

c. Assess blood pressure with a large cuff.

Which step in the nursing process includes the careful taking of a history and a nursing examination? a. Planning b. Nursing diagnosis c. Assessment d. Implementation

c. Assessment

Nurses understand the problem that clients have when they are repeatedly asked the same questions. To best avoid this problem, which intervention should nurses perform when beginning to collect assessment data? a. Tell the client the questions will be quick. b. Organize all questions into categories. c. Carefully review the client's record. d. Make the questions short.

c. Carefully review the client's record.

A nurse caring for an older adult client who has dementia observes another nurse putting restraints on the client without a physician's order. The client is agitated and not cooperating. What would be the best initial action of the first nurse in this situation? a. File an incident report and have the second nurse sign it. b. Contact the physician for an order for the restraints. c. Confront the nurse and explain how this could be dangerous for the client. d. Report the nurse applying the restraints to the supervisor.

c. Confront the nurse and explain how this could be dangerous for the client.

A nurse who recently graduated is performing an assessment on a client who was admitted for nausea and vomiting. During the assessment, the client reports mild chest pain. The nurse does not know whether the chest pain is related to the gastrointestinal symptoms or should be reported to the physician. Which action should the nurse perform next? a. Wait and see whether the pain subsides. b. Chart the information. c. Consult with another nurse. d. Call the family.

c. Consult with another nurse.

Which part of the client record should the nurse review to find recommendations made by a gastrointestinal specialist? a. Medical history b. Laboratory reports c. Consultation d. Progress notes

c. Consultation

The nurse is caring for the teenage child of immigrants. The teenager voices distress because after living in the country for several years, he no longer wants to participate in some of the tradition religious rituals that are important to his parents. What is the teenager experiencing? a. Culture shock b. Cultural ethnocentrism c. Cultural change d. Cultural relativity

c. Cultural change

A client is admitted with end-stage pancreatic cancer and is experiencing extreme pain. The client asks the nurse whether an acupuncturist can come to the hospital to help manage the pain. The nurse states, "You won't need acupuncture. We have pain medications." Which characteristic has the nurse displayed? a. Stereotyping b. Cultural conflict c. Cultural imposition d. Culture shock

c. Cultural imposition

Which term describes the tendency to impose one's cultural beliefs, values, and patterns of behavior on a person from a different culture? a. Acculturation b. Cultural blindness c. Cultural imposition d. Cultural taboos

c. Cultural imposition

The nurse is caring for a client who is postoperative 3 days from coronary artery bypass graft. The client has a prescription to ambulate. What is the best action by the nurse? a. Allow the client to slowly ambulate independently. b. Instruct the family to assist the client with ambulation. c. Discuss with the client the need for assistance during ambulation. d. Obtain a prescription for physical therapy consult to ambulate the client.

c. Discuss with the client the need for assistance during ambulation.

A nurse administers medications to a client. Which step of the nursing process would the nurse perform next? a. Assessing b. Diagnosing c. Evaluating d. Planning

c. Evaluating

A woman has delivered a healthy newborn and is scheduled to go home today, her third post-partum day. Her vital signs are stable. How often would the nurse expect to take the vital signs of a stable in-patient? a. Every 12 hours b. Every 2 hours c. Every 8 hours d. No further vitals are needed e. Every 1 to 2 hours

c. Every 8 hours

Which type of assessment would the nurse be expected to perform on the client who is 1 day postoperative following a cholecystectomy? a. Initial b. Time-lapse c. Focused d. Emergency

c. Focused

A nurse has completed a client assessment and is preparing to identify appropriate nursing diagnoses. Which areas would the nurse likely address in the diagnosis? Select all that apply. a. Heart failure b. Pneunomia c. Impaired mobility d. Imbalanced nutrition e. Ineffective coping

c. Impaired mobility d. Imbalanced nutrition e. Ineffective coping

The health care provider is in a hurry to leave the unit and tells the nurse to give morphine 2 mg IV every 4 hours as needed for pain. What action by the nurse is appropriate? a. Write the order in the client's record. b. Add the new order to the medication administration record. c. Inform the health care provider that a written order is needed. d. Call the pharmacy to have the order entered in the electronic record.

c. Inform the health care provider that a written order is needed.

The nurse is calling a health care provider to give an update on a client's condition. The nurse receives a telephone order and, when requests that the order be read back to the provider for confirmation, the provider states, "I don't have time for this." What is the most appropriate action by the nurse? a. Proceed with the order since the nurse heard it the first time b. Ask the secretary to call the provider back and take the order c. Inform the provider, to ensure safety for the client, it must be read back d. Don't follow through with the order, and delete it from the record

c. Inform the provider, to ensure safety for the client, it must be read back

Which is the most appropriate reason for a nurse to ask a client what the client would like to be called? a. It ignores the policies of the facility. b. It allows the client to control the situation. c. It communicates respect for the client. d. It signifies that the nurse wants to be friendly.

c. It communicates respect for the client.

A concise document that provides most of the client's nursing and medical information is a(n): a. past chart. b. office record. c. Kardex. d. nursing care plan.

c. Kardex.

The nurse is caring for a client with a nursing diagnosis of deficient fluid volume. The nurse has implemented the plan of care and on evaluation finds that the client continues to exhibit symptoms of deficient fluid volume. What should the nurse do next? a. Develop an additional nursing diagnosis to meet the client's health needs. b. Change the nursing diagnosis because the client's problem was falsely identified. c. Modify the plan of care and interventions to meet the client's needs. d. Reassess the client for more symptoms of deficient fluid volume.

c. Modify the plan of care and interventions to meet the client's needs.

While evaluating a client's plan of care, the nurse would be most likely identify which problem as being associated with the implementation phase? a. Nursing orders are superficial. b. Outcomes are incorrectly developed. c. Nurses are not aware of client priorities and the plan of care. d. A long-term goal is vague and generalized.

c. Nurses are not aware of client priorities and the plan of care.

The nurse documents that a client does not have pain prior to the administration of pain medication. The client, however, requested medication for increasing postsurgical pain. What is the appropriate action to correct the pain assessment documented in the client's paper medical record? a. Write over the entry in another color pen. b. Scribble through the entry. c. Place one line through the entry and initial it. d. Obtain white-out to cover the entry.

c. Place one line through the entry and initial it.

Which actions should the nurse perform to limit casual access to the identity of clients? Select all that apply. a. Making the names of clients on charts visible to the public b. Posting information linking a client with diagnosis, treatment, and procedure on whiteboards c. Placing fax machines, filing cabinets, and medical records in areas that are off-limits to the public d. Obscuring identifiable names of clients and private information about clients on clipboards e. Keeping record of people who have access to clients' records

c. Placing fax machines, filing cabinets, and medical records in areas that are off-limits to the public d. Obscuring identifiable names of clients and private information about clients on clipboards e. Keeping record of people who have access to clients' records

The nurse is assessing a client with vascular dementia. As a result of this cognitive deficit, the client is unable to provide many of the data that are required. How should the nurse bestproceed with this assessment? a. Perform the assessment in several short episodes rather than at one sitting. b. Obtain the client's records from admissions to other institutions. c. Supplement the client's information by speaking with family or friends. d. Limit the assessment to objective data.

c. Supplement the client's information by speaking with family or friends.

A client presents to the clinic for a routine postoperative visit. The nurse assesses the site of the incision and determines that the edges of the incision are approximated, sutures have been removed, and there is no redness or edema at the site. The incision appears to be well healed. The nurse reviews the plan of care and notes that one nursing diagnosis is related to potential infection related to impaired skin integrity. The nurse determines that this is no longer an issue for the client. Which change should the nurse make to the plan of care? a. Terminate the plan of care. b. Modify the plan of care related to infection. c. Terminate the plan of care as it relates to infection. d. Continue the plan of care.

c. Terminate the plan of care as it relates to infection.

Which client outcomes are psychomotor outcomes? Select all that apply. a. The client identifies five low-sodium foods. b. The client describes how to empty a Jackson-Pratt drain. c. The client measures capillary blood glucose level. d. The client self-catheterizes using clean technique. e. The client reports imagery is effective in controlling anxiety.

c. The client measures capillary blood glucose level. d. d. The client self-catheterizes using clean technique.

The nurse is using an interpreter to communicate with a client who speaks a different language. What would be the best way to choose an interpreter for this client? a. The interpreter should speak in a loud voice. b. The interpreter should conduct the conversation quickly to avoid misinterpretation. c.The interpreter should understand the health care system. d. The interpreter should always make direct eye contact.

c. The interpreter should understand the health care system.

While assisting a client with a delivery, a nurse takes a photo of the newborn and posts it on a social media website. What action may occur related to this privacy violation? a. The nurse could be fired but would not face criminal charges or jail time. b. No action will be taken as long as the parents don't find out. c. The nurse could be fined or even go to jail for violating HIPAA. d. There will be no repercussions if the nurse takes the photo down from the social media page.

c. The nurse could be fined or even go to jail for violating HIPAA.

The nurse is finding it difficult to plan and implement care for a client and decides to have a nursing care conference. What action would the nurse take to facilitate this process? a. The nurse, along with other nurses, visits clients with similar problems individually at each client's bedside in order to plan nursing care. b. The nurse consults with someone in order to exchange ideas or seek information, advice, or instructions. c. The nurse meets with nurses or other health care professionals to discuss some aspect of client care. d. The nurse sends or directs someone to take action in a specific nursing care problem.

c. The nurse meets with nurses or other health care professionals to discuss some aspect of client care.

Which is not a purpose of the client care record? a. To facilitate reimbursement b. To serve as a legal document c. To serve as a contract with the client d. To assist with care planning

c. To serve as a contract with the client

The health care team has convened to discuss the care of an end-of-life client who is not able to achieve an acceptable level of comfort. The physician asks for the nurse's perspective of the situation. Which standard for establishing and sustaining healthy work environments does this action represent? a. Skilled communication b. Effective decision making c. True collaboration d. Appropriate staffing

c. True collaboration

A nurse is assigned to care for a client who does not speak the dominant language. An interpreter has been contacted and will be at the bedside shortly. Which action by the nurse would be mosteffective in reassuring the client until the interpreter arrives? a. Using gestures and symbols to communicate with the client b. Enlisting other healthcare providers to assist with communication c. Using reassuring body language and making eye contact to assess needs d. Avoiding going into the client's room until the interpreter arrives

c. Using reassuring body language and making eye contact to assess needs

Which strategy would provide the most effective form of change of shift report? a. Discussing the client's visitors and complaints during the prior shift. b. Providing the oncoming nurse the client's clipboard prior to leaving the unit. c. Utilizing a reporting form and allowing time for any questions. d. Recording the report for the oncoming shift prior to leaving the unit.

c. Utilizing a reporting form and allowing time for any questions.

A nurse is arranging for home care for clients and reviews the Medicare reimbursement requirements. Which client meets one of these requirements? a. a client whose status is stabilized b. a client whose rehabilitation potential is not good c. a client who is homebound and needs skilled nursing care d. a client who is not making progress in expected outcomes of care

c. a client who is homebound and needs skilled nursing care

The client identifies three strategies for minimizing leakage of an ileostomy bag. This is an example of: a. an affective outcome. b. a psychomotor outcome. c. a cognitive outcome. d. a physiologic outcome.

c. a cognitive outcome.

The nurse is caring for a client with hypertension, and only documents a blood pressure of 170/100 mmHg when all other vital signs are normal. This reflects what type of documentation? a. SOAP b. focus c. charting by exception d. narrative

c. charting by exception

Nurses are responsible for delivering culturally competent care for all clients. Culturally competent care does not account for: a. individual values. b. developmental level. c. client's height. d. available technology.

c. client's height.

A new mother is having difficulty breastfeeding a newborn infant. A goal was established stating that the baby would be nursing every 2 to 3 hours by age 1 week. The mother presents to the follow-up center at 1 week and reports having discontinued breastfeeding. The nurse evaluates the original goal as: a. met b. partially met c. completely met d. inappropriately chosen for this client

c. completely unmet

The nurse is caring for a client who just informed her that he noticed some blood in the toilet after a bowel movement. The nurse assesses the client's anal area and notes a deep linear separation in the skin that extends into the dermis. The nurse recognizes that this skin lesion is characteristic of which kind of lesion? a. ulcer. b. erosion. c. fissure. d. crust.

c. fissure

Persistent gaps between the health status of minorities and non-minorities are defined as: a.racism. b. ethnocentrism. c. health disparities. d. cultural relativity.

c. health disparities.

During the introductory phase of interviewing a client for the purpose of obtaining information for the nursing history, the nurse should: a. inform the client of the maintenance of confidentiality. b. assess personal feelings regarding similar clinical situations. c. implement supportive nursing interventions. d. review literature pertinent to the client's attributes.

c. inform the client of the maintenance of confidentiality.

The nurse is assessing the client's behavioral response to a nursing intervention. This type of evaluation is known as: a. structural evaluation. b. behavior modification. c. outcome evaluation. d. process evaluation.

c. outcome evaluation.

Prior to the first visit following gastrectomy, the client will have a weight loss of 10 lb (4.5 kg). This is an example of which type of evaluative statement? a. cognitive b. psychomotor c. physical changes d. affective

c. physical changes

The nursing instructor is teaching students about assessment and the importance of having baseline data when caring for clients. The instructor should inform the students that the best place to get baseline data is: a. the health record from a previous admission. b. the client record from the physician's office. c. the initial comprehensive client assessment. d. the focus assessment done when admitted to the ER.

c. the initial comprehensive client assessment.

Select the best description of how the nurse applies the nursing process in caring for clients. The nurse: a. applies intuition and routine care for clients. b. employs communication to meet the client's needs. c. uses critical thinking to direct care for the individual client. d. uses scientific problem solving to meet client problems.

c. uses critical thinking to direct care for the individual client.

A client's record can be more accurate if the nurse: a. summarizes client care at the end of the shift. b. charts at least every 6 hours. c. uses point-of-care documentation. d. delegates charting appropriately.

c. uses point-of-care documentation.

The nurse is gathering subjective data from a client during an interview after a suicide attempt. Which assessment data gathered by the nurse would be documented as subjective data? Select all that apply. a.Blood pressure 140/82 mm Hg b.Ecchymosis on upper left arm c.Client states, "I am in pain." d.Clothes unkempt and hair greasy e.Client states, "I feel so sad all of the time."

c.Client states, "I am in pain." e. Client states, "I feel so sad all of the time."

Which pulse site is generally used in emergency situations?

carotid

Cues that all relate to the same client problem may be grouped together in a process known as: categorizing. diagnosing. grouping. clustering.

clustering. Cue clustering brings together cues that if viewed separately would not convey the same meaning. The cues are not being categorized or diagnosed. Grouping is not proper terminology.

A nurse is collecting a health history on a client. When asked about alcohol, tobacco, and drug use, the client states, "I quit smoking 10 years ago." However, the nurse observes an open package of cigarettes in the client's shirt pocket. What is the most appropriate response by the nurse? a) "Why did you tell me you quit smoking?" b) "I know that you are lying about not smoking, so tell me how much you smoke each day." c) "You said that you do not smoke, but you have an open package of cigarettes in your pocket." d) "Are you having difficulty quitting smoking?"

d) "Are you having difficulty quitting smoking?"

A pregnant client presents to the emergency department with vaginal bleeding. A transvaginal ultrasound is performed, and the health care provider informs the client that there are normal fetal heart tones noted. The client begins to tear-up and has a worried appearance. To facilitate therapeutic communication, what statement would the nurse make after observing the client's nonverbal communication? a) "This is great news. You don't have anything to worry about and the baby is doing well." b) "Take your time and tell me how you are feeling. I have plenty of time to answer your questions and discuss any thoughts or feelings with you." c) "I can help you, please talk to me so that I know how I can help you." d) "Close your eyes and take a deep breath. I know you were frightened, but the baby is healthy and everything is going to be okay."

d) "Close your eyes and take a deep breath. I know you were frightened, but the baby is healthy and everything is going to be okay."

Which statement by a nurse case manager regarding this nurse's role in client care is most accurate? a) "Even though I do not provide care to clients, my work is very important." b) "Moving away from client care is a necessary step to advancing my career." c) "I provide a critical service that is necessary for financial reimbursement." d) "I provide indirect care to my clients by coordinating their treatment with other disciplines."

d) "I provide indirect care to my clients by coordinating their treatment with other disciplines."

A nurse is preparing to assess a client with abdominal pain. Which statement is most appropriate for the nurse to use to gain cooperation from the client? a) "I am going to examine your abdomen." b) "I need to report what is going on to your health care provider. Can I look at your abdomen?" c) "Open your shirt, I need to look at your abdomen." d) "Let me explain what I am going to do and how you can help."

d) "Let me explain what I am going to do and how you can help."

A nurse is caring for a client experiencing biliary colic from uncomplicated cholelithiasis. The client asks, "My doctor says I should have surgery to remove my gallbladder. Do you think it is really necessary?" What is the nurse's best response? a) "You should follow your physician's recommendation and have the surgery." b) "When you see the physician this morning, request more information about the surgery." c) "It is a minimally invasive surgery with rapid recovery time, so you will do fine." d) "Share with me the advantages and disadvantages of your options as you see them."

d) "Share with me the advantages and disadvantages of your options as you see them."

A client is scheduled for thoracentesis. The nurse assesses that the client appears anxious about the procedure and needs honest support and reassurance. What is the most appropriate response by the nurse to this client? a) "The procedure may take only 2 minutes, so you might get through it by mentally counting up to 120." b) "I will be by your side throughout the procedure; the procedure will be painless if you don't move." c) "You may feel very uncomfortable when the needle goes in, but you should breathe rhythmically." d) "The needle causes discomfort or pain when it goes in, but I will be by your side throughout and will help you hold your position."

d) "The needle causes discomfort or pain when it goes in, but I will be by your side throughout and will help you hold your position."

The nurse is interviewing a client to obtain the health history. Which question would the nurse ask first? a) "Are you having any pain?" b) "Do you have any allergies?" c) "What medications do you normally use?" d) "What brings you here today?"

d) "What brings you here today?"

The client tells the nurse, "I think the nurse last night may have given me the wrong medication, but I was afraid to say anything." What is the nurse's most appropriate response? a) "I will discuss your concerns with the night nurse." b) "You always have the right to refuse any medication or treatment." c) "I will report your concerns to the nurse manager." d) "You should always speak up if you have any questions about your care."

d) "You should always speak up if you have any questions about your care."

A nurse assesses a postoperative client's level of consciousness and documents the following: the client's eyes open spontaneously; the client accurately responds to instructions, converses, and is oriented to time, place, and person. What score would this client receive on the Glasgow Coma Scale? a) 8 b) 12 c) 4 d) 15

d) 15

What nursing care behavior by the nurse engenders a client to trust the nurse? a) A nurse answers the client's questions about an upcoming test while completing documentation in the EHR. b) A nurse tells the client, "My shift will be over in 45 minutes, I will let the oncoming nurse know you have questions about tomorrow's test." c) A nurse tells the client, "Do not worry about the test, I have never cared for anyone that had problems with it." d) A nurse answers the client's questions about an upcoming test in a calm gentle voice while making eye contact with the client.

d) A nurse answers the client's questions about an upcoming test in a calm gentle voice while making eye contact with the client.

The nurse is completing the admission assessment on a client with a diagnosis of peripheral arterial disease (PAD). Which assessment finding is most significant? a) Hairless, shiny legs b) 2+ edema to lower extremities c) Thick overgrown toenails d) An absent popliteal pulse

d) An absent popliteal pulse

A nurse is caring for a client with paraplegia. Using observation to examine the client's skin, what finding might indicate the presence of a pressure injury? a) An area of swollen, pale red bumps on the front of the neck. b) An intact faded purple area on the shoulder blades, with a yellowish tint. c) A circular red, scaly area that itches on the top of the forearm arm. d) An intact red area on the buttocks.

d) An intact red area on the buttocks.

The nurse assesses a male client's genitalia and finds that the scrotal contents are asymmetrical. What action does the nurse take? a) Call the health care provider and request diagnostic testing. b) Insert an indwelling urinary catheter to ensure urine flow. c) Request a prostate-specific antigen diagnostic blood test. d) Ask the client about any usual genital observations.

d) Ask the client about any usual genital observations.

The nurse is preparing to do a focused assessment of the abdomen on a client following an abdominal hysterectomy. Which intervention is most important for the nurse to do prior to the physical assessment? a) Place the client in a semi-Fowler's position. b) Measure height and weight. c) Warm the equipment. d) Ask the client to empty her bladder.

d) Ask the client to empty her bladder.

A 52-year-old male client is admitted to the medical-surgical unit with a 3-day history of sharp, nonradiating epigastric pain and vomiting. He tells the nurse that he hasn't seen any blood in his stool and that he usually drinks a six-pack of beer a day. In trying to pinpoint the cause of the client's pain, which action would the nurse take? a) Rate the client's pain on a scale of 0 to 10, with 10 being the most severe. b) Ask the client to compare this pain with the most severe pain he's ever experienced. c) Ask the client if there was ever a time during those 3 days when he did not have pain. d) Ask the client to tell her more about the pain.

d) Ask the client to tell her more about the pain.

One hour after receiving pain medication, a postoperative client reports intense pain. What is the nurse's appropriate first action? a) Assist the client to reposition and splint the incision. b) Discuss the frequency of pain medication administration with the client. c) Consult with the physician for additional pain medication. d) Assess the client to determine the cause of the pain.

d) Assess the client to determine the cause of the pain.

Which technique should the nurse use to assess the pupillary light reflex on a client? a) Have the client focus on a distant object, then ask the client to look at the penlight being held about 4 cm from the nose and observe for pupil constriction. b) Use an ophthalmoscope to focus light on the sclera and observe for a reflection on each eye. c) Ask the client to follow the penlight in six directions and observe for bilateral pupil constriction. d) Bring a narrow beam of light from the temple toward the eye, observing for direct and consensual pupillary constriction.

d) Bring a narrow beam of light from the temple toward the eye, observing for direct and consensual pupillary constriction.

The health care provider asks the nurse to perform a Weber test for a client with suspected conduction hearing loss. Which finding should be reported? a) Client hears vibrations in both ears. b) Client hears vibrations when the tuning fork is placed near the ear. c) Client hears vibrations in the affected ear. d) Client hears vibrations in the unaffected ear.

d) Client hears vibrations in the unaffected ear.

A client on the medical-surgical unit is scheduled for several diagnostic tests. The nurse is concerned that the tests will be too tiring for the client. What would be the nurse's most appropriate action? a) Instruct the client to refuse the diagnostic tests if the client becomes too fatigued. b) Review the physician's progress notes to determine if any of the tests are not indicated. c) Coordinate with the other disciplines to determine if all the tests scheduled are necessary. d) Coordinate with the other disciplines to schedule the tests with adequate rest for the client.

d) Coordinate with the other disciplines to schedule the tests with adequate rest for the client.

The nurse must give instructions before discharge to a 13-year-old in a sickle cell crisis. Three of the client's friends from school are visiting. In order to assure effective instruction, what should the nurse plan to do? a) Leave written information for the client to read later. b) Ask the client if the client has any questions. c) Give the visitors instructions to leave in 10 minutes. d) Delay the instruction until the visitors leave.

d) Delay the instruction until the visitors leave.

A nurse is catheterizing a client. Which scenario demonstrates steps the nurse would take to ensure client respect and privacy? a) Ask family members to leave the room briefly, close the door, and ask the client if he or she would like a sheet to cover oneself. b) Bring in another nurse to provide support and prevent questionable behaviors, explain the procedure to the client, and close the door and privacy curtain prior to beginning the procedure. c) Verify the client's identifiable information, explain the procedure to the family, and pull the privacy curtain to allow the client to feel comfortable in having family close by. d) Explain the procedure to the client, close the door to the room, and cover all areas of the client, only exposing the area for catheterization.

d) Explain the procedure to the client, close the door to the room, and cover all areas of the client, only exposing the area for catheterization.

During morning report, the night nurse tells the oncoming nurse that the client has been medicated for pain and is resting comfortably. Thirty minutes later, the client calls and requests pain medication. What is the nurse's appropriate first action? a) Medicate the client with the ordered pain medication. b) Instruct the client in nonpharmacologic pain management. c) Determine the frequency of pain medication. d) Go to the client and assess the client's pain.

d) Go to the client and assess the client's pain.

A nurse is performing a head and neck assessment of a client suspected of having leukemia. How would the nurse detect enlarged lymph nodes commonly associated with this disease? a) Inspect and palpate the left and then the right carotid arteries. b) Inspect the client's ability to move his neck. c) Palpate the thyroid gland. d) Inspect and palpate the supraclavicular area.

d) Inspect and palpate the supraclavicular area.

The nurse and the physical therapist discuss the therapy schedule and goals for a client on a rehabilitation unit. What type of communication is occurring between the nurse and the therapist? a) Small-group b) Organizational c) Intrapersonal d) Interpersonal

d) Interpersonal

A 56-year-old client with Mexican heritage has a diagnosis of heart failure. The nurse's morning lung assessment of the client reveals crackles in the mid to lower lungs and respiratory rate of 32. The nurse notices that the client is restless, and his skin has an ashen appearance. Which nursing action is the priority intervention? a) Assess capillary refill. b) Assess fluid intake. c) Limit the client's activity. d) Measure the pulse oximetry.

d) Measure the pulse oximetry.

A nurse gives a speech on nutrition to a group of pregnant women. Within the model of the communication process, what is the speech itself known as? a)Channel b) Stimulus c) Source d) Message

d) Message

The nurse is assigned a client who had an uneventful colon resection 2 days ago and requires a dressing change. To which nursing team member should the nurse avoid delegating the dressing change? a) A senior nursing student present for clinical b) Licensed practical nurse c) Registered nurse d) Nursing assistant

d) Nursing assistant

A home care nurse discusses with a client when visits will occur and how long they will last. In what phase of the nurse-client relationship is this type of agreement established? a) Evaluation phase b) Working phase c) Termination phase d) Orientation phase

d) Orientation phase

The nurse is palpating a client's precordium. Which result is an expected clinical finding? a) Palpable vibration over the right sternal border b) Palpable thrill over the aortic area c) Palpable heave over the pulmonic area d) Palpable pulsation over the mitral area

d) Palpable pulsation over the mitral area

A nurse is performing a physical assessment for an older adult client who recently had a hip replacement. In what position would the nurse place this client to examine the hip joint? a) Sims b) Supine c) Dorsal recumbent d) Prone

d) Prone

A client with hypertension being seen for follow-up care has a blood pressure of 160/100 mm Hg. The client reports following the treatment regimen closely and that blood pressure readings have been elevated for the last 2 weeks. What is the nurse's most appropriate action? a) Interview the family to determine if the client is giving accurate information. b) Inform the client that the blood pressure medication will have to be changed. c) Reinforce the instructions for the treatment regimen to the client. d) Report the findings to the physician for further plans.

d) Report the findings to the physician for further plans.

Nursing interventions for the client after prostate surgery include assisting the client to ambulate to the bathroom. The nurse concludes that the client no longer requires assistance. What is the nurse's best action? a) Consult with the physical therapist to determine the client's ability. b) Instruct the client's family to assist the client to ambulate to the bathroom. c) Continue assisting the client to the bathroom to ensure the client's safety. d) Revise the care plan to allow the client to ambulate to the bathroom independently.

d) Revise the care plan to allow the client to ambulate to the bathroom independently.

The nurse has performed a Romberg test in the context of a client's neurologic assessment. The client has failed the test. The nurse should consequently identify what nursing diagnosis? a) Disturbed Thought Processes b) Acute Confusion c) Chronic Confusion d) Risk for Falls

d) Risk for Falls

Which statement accurately represents a characteristic of the third or fourth heart sound? a) S4 is the fourth heart sound, represented by "lub-dub-dee." b) S4 is considered normal in children and adults but abnormal in older adults. c) S3 is best heard with the stethoscope bell at the mitral area, with the client lying on the right side. d) S3 is considered normal in children and young adults and abnormal in middle-aged and older adults.

d) S3 is considered normal in children and young adults and abnormal in middle-aged and older adults.

The client recently immigrated from Mumbai, India. The client was just admitted to the nurse's unit postoperatively following gallstone removal. The client does not speak the dominant language. When using the hospital's interpretive services, which is most important? a) Ensure that family members are present. b) Give all of the discharge instructions at once. c) Have the interpreter write out all of the information listed in the unit brochure. d) Speak directly to the client.

d) Speak directly to the client.

The nurse has assessed a client and determined that the client has abnormal breath sounds and low oxygen saturation level. The nurse is performing what type of nursing intervention? a) Supportive b) Maintenance c) Collaborative d) Surveillance

d) Surveillance

A new unlicensed assistive personnel (UAP) is preparing to ambulate an obese client. The registered nurse (RN) is concerned about the UAP's ability to safely ambulate the client. Which would be the nurse's most appropriate action? a) Tell the UAP that a different UAP should ambulate the client. b) Tell the UAP to ask the client whether the client is comfortable with the UAP assisting ambulation. c) Tell the UAP not to ambulate the client at this time. d) Tell the UAP that the RN will assist the UAP with the client's ambulation.

d) Tell the UAP that the RN will assist the UAP with the client's ambulation.

A client who has been in a vegetative state for years is scheduled for an elective surgery. The nurse is questioning whether the procedure is necessary. What is the nurse's appropriate first action? a) The nurse should address the concern with the surgeon. b) The nurse should address the concern with the hospital attorney. c) The nurse should address the concern with the hospital ethics committee. d) The nurse should address the concern with the client's family

d) The nurse should address the concern with the client's family

The client has requested a translator so that she can understand the questions that the nurse is asking during the client interview. What is important when working with a client translator? a) Talking loudly helps the translator and the client understand the information better. b) Talking directly to the translator facilitates the transfer of information. c) It is always okay to not use a translator if a family member can do it. d) Translators may need additional explanations of medical terms.

d) Translators may need additional explanations of medical terms.

The nurse is preparing a client for an emergency exploratory laparoscopy. Before the procedure, it is most important for the nurse to take which action? a) Ensure that the preoperative check list is completed. b) Locate the laboratory test results in the chart. c) Document that the preoperative medication was administered. d) Verify that the procedural consent form is signed.

d) Verify that the procedural consent form is signed.

The nurse should use the bell of the stethoscope during auscultation of: a) a client's bowel sounds. b) a client's breath sounds. c) a client's apical heart rate. d) a client's heart murmur.

d) a client's heart murmur.

The nurse should use the bell of the stethoscope during auscultation of: a) a client's breath sounds. b) a client's bowel sounds. c) a client's apical heart rate. d) a client's heart murmur.

d) a client's heart murmur.

While assessing a 48-year-old client's near vision, the nurse can anticipate the client will state that her vision is: a) 20/20. b) clear. c) clouded. d) blurred.

d) blurred.

Which framework is used during the focused assessment? a) conceptual framework b) functional health assessment c) head-to-toe framework d) body systems framework

d) body systems framework

he nurse is caring for a client. When does the nurse determine that nursing care will be most effective related to nurse-client communication? a) similar personality traits b) similar values c) compatible realities d) common understanding

d) common understanding

When documenting client care, the nurse understands that the most important reason for correct and accurate documentation is: a) reducing legal liability risks. b) assisting in organization of care. c) noting the client's response to interventions. d) conveying information.

d) conveying information.

When percussing the liver, the sound should be: a) hyperresonant. b) resonant. c) flat. d) dull.

d) dull.

The nurse is beginning an assessment on a nonverbal client. The nurse must first: a) verbalize all steps of the nursing assessment when interacting with the client. b) speak loudly when interacting with the client. c) use various forms of communication when interacting with the client. d) establish eye contact prior to assessing, touching, and interacting with the client.

d) establish eye contact prior to assessing, touching, and interacting with the client.

A client is being treated for chronic obstructive pulmonary disease. The nurse auscultates the client's lungs following a period of coughing. The findings of this assessment are an example of: a) baseline data. b) comprehensive data. c) subjective data. d) objective data.

d) objective data.

Upon assessment of a client with myasthenia gravis, the nurse observes drooping of the upper eyelids. This finding is known as: a) ectropion. b) miosis. c) entropion. d) ptosis.

d) ptosis.

A nurse is at the end of a busy shift on a medical-surgical unit. The nurse enters a room to empty the client's urinary catheter and the client says, "I feel like you ignored me today." In response to the statement, the nurse should: a) smile at the client and apologize. b) ignore the statement and empty the urinary catheter. c) inform the client that the unit was very busy that day. d) sit at the bedside and allow the client to explain the statement.

d) sit at the bedside and allow the client to explain the statement.

A nurse follows set guidelines for administering pain medication to clients in a critical care unit. This nurse's authority to initiate actions that normally require the order or supervision of a physician is termed: a) protocols. b) nursing interventions. c) collaborative orders. d) standing orders.

d) standing orders.

A nurse is preparing to enter a client's room to perform wound care. The shift report revealed that this client has a tunneling wound in the sacral area that cannot be staged. The wound was also documented as having a foul odor. The nurse is nervous because the nurse has not performed wound care on a complex wound in the past. Using effective intrapersonal communication, this nurse should: a) inform the client that several nurses will be needed to care for this wound. b) tell the unlicensed assistive personnel (UAP) to gather supplies and to prepare to cleanse and dress the wound. c) ask the charge nurse to change the assignment. d) tell oneself to "remain calm" and remember that the nurse was trained to perform this skill.

d) tell oneself to "remain calm" and remember that the nurse was trained to perform this skill.

When recording or documenting outcome attainment in the chart, nurses are to be very clear with the descriptions used. Which term is appropriate? a. "Inadequate skills" b. "Great response" c. "Extremely well-mannered" d. "Demonstrated steps"

d. "Demonstrated steps"

The nurse just attended a seminar on cultural diversity. Which statement by the nurse would require further education? a. "Ethnicity begins at birth or through adoption of characteristics." b. "People of the same ethnicity share many of the same cultural and social beliefs." c. "Ethnicity can often determine dialect and political interests." d. "Ethnicity and race are the same thing and are affected by cultural practice."

d. "Ethnicity and race are the same thing and are affected by cultural practice."

A client is completing a health history form and asks the nurse, a. "Why does this ask for my ethnicity instead of my race?" What is the appropriate nursing response? b. "Race and ethnicity are the same thing." c. "Race is based on physical characteristics, and ethnicity is specific to where you live." d. "Ethnicity is a social category that is being used to replace racial categories." e. "Ethnicity is a sense of identity with a collective cultural group."

d. "Ethnicity is a social category that is being used to replace racial categories."

Which statement indicates that a plan to assist a client in developing and following an exercise program has been effective? a. "I guess I will begin the activity we discussed next week." b. "I have just been too busy to do my daily exercises." c. "I know I should exercise, but my health is not very good." d. "I have lost 10 pounds (4.5 kg) because I walk 2 miles (3.2 km) every day."

d. "I have lost 10 pounds (4.5 kg) because I walk 2 miles (3.2 km) every day."

The parents of a hospitalized 10-year-old ask the nurse if they can review the health care records of their child. What is the appropriate response from the nurse? a. "No, the physician will not give you access to review the records." b. "Only the client has the right to review the health care records." c. "Are you questioning the care of your child?" d. "I will arrange access for you to review the record after you put your request in writing."

d. "I will arrange access for you to review the record after you put your request in writing."

A nurse asks a nurse manager why staff nurses on the unit cannot document in a separate record (instead of the client record) to make it easier to find information on nursing-specific actions. What is the best response by the nurse? a. "The electronic health record we use does not allow us to use different formats." b. "It would be easier to do it that way. You could develop a tool to use." c. "The facility requires us to document client care this way because of the computer application used." d. "Legal policy requires nursing practice to be permanently integrated into the client record."

d. "Legal policy requires nursing practice to be permanently integrated into the client record."

The nurse is caring for a client whose spouse wishes to see the electronic health record. What is the appropriate nursing response? a. "Let me get that for you." b. "The provider will need to give permission for you to review." c. "I am sorry I can't access that information." d. "Only authorized persons are allowed to access client records."

d. "Only authorized persons are allowed to access client records."

A nurse is providing a change-of-shift report on a client who has had a restless night, is experiencing anxiety, and requires frequent repositioning. Which statement indicates a correct way of conducting an effective handoff at change of shift? a. "No medical issues overnight that require immediate attention." b. "The client was very restless last night so you may need to call the health care provider today to get a prescription for the client's anxiety." c. "The unlicensed assistive personnel turned the client every 2 hours last night, but the client should continue to be repositioned during the day." d. "The client had a good deal of anxiety last night and requested to be turned and repositioned frequently."

d. "The client had a good deal of anxiety last night and requested to be turned and repositioned frequently."

A nurse manager identifies a need for further instruction when a new nurse makes which statement? a. "The client is usually the best source for collecting data." b. "Caregivers can be a helpful source of data when the client has a limited capacity for information." c. "Family members are a good source of data when the client is a young child." d. "The client is always the best source for collecting data."

d. "The client is always the best source for collecting data."

The nurse cares for clients in the emergency department (ED). Which statement made by a health care professional is an indication of racism in the ED? a. "Mexicans are just here seeking drugs." b. "Emergency department nurses have to be very patient and caring people to deal with these people." c. "Asians are too stoic and bear their pain silently." d. "White people are superior to Black people."

d. ''White people are superior to Black people."

Which learner enjoys learning that takes place in the clinical setting? a. Sequential thinkers b. Grade-oriented students c. Learning-oriented students d. Active experimenters

d. Active experimenters

A nurse is educating a pregnant client in preterm labor on the use of the client's home monitoring equipment and medications. Which factor could impede the client's ability to learn? a. Preparation b. Intelligence c. Previous knowledge d. Anxiety

d. Anxiety

For a client with a self-care deficit, the long-term goal is that the client will be able to dress oneself by the end of the 6-week therapy. For best results, when should the nurse evaluate the client's progress toward this goal? a. When the client is discharged b. At the end of the 6-week therapy c. Only when the client shows some progress d. As soon as possible

d. As soon as possible

When is the best time for a nurse to take a client's health history? a. After the client is settled and feels ready b. Anytime before the client is discharged c. WIthin 24 hours of admission d. As soon as possible after a client presents for care

d. As soon as possible after a client presents for care

Which action would the nurse perform in the assessment phase of the nursing process? a. Coming up with a nursing diagnosis based on a potential health risk b. Determining whether the client's goals for wellness have been met c. Developing a plan to manage the client's health problems d. Asking the client whether the client has cultural preferences

d. Asking the client whether the client has cultural preferences

The nurse admits a client to the critical care unit to rule out a myocardial infarction. The client has several family members in the waiting room. Which nursing action is most appropriate? a. Allow all the visitors into the room. b. Explain to the family that too many visitors will tire the client. c. Insist that only one family member can be in the room at a time. d. Assess the client's beliefs about family support during hospitalization.

d. Assess the client's beliefs about family support during hospitalization.

A nurse administers intravenous fluids to a client diagnosed with dehydration. After the fluids are completed, the client's blood pressure is increased and pulse is decreased. During the final phase of the nursing process, what should the nurse do? a. Check the client's skin turgor. b. Formulate a plan of care based on risk for dehydration. c. Administer an additional liter of intravenous fluids. d. Determine whether the prescribed treatment was effective.

d. Determine whether the prescribed treatment was effective.

According to the Candian Nurses Association (CNA), what is the primary source of evidence to measure performance outcomes against standards of care? a. Psychomotor skills b. Accreditation c. Clinical judgment d. Documentation

d. Documentation

A nurse uses a bed scale to perform a client's daily weight. The nurse notes that today's weight is 3 kg less than the previous day's. What is the nurse's most appropriate action? a. Report this finding promptly to the client's primary care provider. b. Increase the frequency of the client's weight assessments. c. Encourage the client to increase food and fluid intake. d. Ensure that the scale is correctly calibrated and repeat the assessment.

d. Ensure that the scale is correctly calibrated and repeat the assessment.

Place the nursing activities in the order that they would most likely occur when a health care professional uses the nursing process. a. Interpreting and analyzing client data b. Modifying the plan of care (if indicated) c. Carrying out the plan of care d. Establishing the database e. Establishing priorities f. Measuring how well the client has achieved desired outcomes

d. Establishing the database a. Interpreting and analyzing client data e. Establishing priorities c. Carrying out the plan of care f. Measuring how well the client has achieved desired outcomes b. Modifying the plan of care (if indicated)

Once the nurse has administered pain medication, it is the nurse's responsibility to determine its effect and any other results. When accomplishing this follow-up with the client, the nurse is in which step of the nursing process? a. Assessment b. Planning c. Implementation d. Evaluation

d. Evaluation

A client comes to the emergency department with a productive cough and an elevated temperature. Which type of assessment would the nurse most likely perform on this client? a. Time-lapse b. Head-to-toe c. Emergency d. Focused

d. Focused

Which nursing intervention reflects practice according to Madeline Leininger's transcultural nursing theory? a. Contacting a chaplain for every client b. Providing the same care to each client who has had a myocardial infarction c. Planning dietary interventions according to physiological condition d. Incorporating the client's request for complementary treatment therapy

d. Incorporating the client's request for complementary treatment therapy

Which is a characteristic of person-centered care? a. It is independent of other disciplines. b. It can be used in hospital settings. c. It involves general care for all clients. d. It is a framework for providing care.

d. It is a framework for providing care.

Nurses at a health care facility maintain client records using a method of documentation known as charting by exception (CBE). What is a benefit of this method of documentation? a. It provides and refers to a client's problem by a number. b. It records progress under problems, intervention, and evaluation. c. It documents assessments on separate forms. d. It provides quick access to abnormal findings.

d. It provides quick access to abnormal findings.

A client will be transferred from the cardiovascular intensive care unit to the telemetry unit for continued care. Which documentation correctly demonstrates how the nurse would prepare information to be conveyed to the receiving nurse during a verbal handoff report? a. Mr. Alfred Jones, 8 days post-CABG to correct RVEF is being transferred to the telemetry unit. Vitals are BP 130/82, P 82 and irregular, R 21, T 99.2F (37.3C). Client is currently receiving D51/2 NS + 20 mEq KCl at 125 ml/hr in 18 gauge LFA PIV. Oxycodone pain medication administered at 0800 along with PRN acetaminophen. b. Alfred Jones, 76-year-old male-Transferring for monitoring for the next 7 days. Vitals are stable. IV fluids are currently being administered through R wrist with D51/2 NS + 20 mEq KCl at 125 ml/hr. Pain at incision relieved with a combination of oxycodone and acetaminophen at 0845. c. MR#12345, Alfred Jones, 76-year-old male 8 days post-op for RVEF. Transferring for monitoring for the next week. Braden score 13 and vitals are stable. IV fluids are currently being administered through R wrist with D51/2 NS + 20 mEq KCl at 125 ml/hr with orders to continue for 3 days. Pain at incision rated at 4 on a scale of 0-10 relieved with a combination of oxycodone and acetaminophen at 0845 with relief within 30 minutes. d. Mr. Alfred Jones, MR#12345, 76-year-old male, 8 days post-CABG to correct RVEF. Skin mostly warm and dry. Braden score 13. Vitals stable and documented in EHR. Client being transferred with D51/2 NS + 20 mEq KCl at 125 ml/hr in 18 gauge LFA PIV. Pain noted at 4 on the number scale. Oxycodone administered at 0800 with no relief reported. PRN acetaminophen administered at 0845 with pain decreased to 3 within 30 minutes.

d. Mr. Alfred Jones, MR#12345, 76-year-old male, 8 days post-CABG to correct RVEF. Skin mostly warm and dry. Braden score 13. Vitals stable and documented in EHR. Client being transferred with D51/2 NS + 20 mEq KCl at 125 ml/hr in 18 gauge LFA PIV. Pain noted at 4 on the number scale. Oxycodone administered at 0800 with no relief reported. PRN acetaminophen administered at 0845 with pain decreased to 3 within 30 minutes.

The nurse is caring for a client for the third day in a row on the hospital unit. At the client's evening vital sign assessment, the nurse notices that the radial pulse is much slower than the apical pulse. This finding is new. What should the nurse do next? a. Recheck the client's pulse at the next scheduled assessment time and document the findings on the chart. b. Document the findings on the chart and recheck in 1 hour. c. Notify the physician of the change and document the finding. d. Notify the physician after the next scheduled assessment time if the pulse is unchanged. e. Recheck the client's pulse in 2 hours.

d. Notify the physician of the change and document the finding.

After performing the admission assessment on an older adult client, the nurse documents the following, "Client observed fidgeting with covers; facial grimacing when turning from side to side." This documentation is an example of which type of data? a.Unreliable b.Subjective c.Physical d.Objective

d. Objective

The client reports, "I have a few drinks with friends every week." Which nursing action exemplifies using a focused assessment in this case? a. Interviewing friends to ascertain the client's exercise habits b. Asking the client to discuss social functioning c. Performing an abdominal assessment d. Obtaining data regarding the amount and frequency of drinking

d. Obtaining data regarding the amount and frequency of drinking

The nurse is conducting a client interview and notices that the client answers every question with a "yes" or "no" response. What is most likely the cause of this action by the client? a. Sleepiness b. Hunger c. Low anxiety d. Pain

d. Pain

Which is the most appropriate example of the assessment phase of the nursing process? a. Documenting the administration of a medication provided for pain b. Evaluating the temperature of a client given medication for a fever c. Including a nursing diagnosis of Acute Pain in the client's plan of care d. Palpating a mass in the right lower quadrant of the abdomen

d. Palpating a mass in the right lower quadrant of the abdomen

A nurse has developed a plan of care for an adult client. What nursing function is important when using nursing diagnoses to guide the care of this client? a. Add a new nursing diagnosis in the nurse's own words to individualize the plan of care. b. Keep resolved nursing diagnoses as part of the plan of care in case the related problems return. c. Do not allow the client to review the client's own nursing diagnoses. d. Prioritize the nursing diagnoses.

d. Prioritize the nursing diagnoses.

The nurses at a health care facility were informed of the change to organize the clients' records into problem-oriented records. Which explanation could assist the nurses in determining the advantage of using problem-oriented records? a. Problem-oriented recording has numerous locations for information where members of the multidisciplinary team can make entries about their own specific activities in relation to the client's care. b. Problem-oriented recording makes it difficult to demonstrate a unified approach for resolving clients' problems among caregivers. c. Problem-oriented recording gives clients the right to withhold the release of their information to anyone. d. Problem-oriented recording emphasizes goal-directed care to promote the recording of pertinent data that will facilitate communication among health care providers.

d. Problem-oriented recording emphasizes goal-directed care to promote the recording of pertinent data that will facilitate communication among health care providers.

Which is the best example of person-centered care provided by a registered nurse? a. Administration of pain medication every 4 hours to a client who is postoperative b. Development of a plan of care for a new admission c. Insertion of a nasogastric tube for gastric decompression d. Reassuring a client who is anxious about a procedure

d. Reassuring a client who is anxious about a procedure

The nurse is assessing the temperature of an 8-month-old infant using a tympanic membrane thermometer. The reading is 95.2°F (35.1°C). What should the nurse do next? a. Cover the infant. b. Ask the parent whether the child has been exposed to cold temperatures. c. Assess the skin for signs of cyanosis. d. Recheck the temperature, paying close attention to technique.

d. Recheck the temperature, paying close attention to technique.

Which action should the nurse perform in the evaluation phase? a. Carry out treatment procedures. b. Set priorities for care. c. Record interventions. d. Revise the plan of care.

d. Revise the plan of care.

The nurse is performing an assessment on a newly admitted client. The client states, "I feel really nervous." This is an example of which type of data? a. Intuition b. Hunch c. Objective d. Subjective

d. Subjective

Which action by the nurse is compliant with the Health Insurance Portability and Accountability Act (HIPAA)? a. Obtaining only the client's verbal acknowledgement of having been informed of the disclosure of information b. Releasing the client's entire health record when only portions of the information are needed c. Disclosing client health information for research purposes after obtaining permission from the client's physician d. Submitting a written notice to all clients identifying the uses and disclosures of their health information

d. Submitting a written notice to all clients identifying the uses and disclosures of their health information

A client who was admitted to the acute care unit with angina pectoris is no longer having chest pain. Based on this assessment, what does the nurse decide to do with the plan of care for chest pain? a. Establish a new plan of care. b. Keep the diagnosis of chest pain since it could occur again to prevent rewriting it. c. Continue the current plan of care since it is already complete. d. Terminate the plan of care related to the nursing diagnosis of chest pain.

d. Terminate the plan of care related to the nursing diagnosis of chest pain.

The nurse is summarizing the key points of the interview. This nursing activity occurs during which phase? a. Working phase b. Preparatory phase c. Introductory phase d. Termination phase

d. Termination phase

A nurse receives a report for a client who is going to surgery in the morning. The nurse is informed that the client is Jehovah's Witness. What education is needed for this particular client? a. The client needs to be informed that blood may be given if needed in an emergency situation so informed consent can be obtained. b. The client should be informed that surgery is not an option because blood will need to be transfused. c. The client should be informed that he or she can discuss the possible need for blood with a spiritual counselor prior to surgery to make an informed decision. d. The client needs to have a discussion with the health care team about blood product preferences.

d. The client needs to have a discussion with the health care team about blood product preferences.

The nurse is reassessing a client after pain medication has been administered to manage the pain from a bilateral knee replacement procedure. Which statement most accurately depicts proper documentation of pain assessment? a. The client is receiving sufficient relief from pain medication, stating no pain in either knee. b. The client appears comfortable and is resting adequately and appears to not be in acute distress. c. The client appears to have a low tolerance for pain and frequently reports intense pain. d. The client reports that on a scale of 0 to 10, the current pain is a 3.

d. The client reports that on a scale of 0 to 10, the current pain is a 3.

The following statement is documented in a client's health record: "Patient c/o severe H/A upon arising this morning." Which interpretation of this statement is most accurate? a. The client has a history of severe complaints in the morning. b. The client has symptoms in the morning associated with a heart attack. c. The client is coughing and experiencing severe heartburn in the morning. d. The client reports waking up this morning with a severe headache.

d. The client reports waking up this morning with a severe headache.

A client has been diagnosed with PVD. On which area of the body should the nurse focus the assessment? a. Heart rate and rhythm b. The abdominal area c. Lung sounds d. The lower extremities

d. The lower extremities

Which nursing action reflects evaluation? a. The nurse identifies that the client does not tolerate activity. b. The nurse sets a tolerable pain rating with the client. c. The nurse auscultates the client's lungs and abdomen. d. The nurse assesses urine output following administration of a diuretic.

d. The nurse assesses urine output following administration of a diuretic.

Which action by the nurse is an example of peer review? a. The nurse seeks feedback from the nurse manager regarding job performance for the previous year. b. The nurse seeks information from the unlicensed assistive personnel (UAP) regarding the family's response to the nurse's education. c. The new graduate nurse consults the policies and procedures of the institution prior to skill implementation. d. The nurse preceptor provides feedback to the new graduate nurse after 6 weeks of orientation.

d. The nurse preceptor provides feedback to the new graduate nurse after 6 weeks of orientation.

Which statements are true about informatics in nursing practice? Select all that apply. a. Computers do not help with communication, but deter it because of the lack of personal interaction. b. Informatics only involves documentation of timely and accurate charting. c. Nurses should value technologies that support error prevention and care coordination. d. The use of informatics can help manage knowledge and mitigate error. e. Utilization of information services helps to support decision making.

d. The use of informatics can help manage knowledge and mitigate error. c. Nurses should value technologies that support error prevention and care coordination. e. Utilization of information services helps to support decision making.

A nurse practitioner in private practice with a physician is providing psychiatric care to a client with a history of being abused by a spouse. During the last visit, the client stated an intent to leave the spouse. In the next visit, the nurse practitioner will reassess the client's commitment to this intended change. What type of assessment is the nurse practitioner implementing? a.Focused b.Emergency c.Complete d.Time-lapse

d. Time-lapse

A nurse is preparing to auscultate a client's abdomen for the presence of bowel sounds. Which is the appropriate action of the nurse? a. Assist the client to a sitting position. b. Palpate the abdomen before auscultating. c. Uncover the client to expose the chest and abdomen. d. Warm the diaphragm of the stethoscope.

d. Warm the diaphragm of the stethoscope.

The nurse working on a medical unit always performs hand hygiene between contact with each client. In addition to being understood as an infection control measure, this practice can be understood as: a. a custom. b. an obsession. c. a coping strategy. d. a ritual.

d. a ritual

The charge nurse overhears two new graduate nurses talking in the break-room. One graduate nurse states, "I hate getting reports from the older nurses; they are just too slow." The charge nurse understands that the nurse is demonstrating what? a. stereotyping b. cultural shock c. ethnocentrism d. ageism

d. ageism

During the preparatory phase of interviewing for the purpose of obtaining information for the nursing history, the nurse should: a. identify existing and potential health problems. b. clarify the client's health status. c. develop the nursing plan of care. d. review as much information as possible.

d. review as much information as possible.

The nurse is interviewing a newly admitted client. Quoting statements made by the client will help in maintaining what type of assessment data? a. reimbursement b. organization c. objectivity d. subjectivity

d. subjectivity

The nurse is preparing a SOAP note. Which assessment findings are consistent with objective client data? a. concerned with feeling tired b. describes wound as itchy c. pain rating of 4 on a scale of 0-10 d. urine output 100 ml

d. urine output 100 ml

The nurse is teaching about sources of calcium for a client newly diagnosed with lactose intolerance. The client would require further teaching if which food is chosen? a. green, leafy vegetables b. prunes c. egg yolks d. yogurt

d. yogurt

After sustaining a wrist fracture in a recent fall, a client is suspected of having osteoporosis. Which data best demonstrates the nursing focus of assessment? a.The client's serum calcium levels are below the reference range. b.The client demonstrates an unsteady gait and spinal kyphosis. c.The results of the client's bone scan indicate decreased bone density. d.The client claims mobility and independence have declined in recent years.

d.The client claims mobility and independence have declined in recent years.

An older adult client who has been living in an assisted living facility for several months informs a visiting family member that a nurse is coming to do some kind of checkup. Which type of check would be most appropriate for the nurse to perform on this client? a.Emergency assessment b.Focused assessment c.Developmental stage assessment d.Time-lapsed assessment

d.Time-lapsed assessment

A nurse is assessing an apical pulse on a cardiac client. The client is taking digoxin, which is a cardiac medication. The nurse can anticipate that the digoxin will:

decrease the apical pulse.

Which is not a characteristic used to describe the pulse?

depth

When assessing an adult client's pulse at 125 beats/min, which step would the nurse take first to determine intervention?

determine cause

The body loses heat continually through several different processes. Which process is an example of how heat is lost through evaporation?

diaphoresis

Which client should not have a temperature assessed rectally?

diarrhea

The nurse is assessing the pulse amplitude for a client. Documentation by the nurse states, "Pulses are +1 in the lower left extremity." What amplitude is the nurse assessing?

diminished, weaker than expected

A nurse is completing a vision exam with the Snellen eye chart and records the client's vision as 20/30 or 6/9. The client asks the nurse, a) "What does that mean?" How should the nurse respond? b) "Your vision is perfect; you can read the entire chart, and you do not need glasses." c) "Your vision in your right eye is slightly different than that of your left eye." d) "Your vision is better than average; you can read from 30 ft (9 m) what a person with normal vision can read from 20 ft (6 m)." e) "You are able to read at 20 ft (6 m) what a person with normal vision can read at 30 ft (9 m)."

e) "You are able to read at 20 ft (6 m) what a person with normal vision can read at 30 ft (9 m)."

Upon assessing a client who is hemorrhaging, the nurse is most likely to assess which compensatory change in vital signs?

increased pulse rate

The temperature is 102°F (39°C) during a heat wave. The nurse can expect admissions to the emergency room to present with:

increased temperature

During the introductory phase of interviewing a client for the purpose of obtaining information for the nursing history, the nurse should: a.inform the client of the maintenance of confidentiality. b.implement supportive nursing interventions. c.review literature pertinent to the client's attributes. d.assess personal feelings regarding similar clinical situations.

inform the client of the maintenance of confidentiality.

A nurse can most accurately assess a client's heart rate and rhythm by which of the following methods?

listen with the stethoscope at the fifth intercostal space left mid-clavicular line

Which outcome best reflects achievement of the goal, "The client will demonstrate correct steps in taking his own pulse rate"?

palpation of the radial pulse on the thumb side of the inner aspect of the wrist.

Nurse T. has auscultated Mr. Weinstein's apical pulse while a colleague simultaneously palpated his radial pulse. This assessment of Mr. Weinstein's apical-radial pulse indicates that the two values differ significantly, a finding that suggests which health problem?

peripheral vascular disease

A nursing student is assessing blood pressure in an adult client. Which action by the nursing student would require intervention from the nursing instructor?

pumping the blood pressure cuff up to 200 mmHg routinely

The nurse has delegated an unlicensed assistive personnel (UAP) to obtain a temperature reading for a client who has neutropenia. Which route used by the UAP requires immediate intervention?

rectal

Which site results in measuring a client's core body temperature?

rectal

Infants' and children's pulses vary most with:

respirations

A nurse is assessing the blood pressure of a team of healthy athletes at the heath care facility. Which observation can be made by the nurse and athletes by measuring the blood pressure?

the ability of the arteries to stretch

The nurse teaches a client diagnosed with hypertension to self-measure blood pressure with an automated device. Which client behavior indicates the need for additional teaching?

the client sits in the chair with feet flat on the floor and arm below the level of the heart.

The nurse is assessing a new client's blood pressure, using a manual sphygmomanometer. Which sound constitutes the client's systolic blood pressure?

the first appearance of faint but distinctive tapping sound

When developing an appropriate nursing diagnosis, the nurse needs to keep in mind that: the interventions planned must be within the nurse's scope of practice. the problem's existence requires validation by the physician. the main focus is on monitoring the body's pathophysiologic response. the signs and symptoms of the disease are part of the information conveyed.

the interventions planned must be within the nurse's scope of practice. A nursing diagnosis describes an actual, risk, or health promotion response to a health problem that nurses are responsible for treating independently. Nursing diagnoses describe the client's response to the disease process, developmental stage, or life process and provide a convenient way to communicate nursing therapies or interventions. Nursing diagnoses carry legal ramifications. Only health care problems within the scope of nursing practice may be identified as nursing diagnoses. A nurse may not diagnose a medical disease and is not licensed to independently treat such a problem. Medical diagnoses, not nursing diagnoses, require validation by the physician that the problem exists, are focussed on pathophysiologic responses of body organs and systems, and convey information about signs and symptoms of disease.

A nurse is preparing to interview a client as part of the assessment. The nurse demonstrates knowledge of communication skills when the nurse:

uses broad, open statements to communicate with the client.

The nurse is aware that nursing diagnoses are: within the nursing scope of practice to develop and client-focused. collaborative and depend on the medical diagnosis. based on assessment data and the primary care provider's input. dictated by the medical diagnoses and change day by day.

within the nursing scope of practice to develop and client-focused. Nursing diagnoses are within the nursing scope of practice to develop and are client-focused. They are developed collaboratively with the client, are based on assessment data, and can change from day to day as the client's responses change. They are not do not depend on nor are they dictated by medical diagnoses, and they are not based on the input of the primary care provider.

How should a nurse best document the assessment findings that have caused the nurse to suspect that a client is depressed following a below-the-knee amputation?

"Client states, 'I don't see the point in trying anymore.'"

A nurse is caring for a client with orthostatic hypotension. Which nursing interventions are appropriate to decrease the risk of falls? Select all that apply.

-encourage oral fluid intake -encourage slow movement from the bed to the chair -encourage the client to use the call light prior to getting out of bed -encourage the use of the call light for help to the bathroom

The normal adult temperature obtained through the oral route ranges from:

97.6°F to 99.6°F (36.4°C to 37.6°C)

A community health nurse has been visiting a diabetic client whose morning fasting glucose levels are constantly elevated. Upon further assessment, the nurse determines that the client's spouse does not understand how to prepare meals following the prescribed diabetic diet. Using Dorothea Orem's Self-Care Theory, how can the nurse help meet the needs of this client? A. Arrange an evaluation appointment with a dietitian. B. Set up diabetic home meal delivery for the client. C. Assist the spouse with online grocery shopping. D. Authorize an unlicensed nursing personnel to cook for the client.

A. Arrange an evaluation appointment with a dietitian.

Which action would the nurse perform in the assessment phase of the nursing process?

Asking the client whether the client has cultural preferences

A nurse is using quantitative research for a question related to nursing practice. When conducting the research, which aspect would the nurse need to keep in mind? Select all that apply. A. The focus needs to be holistic. B. Concepts are fairly well-developed. C. An existing body of literature is available. D. Data are collected in real-world settings. E. Statistical processes are used to interpret the data.

B. Concepts are fairly well-developed. C. An existing body of literature is available. E. Statistical processes are used to interpret the data.

A client who is scheduled for coronary angioplasty is concerned about whether the surgery is safe and wonders whether it would be beneficial. Which nursing diagnosis relates to this client's condition? Ineffective Coping related to anxiety and fear of surgery Anxiety related to fear of death during surgery Fear related to potential risk and surgical outcomes Knowledge Deficit: treatment regimen related to surgical outcomes

Fear related to potential risk and surgical outcomes.

27. According to the Nursing Interventions Classification (NIC) system, the most basic level of nursing intervention is:

Physiological

The health care provider tells the nurse that the desired body temperature for the client is 99.8°F (37.7°C). When does the nurse turn off the blanket?

When the client's temperature reaches 100.8°F (38.22°C).

What is the priority assessment for the nurse when developing a plan of care for a client living in poverty? a. Access to care b. Access to health insurance c. Access to affordable housing d. Access to financial assistance

a. Access to care

Upon admission to the hospital, the client states, "I am having surgery to correct my back. I have pain in the lower back and the doctor is going to do a lumbar laminectomy." This statement reflects the client's: a) chief concern. b) symptoms. c) review of systems. d) objective assessment.

a) chief concern.

It is important for the nurse to empathize with the client to develop a positive, therapeutic relationship. What is a characteristic of empathy? a) Experiencing feelings similar to those of the client b) Identifying with the client's feelings c) Caring for the client without negative judgment d) Conveying genuine care to the client

b) Identifying with the client's feelings

An experienced nurse is orienting a new nurse to the unit. Which activity demonstrates the nurse is an effective caregiver? a) The nurse leaves the room when a client is crying to provide privacy. b) The nurse uses open-ended questions when working with a crying client. c) The nurse documents the client was crying at the end of the shift. d) The nurse calls the hospital chaplain to talk with the client.

b) The nurse uses open-ended questions when working with a crying client.

The nurse is assessing a 4-year-old child who has multiple bee stings. Which assessment finding would require immediate action by the nurse? a) Heart rate of 100 b) Wheezing on auscultation c) Erythema at sting site d) Crying with burning pain

b) Wheezing on auscultation

A client is scheduled for a CABG procedure. What information should the nurse provide to the client? a. "The CABG procedure will help increase intestinal motility and prevent constipation." b. "A coronary artery bypass graft will benefit your heart." c. "A complete ablation of the biliary growth will decrease liver inflammation." d. "The CABG procedure will help identify nutritional needs."

b. "A coronary artery bypass graft will benefit your heart."

"Measurable qualities, attributes, or characteristics that identify knowledge or health status" defines: a. standards b. criteria c. evaluations d. evidence-based practice

b. criteria

The nurse has finished assessing a newly admitted 6-month-old Native American/First Nations client. Which clinical findings should be immediately reported to the health care provider? a) the abdomen appearing large in relation to the pelvis b) a blue-black macular area over the sacral area c) circumoral cyanosis when the client is at rest d) the anterior fontanel bulging when the client cries

c) circumoral cyanosis when the client is at rest

A nurse who works on a day-surgery unit conducts a thorough, head-to-toe assessment of each client prior to the client's scheduled surgery. The nurse would document an unexpected finding if unable to palpate a client's: a) lymph nodes. b) liver. c) peripheral pulses. d) thyroid gland.

c) peripheral pulses.

When the nurse is administering medication, an older adult client states, "Why does everyone keep asking my name? I've been here for days." How should the nurse respond to the client? a) "It is a habit that nurses develop in school." b) "We ask your name to show that we respect your rights." c) "It is a hospital policy to reduce the potential for errors." d) "We ask your name to ensure that we are treating the right client."

d) "We ask your name to ensure that we are treating the right client."

The nurse is assessing the spine of a 63-year-old woman who states, "I hope I don't end up with a big hump on my back like my mother did." The nurse knows the patient is referring to a condition known as: a. spina bifida b. arthritis c. lordosis d. scoliosis e. kyphosis

e. kyphosis

A client informs the nurse that a mercury thermometer is used at home to take the temperature of her children when they are sick. What health education by the nurse is most appropriate?

encourage the client to use an alternative type of thermometer to assess temperature in the home

When performing an assessment, the nurse should focus most on the developmental stage for which client? a.Young adult b.Adolescent c.Middle-age adult d.Toddler

d. toddler

While recording the pulse of an infant at 160 beats/min, the parent asks if it is normal for the infant's pulse rate to be so high. Which statement by the nurse best answers the parent's question?

"A heart rate of 160 beats/min is normal for a healthy infant."

When assessing a client's pulse, the nurse is able to palpate the pulse for some time before losing it upon exerting a little bit more pressure. The pulse is beating at 80 bpm. Which of these should the nurse document as the character of the client's pulse?

thready pulse

A nurse is calculating the cardiac output of an adult with a stroke volume of 75 mL (75 × 109/L) and a pulse of 78 beats/min. What number would the nurse document for this assessment?

5,850 mL (5,850 × 109/L)

The nurse is educating a patient at risk for atherosclerosis. What nonmodifiable risk factor does the nurse identify for the patient?

Positive family history

43. The nurse recognizes that an example of a cognitive outcome is:

The client identifies three foods high in potassium by August 8.

Care provided to a client following surgery and until discharge represents which phase of the nurse-client relationship? a) Termination phase b) Working phase c) Evaluation phase d) Orientation phase

b) Working phase

In SBAR, what does R stand for? a. Response b. Recommendations c. Report d. Reinforcing data

b. Recommendations

A client presents to the Emergency Department with a temperature of 100.6F (38.1°C) and BP of 108/60 mmHg. What intervention does the nurse anticipate providing?

oral fluids

21. What verbs should the nurse use to write outcomes that are measurable? Select all that apply.

-Verbalize -Define

The nurse is taking the apical pulse of a 6-month-old infant. Upon completion, the nurse tells the parent the baby's pulse is 140 beats per minute. The parent is concerned, stating, "That seems kind of high!" The nurse responds:

"I know it seems fast, but normal infant heart rates are 100-160 beats per minute."

The nurse instructs a mother of young children how to properly use a nonmercury glass thermometer. Which statement made by the client indicates a need for further instruction?

"I will clean the thermometer in the dishwasher."

A nurse is attending a job fair at the local community center and stops at a booth that is demonstrating technology for client care. While touring the booth, the nurse meets an informatics nurse specialist (INS). Being unfamiliar with this position, the nurse asks the INS, "How did you get this position?" Which response by the nurse would be most appropriate? "I am required to have a bachelor's degree in nursing." "I've had graduate-level education focusing on informatics." "I was involved in getting our electronic health record off the ground." "I have a doctorate in nursing with a minor in computer technology."

"I've had graduate-level education focusing on informatics."

Which are examples of subjective data? Select all that apply.

*Anxiety *Light-headedness *Nausea

44. Which is an appropriate expected outcome for a client?

Client will ambulate safely with walker in the room within 3 days of physical therapy.

The nurse is caring for a client admitted to the hospital for renal calculi. What is the action to take first? a) Assess for bladder distention. b) Diet as tolerated. c) Strain urine after each void. d) Force fluids by mouth.

a) Assess for bladder distention.

When assessing the firmness of a client's abdomen, the nurse should use which assessment technique?

Palpation

A nurse needs to measure the blood pressure of a client who has just undergone a bilateral mastectomy. How should the nurse measure the blood pressure?

over the clients thigh

45. A client is required to have nothing by mouth (NPO) for 8 hours prior to a test scheduled for tomorrow. What action by the nurse best communicates this change in basic care needs for the client?

Updating the diet orders in the client's plan of care

An obese client has developed peripheral edema as a consequence of heart failure, making it very difficult for the student nurse to accurately palpate the client's peripheral pulses. How should the nurse proceed with this assessment?

auscultate the client's apical pulse

Which nursing qualities are helpful in winning the confidence of clients when first working with them? Select all that apply.

*Respect for client *Competence *Professionalism *Caring

Which items reflect the assessment phase of the nursing process? Select all that apply.

*The nurse asks the client, "How would you rate your pain?" *The client's abdomen is firm and distended with hypoactive bowel sounds. *The client states, "I rarely sleep more than 6 hours."

A client reports feeling "different" than earlier in the day. When would the nurse anticipate assessing vital signs?

immediately

A client is caring for the client's mother-in-law, who is an older adult who requires assistance with performing activities of daily living. Which statement by the client would lead the nurse to make a nursing diagnosis of Caregiver Role Strain? "I just don't have time to take a shower." "I feel great but wish that I could get more sleep." "My mother-in-law and I go for a walk daily." "My mother-in-law makes dinner on Tuesdays, and I cannot stand her cooking."

"I just don't have time to take a shower." Any of these statements could be a clue to caregiver role strain when clustered with other evidence. However, the inability to care for oneself, such as not taking time for a shower, strongly indicates that this client is not coping well.

While performing an assessment on a young client, the nurse is using the Functional Health Patterns Model. When recording the facts that the client exercises daily, hikes weekly, and plays on a softball team regularly, under which heading should these data be clustered ?

Activity/Exercise

Florence Nightingale was a nursing pioneer who challenged prejudices against women and elevated the status of all nurses. Which statement accurately describes one of her accomplishments? A. She established the fact that nursing is the same as medicine. B. She promoted the addition of nursing education as part of a medical degree. C. She established the tenets of the American Red Cross. D. She promoted the publication of books about nursing and health care.

D. She promoted the publication of books about nursing and health care.

A nurse is caring for an adult with fever. The nurse determines that which site is most accurate for obtaining the client's core body temperature?

rectum

While giving a client a bath, the nurse observes the color of the client's skin as having a yellowish tinge. Which question would the nurse ask the client? a. "Have you had diarrhea or constipation lately?" b. "How long have you had these spots?" c. "How much caffeine do you drink per day?" d. "Do you use acetaminophen or drink alcohol?"

"Do you use acetaminophen or drink alcohol?"

Which statement made by the nurse indicates data that would be documented as part of an objective assessment?

"The client's right leg is cold to the touch, from the knee to the foot."

13. A nurse is caring for a client admitted for bowel obstruction, which now has been resolved. The client has an order to "resume oral feeding as tolerated." Which are appropriate nursing interventions related to this medical order? Select all that apply.

-Begin feedings with clear broth. -Consult with a dietitian regarding appropriate foods. -Auscultate for bowel sounds.

The nurse is obtaining vital signs for a client and assesses a heart rate of 124 beats per minute. What additional assessment information would be important to obtain that would explain the tachycardia? Select all that apply.

-The client has reports of pain of 8 on a scale of 0 to 10 -The client just finished ambulating with physical therapy -The client has a temperature of 101.8°F (38.8°C)

The nurse is obtaining vital signs for a client and assesses a heart rate of 124 beats per minute. What additional assessment information would be important to obtain that would explain the tachycardia?

-The client has reports of pain of 8 on a scale of 0-10 -The client just finished ambulating with physical therapy -The client has a temperature of 101.8 degrees Fahrenheit

The nurse is assessing a client's brachial artery blood pressure. Which nursing actions are performed correctly? Select all that apply.

-The nurse centers the bladder of the cuff over the brachial artery about midway on the arm. -The nurse wraps the cuff around the arm smoothly and snugly and fastens it. -The nurse has the client lying or sitting down with the forearm supported at the level of the heart and the palm of the hand upward.

The nurse is preparing to measure a client's rectal temperature. Which supplies and equipment should the nurse have available before beginning the procedure? Select all that apply.

-an electronic thermometer with a rectal probe -disposable probe cover -water-soluble lubricating gel

Which is a characteristic of nursing practiced from early civilization to the 16th century? A. Most early civilizations believed that illness had supernatural causes. B. The physician was the priest who treated disease with prayer. C. The nurse was a nun committed to caring for the needy and homeless. D. Nursing changed from a spiritual focus to an emphasis on knowledge expansion.

A. Most early civilizations believed that illness had supernatural causes.

An informatics nurse specialist has completed the evaluation of an update to a current clinical information system used by the staff at the local hospital and has documented the results. Documentation reveals the need for an improvement in the screen display. Which action would be next? Analyze and Plan Test Train Implement

Analyze and Plan

An informatics nurse is assisting with the development of a new clinical information system that will be implemented in the facility. As part of the process, the team is evaluating the purpose of the system and the technological options available. The team is in which phase of the system development lifecycle? Analyze and plan Design Test Train

Analyze and plan

A 19-year-old college basketball player is being evaluated for injuries after a skiing accident. The nurse determines that the client has a pulse of 52 beats/min. What would be the most appropriate way for the nurse to determine the significance of the client's heart rate? Ask the client whether the heart rate is normal for the client. Compare the client's heart rate to that another teenaged client. Have another nurse reassess the heart rate for accuracy. Determine whether the client has any risk factors for cardiac disease.

Ask the client whether the heart rate is normal for the client. A well-conditioned athlete is very likely to have a pulse rate lower than normal at rest. The key assessment is to compare the current heart rate with the client's baseline. Asking the client would be a simple way of confirming it. Comparing the client's heart rate with that of another teenaged client does not take into account the individual differences of clients. If a nurse is competent in physical assessment, there is no need to have another nurse check the heart rate. The pulse rate of 52 beats/min does not indicate any risk for cardiac disease. The client is also being seen in the emergency room for an urgent health problem. This assessment can wait until later.

A nurse attempts to count the respiratory rate of a client via inspection and finds that the client is breathing at such a shallow rate that it cannot be counted. What is an alternative method of determining the respiratory rate for this client?

Auscultate lung sounds, count respirations for 30 seconds, and multiply by 2.

When assessing a client's radial pulse, the nurse notes an irregular rhythm with a rate of 62 beats per minute. What intervention should the nurse implement next?

Auscultate the apical pulse for 60 seconds

28. Which client outcome requires modification

By the end of instruction, client will know how to perform dressing changes.

Which example of client care is not the responsibility of the nurse? Monitoring for changes in health status Promoting safety and preventing harm; detecting and controlling risks Tailoring treatment and medication regimens for each individual Confirming a medical diagnosis

Confirming a medical diagnosis The nursing scope of practice dictates what is allowed and not allowed when providing nursing care. Confirming a medical diagnosis is not in the scope of nursing practice. Monitoring for changes in a client's health status, promoting safety and preventing harm, and tailoring treatment and medication regimens to the client's schedule of activities are all nursing care responsibilities.

What are nursing diagnoses based on? Symptoms Signs Cues Problems

Cues Nursing diagnoses are based on clusters of significant and related cues, or subjective (i.e., symptoms) and objective (i.e., signs) data that the nurse gathers during assessment. Problems are what nursing diagnoses describe, not what they are based on.

When the nurse researcher informs the participant that the participant's identity will not be linked with the information that is collected, the researcher is ensuring the participant's: A. anonymity. B. protection from harm. C. ability to withdraw. D. confidentiality.

D. confidentiality.

An informatics nurse specialist is collecting data from the clinical information system about the demographics of individuals diagnosed with heart failure admitted to the facility over the past five years. The nurse specialist is preparing a presentation to the facility's executive board. To promote understanding of this complex information, the nurse specialist prepares the data results using a pie chart and a bar graph. The nurse specialist is using which area of analytics? Data visualization Predictive analytics Big Data Data mining

Data visualization

A group of nurses are participating in being the first group of staff to use a new electronic pain assessment tool. The group is discussing whether or not the system is easy to use. During the discussion, the group mentions that "the shortcuts provided are really helpful and save valuable time." The informatics nurse specialist interprets this statement as reflecting which concept? Forgiveness Efficient interactions Naturalness Effective use of language

Efficient interactions

An informatics nurse is participating in an online continuing education course about nursing informatics. The nurse demonstrates successful comprehension of the course by identifying which individual as being considered the first informatics nurse? Florence Nightingale Dorothea Orem Hildegard Peplau Virginia Henderson

Florence Nightingale

When developing nursing diagnoses, the nurse should focus on which area? Actions to be initiated for treatment Human responses to actual or potential health problems Pathophysiological responses occurring in body systems Problem validation through physician collaboration

Human responses to actual or potential health problems The main focus of nursing diagnoses is on monitoring human responses to actual or potential health problems, whereas the main focus of medical diagnoses and collaborative problems is on monitoring the pathophysiological responses of body organs or systems. Actions to be initiated for treatment are the main focus for interventions or treatment. Collaboration with the physician to validate the problem reflects medical diagnoses or collaborative problems.

Which best describes the purpose of nursing diagnoses? Identification of client problems that nurses can treat independently Identification of signs and symptoms that identify diseases Identification of client problems that require collaboration with other health care professionals to treat Identification of actual client problems, not including potential problems

Identification of client problems that nurses can treat independently Nursing diagnoses are written to describe client problems that nurses can treat independently. Medical diagnoses identify diseases, whereas nursing diagnoses focus on unhealthy responses to health and illness. Collaborative problems require that a nurse work with other health care professionals, and the treatment comes from nursing, medicine, and other disciplines. Nursing diagnoses identify actual and potential client problems.

An informatics nurse specialist is gathering data from electronic health records at the facility about clients who have had central venous catheters inserted for more than the recommended time as specified by the facility's protocol. The nurse specialist is collecting this data most likely for which purpose? Identify clients at risk for infection Determine client satisfaction with care Correlate the client's diagnosis with intervention Evaluate client care

Identify clients at risk for infection

The nurse is admitting a client who is unable to identify person, place, or time. To properly analyze these data, what action must the nurse take? Determine the client's medical diagnosis for clarification. Interview the client's family to assess the client's usual level of cognition. Assess the client's vital signs to determine the client's baseline. Ensure precautions are taken to prevent injury to the client.

Interview the client's family to assess the client's usual level of cognition.

A client monitoring his BP at home notices that his BP is higher in one arm than the other. He calls his health care provider for guidance. What is the most appropriate information for the nurse to give this client?

It has been found that most people have differences in BP between arms and that he should use the arm that gives him the highest reading for accurate results.

A nurse is assessing a newborn at the health care facility when the mother of the child asks the nurse why the body temperature of her baby is unstable. Which response by the nurse would be most appropriate?

It is because of the immature ability to regulate temperature in general

An informatics nurse specialist has received an update from the vendor who is providing the electronic health record. During which phase of the system development lifecycle would this update be made? Test Maintain Implement Evaluate

Maintain

15. A 16-year-old client was admitted to the medical unit 1 hour ago for sickle cell crisis. Vital signs are as follows: temperature, 98.24°F (36.8°C) sublingual; heart rate, 95 beats/min; respiratory rate, 20 breaths/min; blood pressure, 130/65 mm Hg. The client rates pain as a 9/10. The nurse is talking with the medical resident on service to discuss client orders. Which order is the nurse likely to request first for the client?

Narcotic analgesic to treat pain

An informatics nurse specialist is interviewing several nurses who have participated in testing a new electronic assessment tool. The nurses report that the tool "feels so familiar, like we know exactly what it is that we're supposed to do." The nurse specialist interprets this as indicating which concept? Naturalness Consistency Simplicity Forgiveness

Naturalness

The nurse has identified a collaborative problem of Risk for Complications of Electrolyte Imbalance for a client with diarrhea. The client begins to exhibit a decrease in level of consciousness. What is the nurse's most appropriate action? Notify the physician for additional orders. Document the client's level of consciousness. Consult with another nurse to validate the assessment. Decrease stimulation and allow the client to rest.

Notify the physician for additional orders. The client's decreased level of consciousness could indicate that the client is developing an electrolyte imbalance. The change in the client's status requires notification of the physician. Medication orders are required to treat the electrolyte imbalance. Documenting the level of consciousness is appropriate, but not as the priority action. Another nurse is not necessary to check the nurse's assessment. Decreasing stimulation and allowing the client to rest with no further action may result in harm to the client.

The nurse is preparing to assess a rectal temperature on an adult client. What is the appropriate nursing intervention?

Provide privacy for the client.

The nurse is sharing information about a client at change of shift. The nurse is performing what nursing action?

Reporting

Which is a legal responsibility of a nurse who has documented a nursing diagnosis related to a client's kidney failure? Reporting signs and symptoms related to the client's kidney failure Independently managing the client's kidney failure Coordinating the treatment of the client's kidney failure Choosing interventions to resolve the client's kidney failure

Reporting signs and symptoms related to the client's kidney failure

2. A client is scheduled for surgery for an abdominal hysterectomy. During the preoperative assessment, the client states, "I am very nervous and scared to have surgery." What client outcome is the priority?

Resolve the client's anxiety.

Which nursing diagnosis has the priority when caring for an older adult client with Alzheimer disease? Impaired Physical Mobility Risk for Injury Self-Care Deficit Impaired Memory

Risk for Injury

A client comes to a health care facility reporting abdominal pain and vomiting. The client's spouse informs the nurse that the client went out for dinner the previous night. The report that the client went out for dinner the previous night is example of data from which type of source?

Secondary

A client with newly diagnosed hypertension on BP medication has been taking her own BP at home for 2 weeks. When she calls and reports her BP readings to the nurse, the nurse notes an elevated BP in the morning. The client states that she wakes up, has her daily cup of coffee, and takes her BP before eating as she was instructed. What should the nurse recommend to this client?

Take her BP before drinking her morning cup of coffee.

The nurse prepares to take a temperature of a client admitted with a myocardial infarction. The client is eating breakfast. Which action should the nurse choose?

Take the temperature using the axillary route

A nursing diagnosis of "Complicated Grieving" has been identified for a client whose spouse died 1 year ago. What assessment data would be appropriate evidence to justify this diagnosis? Select all that apply. The client states, "I miss my wife every day." The client no longer indulges in usual activities. The client attempted suicide 1 month ago. The client keeps a picture of the client's wife at the bedside. The client states, "I have no interest in doing anything."

The client attempted suicide 1 month ago. The client states, "I have no interest in doing anything." The client no longer indulges in usual activities.

Which assessment findings would support the nursing diagnosis of Impaired Skin Integrity? Select all that apply. Impaired mobility due to recent stroke Unable to turn in bed without assistance Uncontrolled diabetes History of appendectomy Up with assistance to bedside commode

Unable to turn in bed without assistance Uncontrolled diabetes Impaired mobility due to recent stroke

A informatics nurse is working as part of a team that is working to develop a new electronic health record system that will be used in the facility. The team has thoroughly analyzed, planned, and designed an appropriate system and is planning to test the system. Which type of testing would the team do first? Unit Function Integration Performance

Unit

Several nurses are discussing their impressions of the newly implemented electronic health record with an informatics nurse specialist. They say, "There is so much information on one screen, it hard to tell what we should do first. It's not really clear." The informatics nurse specialist interprets the comments as reflecting an issue with which area? Usability Interoperability Optimization Security

Usability

When used in a nursing diagnosis, the descriptor "impaired" has which meaning? Weakened or damaged Consisting of many interconnecting parts or elements Late, slow, or postponed Lack of proportion or relation between corresponding things

Weakened or damaged

A nurse admitting a new client to the hospital needs to determine the client's needs and current problems. What is the priority action of the nurse? a) Contact the health care provider. b) Review the client's past medical records. c) Assist the client with activities of daily living. d )Complete an assessment.

d )Complete an assessment.

A client is diagnosed with bradycardia. For which symptom should the nurse assess first?

dizziness

An informatics nurse specialist is describing the role of informatics in health care to a group of staff at a facility. The nurse specialist determines that the teaching was successful when the group identifies which as a core practice area? electronic health record telehealth patient portals data analytics

electronic health record

The nurse formulates the nursing diagnosis: Disturbed Body Image related to decreased ability to cope with surgical removal of right breast as evidenced by client refuses to look at surgical site and client statement, "I'm ugly. My husband will no longer find me desirable." The decreased ability to cope with the removal of the breast is an example of: NANDA-I label. etiology. problem. defining characteristics.

etiology

Which pathologic condition would result in release of antidiuretic hormone (ADH) by the posterior pituitary?

hemorrhage

16.The nurse recognizes that identifying outcomes/goals must include:

involvement of the client and family.

A nurse makes a nursing diagnosis of Constipation after a client reports not defecating on the last trip to the bathroom. The nurse has no other information on the client's defecation history. This is an example of: inconsistent cues. premature closure. clustering of cues. cluster interpretation.

premature closure.

Which term indicates a potentially serious client condition?

pyrexia

10. A nurse is reviewing the plan of care for a client. Which statement would the nurse identify as an appropriate outcome?

"Client will identify one coping strategy to try by end of week."

25. A nurse is reviewing the plan of care for a client. Which statement would the nurse identify as an appropriate outcome?

"Client will identify one coping strategy to try by end of week."

40. A nurse is developing short-term outcomes for a client with a nursing diagnosis of "Deficient Knowledge related to insulin self-administration as evidenced by statements of therapy being new and never having done it before." When writing the outcomes, which verbs would the nurse use to achieve a psychomotor change in behavior? Select all that apply.

-Demonstrate -Choose

37. These nursing diagnoses appear on a client's care plan. Place in the order in which the nurse will prioritize acting upon them. Use all options.

-Impaired Swallowing -Fluid Volume Deficit -Risk for Impaired Skin Integrity -Altered Body Image

The nurse is providing discharge education for a client diagnosed with hypertension. Which teaching points about monitoring blood pressure should the nurse include in the plan? Select all that apply.

-Recommend taking the blood pressure every day at the same time. -Recommend a cuff size appropriate for the client's limb size. -If using a forearm monitor, tell the client to keep wrist at heart level when using it.

The nurse is caring for a client with a new diagnosis of cancer, and allows the client to verbalize fears relating to how to tell the children. The nurse's intervention reflects which aspect of nursing? A. art of nursing B. science of nursing C. evidence-based practice D. application of research

A. art of nursing

In order for the nurse to be able to apply the knowledge that has been obtained in the formal education process, which theory would best break whole concepts into parts and then determine how the parts would work together as a whole? A. Adaptation theory B. General systems theory C. Developmental theory D. Theory of human needs

B. General systems theory

Which group of nurses would be least likely involved in direct research? Select all that apply. A. Clinical Nurse Specialists B. Novice nurses C. Student nurses D. Nurse Practitioners E. Nurses at the PhD level

B. Novice nurses C. Student nurses

To obtain data about an adult client's sexuality and reproductive pattern, what question is best for the nurse to ask? a) "Has anything changed your sexual performance?" b) "How many children do you have, both living and dead?" c) "What arouses you when you have intercourse?" d) "How often do you have sexual intercourse?"

a) "Has anything changed your sexual performance?"

Which assessment findings would support the nursing diagnosis of Acute Pain? Select all that apply. The client had an abdominal hysterectomy 1 day ago. The client is crying in pain about 20 minutes before pain medicine is due. The client has a history of osteoarthritis. The client had back surgery 2 years ago and expresses the need for ibuprofen on most days. The client is a heavy cigarette smoker.

The client had an abdominal hysterectomy 1 day ago. The client is crying in pain about 20 minutes before pain medicine is due.

A nurse needs to count a client's heart rate. For which reason would the nurse assess the client's apical pulse?

The radial pulse is difficult to obtain.

A nurse is explaining to a nursing student why blood pressure is a frequently used assessment parameter in a wide variety of care settings. What can be inferred from an assessment of a client's blood pressure?

The resistance that the client's heart must overcome when pumping blood

Which is a focus of medical research rather than nursing research? A. Drug metabolism B. Health appraisal C. Prevention of trauma D. Promotion of recovery

A. Drug metabolism

A nurse who works in a pediatric practice assesses the developmental level of children of various ages to determine their psychosocial development. These assessments are based on the work of: A. Erikson. B. Maslow. C. Watson. D. Rogers.

A. Erikson.


संबंधित स्टडी सेट्स

ATI Chapter 33: RN Stress and Coping Quiz

View Set

Study Plan Questions - Marketing Test 2

View Set

Internal Medicine EORE- Vignettes

View Set